="http://www.w3.org/2000/svg" viewBox="0 0 512 512">

9 Metrische Räume

Dieses Kapitel am Anfang des zweiten Semesters stellt einen wichtigen Schritt der Verallgemeinerung dar. In der Tat möchten wir uns von der fast ausschliesslichen Betrachtung von Intervallen als «Grundräume» wegbewegen und allgemeinere Räume, wo Distanzen gemessen werden können, einführen. In diesem allgemeineren Rahmen werden wir einige wichtige Sätze rund um Konvergenz und Stetigkeit, die uns mitunter bereits für Intervalle bekannt sind, beweisen können. Dies stellt zum einen eine willkommene Wiederholung dar und verschafft uns zum anderen den notwendigen, breiteren Blickwinkel für die weitere Theorie wie zum Beispiel für den Beweis des Fundamentalsatzes der Algebra oder für die Fortsetzung der Diskussion von Differentialgleichungen.

9.1 – Konvergenz in metrischen Räumen

9.1.1 – Definition und erste Beispiele

Definition 9.1: Metrik

Ein metrischer Raum [latex](X,\mathrm {d})[/latex] ist eine Menge [latex]X[/latex] gemeinsam mit einer Abbildung [latex]\mathrm {d}: X \times X \to \mathbb {R}_{\geq 0}[/latex], die die Metrik auf [latex]X[/latex] genannt wird und die folgenden drei Eigenschaften erfüllt:

  • (Definitheit) Für alle [latex]x_1,x_2 \in X[/latex] gilt [latex]\mathrm {d}(x_1,x_2) = 0 \iff x_1 = x_2[/latex].
  • (Symmetrie) Für alle [latex]x_1,x_2 \in X[/latex] gilt [latex]\mathrm {d}(x_1,x_2) = \mathrm {d}(x_2,x_1)[/latex].
  • (Dreiecksungleichung) Für alle [latex]x_1,x_2,x_3 \in X[/latex] gilt [latex]\mathrm {d}(x_1,x_3) \leq \mathrm {d}(x_1,x_2) + \mathrm {d}(x_2,x_3)[/latex].

Intuitiv ausgedrückt weist eine Metrik [latex]\mathrm {d}[/latex] auf einer Menge [latex]X[/latex] je zwei Punkten ihre Distanz (ihren Abstand) zu. In dieser Auffassung besagt die Definitheit der Metrik, dass der einzige Punkt, der Abstand Null zu einem gegebenen Punkt [latex]x_1 \in X[/latex] hat, [latex]x_1[/latex] selbst ist. Symmetrie der Metrik besagt, dass der Abstand von [latex]x_1 \in X[/latex] zu [latex]x_2 \in X[/latex] der gleiche ist wie von [latex]x_2[/latex] zu [latex]x_1[/latex]. Fasst man die Distanz zwischen zwei Punkten als die Länge eines kürzesten Weges vom einen zum anderen Punkt auf (was nicht immer möglich ist), dann besagt die Dreiecksungleichung, dass die Länge eines kürzesten Weges von [latex]x_1[/latex] nach [latex]x_3[/latex] höchstens so gross ist wie die Länge eines Weges, den man abläuft, wenn man zuerst den Umweg nach [latex]x_2[/latex] und von dort aus nach [latex]x_3[/latex] geht.

Folgende Beispiele von Metriken sind uns eigentlich bereits bekannt — siehe Lemma 9.2 unten:

  • [latex]X= \mathbb {R}[/latex] mit der Standardmetrik [latex]\mathrm {d}[/latex] definiert durch [latex]\mathrm {d}(x_1,x_2) = |x_1-x_2|[/latex] für [latex]x_1,x_2 \in \mathbb {R}[/latex].
  • [latex]X= \mathbb {C}[/latex] mit der Standardmetrik [latex]\mathrm {d}[/latex] definiert durch [latex]\mathrm {d}(z_1,z_2) = |z_1-z_2|[/latex] für [latex]z_1,z_2 \in \mathbb {C}[/latex].
  • [latex]X= \mathbb {R}^d[/latex] mit der Einsmetrik [latex]\mathrm {d}_1[/latex] definiert durch [latex]\mathrm {d}_1(v_1,v_2) = \| v_1-v_2\| _1[/latex] für [latex]v_1,v_2 \in \mathbb {R}^d[/latex].
  • [latex]X= \mathbb {R}^d[/latex] mit der euklidischen Metrik [latex]\mathrm {d}_2[/latex] definiert durch [latex]\mathrm {d}_2(v_1,v_2) = \| v_1-v_2\| _2[/latex] für [latex]v_1,v_2 \in \mathbb {R}^d[/latex].
  • [latex]X= \mathbb {R}^d[/latex] mit der Maximumsmetrik [latex]\mathrm {d}_\infty[/latex] definiert durch [latex]\mathrm {d}_\infty (v_1,v_2) = \| v_1-v_2\| _\infty[/latex] für [latex]v_1,v_2 \in \mathbb {R}^d[/latex].

Für [latex]X=\mathbb {R}^d[/latex] werden wir im Normalfall die euklidische Metrik [latex]d_2[/latex] benützen und diese auch einfach mit [latex]\mathrm {d}=\mathrm {d}_2[/latex] bezeichnen. Da wir ab nun vermehrt Vektoren anstatt nur reelle oder komplexe Zahlen betrachten werden, werden wir diese einfach durch [latex]v\in \mathbb {R}^d[/latex] (anstatt [latex]\mathbf {v}\in \mathbb {R}^d[/latex]) bezeichnen.

Wie wir auch sehen werden, gibt es viele weitere, interessante Beispiele von metrischen Räumen. Manche dieser erhalten wir mittels Normen [latex]\| {\cdot }\|[/latex] auf Vektorräumen (siehe Definition 5.84).

Lemma 9.2: Eine Norm definiert eine Metrik

Sei [latex]V[/latex] ein Vektorraum über[1] [latex]\mathbb {R}[/latex] und [latex]\| {\cdot }\|[/latex] eine Norm auf [latex]V[/latex]. Dann definiert

[latex]
\begin{aligned}[]\mathrm {d}(v_1,v_2) = \mathrm {d}_{\| {\cdot }\| } (v_1,v_2) = \| {v_1-v_2}\|\end{aligned}
[/latex]

für [latex]v_1,v_2 \in V[/latex] eine Metrik [latex]\mathrm {d}[/latex] auf [latex]V[/latex], die man auch die von der Norm [latex]\| {\cdot }\|[/latex] induzierte Metrik auf [latex]V[/latex] nennt.

Beweis

Es gilt für [latex]v_1,v_2 \in V[/latex]

[latex]
\begin{aligned}[]\mathrm {d}_{\| {\cdot }\| } (v_1,v_2) = 0 & \iff \| {v_1-v_2}\| = 0\\ & \iff v_1 -v_2 = 0\\ & \iff v_1 = v_2\end{aligned}
[/latex]

nach Definitheit der Norm [latex]\| {\cdot }\|[/latex]. Nach Homogenität der Norm für [latex]\alpha = -1[/latex] gilt für [latex]v_1,v_2 \in V[/latex]

[latex]
\begin{aligned}[]\mathrm {d}_{\| {\cdot }\| } (v_1,v_2) &= \| {v_1-v_2}\| = \| {(-1)(v_2-v_1)}\| \\ &= \| {v_2-v_1}\| = \mathrm {d}_{\| {\cdot }\| } (v_2,v_1)\end{aligned}
[/latex]

und somit erhalten wir die Symmetrie von [latex]\mathrm {d}_{\| {\cdot }\| }[/latex]. Zuletzt verwenden wir die Dreiecksungleichung der Norm und erhalten

[latex]
\begin{aligned}[]\mathrm {d}_{\| {\cdot }\| } (v_1,v_3) &= \| {v_1-v_3}\| = \| {(v_1-v_2)+(v_2-v_3)}\| \\ &\leq \| {v_1-v_2}\| + \| {v_2-v_3}\| = \mathrm {d}_{\| {\cdot }\| } (v_1,v_2) + \mathrm {d}_{\| {\cdot }\| } (v_2,v_3)\end{aligned}
[/latex]

für alle [latex]v_1,v_2,v_3 \in V[/latex]. Dies zeigt die Dreiecksungleichung für [latex]\mathrm {d}_{\| {\cdot }\| }[/latex], womit also [latex]\mathrm {d}_{\| {\cdot }\| }[/latex] eine Metrik auf [latex]V[/latex] ist. ∎

Nicht jede Metrik auf einem Vektorraum muss durch eine Norm gegeben sein. Des Weiteren ist das Messen von Distanzen nicht nur auf Vektorräumen von Interesse. Interessante Beispiele dieser Art möchten wir nun besprechen.

Beispiel 9.3: Weitere metrische Räume

  1. (Diskrete Metriken) Sei [latex]X[/latex] eine Menge und [latex]\mathrm {d}_{\rm {diskret}}:X \times X \to \mathbb {R}_{\geq 0}[/latex] definiert durch
    [latex]
    \begin{aligned}[]\mathrm {d}_{\rm {diskret}}(x_1,x_2) = \left \lbrace \begin{array}{cc} 1 & \text {falls } x_1 \neq x_2 \\ 0 & \text {falls } x_1 = x_2\end{array} \right .\end{aligned}
    [/latex]

    für [latex]x_1,x_2 \in X[/latex]. Dann ist [latex](X,\mathrm {d}_{\rm {diskret}})[/latex] ein metrischer Raum. In der Tat ist [latex]\mathrm {d}_{\rm {diskret}}[/latex] definit und symmetrisch per Definition. Des Weiteren erfüllt [latex]\mathrm {d}[/latex] die Dreiecksungleichung: Seien [latex]x_1,x_2,x_3[/latex] Punkte in [latex]X[/latex]. Falls [latex]\mathrm {d}(x_1,x_3) = 0[/latex] gilt, dann ist [latex]\mathrm {d}(x_1,x_3) \leq \mathrm {d}(x_1,x_2) + \mathrm {d}(x_2,x_3)[/latex] trivialerweise erfüllt. Falls [latex]\mathrm {d}(x_1,x_3) = 1[/latex] gilt, dann ist [latex]x_1 \neq x_3[/latex] und [latex]x_2[/latex] ist mindestens von einem Punkt in [latex]\left \lbrace {x_1,x_3} \right \rbrace[/latex] verschieden und die Dreiecksungleichung gilt ebenso.

    Man beachte, dass die diskrete Metrik auf [latex]\mathbb {R}^d[/latex] für [latex]d \geq 2[/latex] nicht durch eine Norm gegeben ist. In der Tat würde eine Norm [latex]\| {\cdot }\|[/latex] mit [latex]\| {v_2-v_1}\| = \mathrm {d}_{\rm {diskret}}(v_1,v_2)[/latex] für alle [latex]v_1,v_2 \in \mathbb {R}^d[/latex] widersprüchlicherweise die Homogenitätseigenschaft in Definition 5.84 nicht erfüllen können.

  2. (Manhattanmetrik) Wir setzen [latex]X = [0,1]^2[/latex] und
    [latex]
    \begin{aligned}[]\mathrm {d}_{\rm {NY}}((x_1,y_1),(x_2,y_2)) = |x_1-x_2|+|y_1-y_2|\end{aligned}
    [/latex]

    für [latex](x_1,y_1),(x_2,y_2) \in [0,1]^2[/latex]. In der Tat erfüllt [latex]\mathrm {d}_{\rm {NY}}[/latex] alle Axiome einer Metrik auf [latex][0,1]^2[/latex], da [latex]\mathrm {d}_{\rm {NY}}=\mathrm {d}_1|_{X\times X}[/latex] die Einschränkung der Einsmetrik [latex]\mathrm {d}_1[/latex] von [latex]\mathbb {R}^2[/latex] auf [latex]X=[0,1]^2[/latex] ist. Die Metrik [latex]\mathrm {d}_{\rm {NY}}[/latex] wird oft auch Manhattan-Metrik genannt. Grund dafür ist, dass man in schachbrettartig angelegten Orten wie zum Beispiel Manhattan auf folgende Weise von [latex](x_1,y_1)[/latex] nach [latex](x_2,y_2)[/latex] gelangt: Man geht zuerst bei gleichbleibender [latex]y[/latex]-Koordinate von [latex](x_1,y_1)[/latex] nach [latex](x_2,y_1)[/latex] und dann bei gleichbleibender [latex]x[/latex]-Koordinate von [latex](x_2,y_1)[/latex] nach [latex](x_2,y_2)[/latex], oder umgekehrt von [latex](x_1,y_1)[/latex] nach [latex](x_1,y_2)[/latex] und dann von [latex](x_1,y_2)[/latex] nach [latex](x_2,y_2)[/latex]. Es gäbe zwar noch andere Möglichkeiten, aber wenn alle Strassen in Manhattan von West-Ost oder Nord-Süd verlaufen, dann misst [latex]\mathrm {d}_{\rm {NY}}[/latex] den relevanten Abstand zwischen zwei Punkten.

  3. (Metrik der französischen Eisenbahn) Wir setzen [latex]X = \mathbb {C}[/latex] und definieren die SNCF-Metrik [latex]\mathrm {d}_{\rm {SNCF}}[/latex] auf [latex]X[/latex] durch
    [latex]
    \begin{aligned}[]\mathrm {d}_{\rm {SNCF}}(z_1,z_2) = \left \lbrace \begin{array}{ll} |z_1-z_2| & \text {falls } z_1,z_2 \text { linear abhängig über } \mathbb {R} \text { sind} \\ |z_1|+|z_2| & \text {falls } z_1,z_2 \text { linear unabhängig über } \mathbb {R} \text { sind}\end{array} \right .\end{aligned}
    [/latex]

    für alle [latex]z_1,z_2 \in \mathbb {C}[/latex]. Der Grund für den Namen dieser Metrik (siehe Übung 9.4) ist, dass eine Bahnreise von einer französischen Stadt bei [latex]z_1[/latex] zu einer anderen bei [latex]z_2[/latex] meist über den Ursprung [latex]z=0[/latex] (auch Paris genannt) führt, ausser wenn [latex]z_1[/latex] und [latex]z_2[/latex] auf derselben — von Paris ausgehenden geraden Strecke liegen. Gewissermassen besteht [latex]\mathbb {C}[/latex] in dieser Metrik also aus unendlich vielen Halbgeraden, die sich nur im Ursprung treffen.

  4. Ein kombinatorischer Graph ist eine endliche Menge von Punkten, die sogenannten Ecken, von welchen einige mit sogenannten Kanten verbunden sind. Diese lassen sich auf natürliche Weise mit mehreren Metriken ausstatten; der Konkretheit halber betrachten wir einen spezifischen Graphen, doch muss ein Graph nicht unbedingt als Teilmenge von [latex]\mathbb {R}^d[/latex] für [latex]d\geq 2[/latex] gegeben sein.
    image

    Man kann nun eine Metrik auf den Ecken (durch [latex]\color{blue} \bullet[/latex] gekennzeichnet) dadurch definieren, dass man benachbarten Ecken die Distanz [latex]1[/latex] zuweist und dies iteriert. Beispielsweise definiert man die Distanz zweier Ecken, die man über zwei aber nicht weniger Kanten erreichen kann, als [latex]2[/latex]. Dazu notwendig ist, dass man von einer Ecke zu jeder anderen Ecke über Ablaufen von Kanten gelangen kann (wie bei obigem Graphen) — diese Eigenschaft nennt sich auch Zusammenhang des Graphen.

    Des Weiteren ist es auch möglich, eine Metrik auf dem kompletten (kontinuierlichen) Graphen zu definieren, indem man die obige Definition auf folgende Weise erweitert. Fasst man eine Kante als Kopie des Intervalles [latex][0,1][/latex] auf, wobei [latex]0[/latex] und [latex]1[/latex] die zwei Ecken der Kante sind, so kann man eine Distanz auf den Kanten über die Distanz [latex]|x-y|[/latex] auf [latex][0,1][/latex] definieren. Ähnlich wie oben kann man nun damit eine Metrik auf dem gesamten Graphen (inklusive den Kanten) definieren.

Weitere interessante Beispiele von Metriken finden Sie in Abschnitt 9.9.2.

Übung 9.4

Zeigen Sie, dass die in Beispiel 9.3(iii) definierten Metriken tatsächlich Metriken sind. Führen Sie des Weiteren die Konstruktion der Metriken in (iv) vollständig und formal durch.

Wir bemerken, dass gegeben eine Teilmenge [latex]Y[/latex] eines metrischen Raumes [latex](X,\mathrm {d})[/latex] die Einschränkung [latex]\mathrm {d}|_{Y \times Y}[/latex] eine Metrik auf [latex]Y[/latex] definiert. Wenn [latex]Y[/latex] mit dieser Metrik versehen ist, nennen wir [latex]Y[/latex] einen Teilraum des metrischen Raumes [latex](X,\mathrm {d})[/latex] und die Metrik [latex]\mathrm {d}|_{Y \times Y}[/latex] die induzierte Metrik. Wenn nicht anders spezifiziert, statten wir Teilmengen eines metrischen Raumes implizit mit der induzierten Metrik aus.

Wichtige Übung 9.5: Umgekehrte Dreiecksungleichung

Sei [latex](X,\mathrm {d})[/latex] ein metrischer Raum. Zeigen Sie, dass für alle [latex]x_1,x_2,y \in X[/latex] gilt

[latex]
\begin{aligned}[]|\mathrm {d}(x_1,y)-\mathrm {d}(x_2,y)| \leq \mathrm {d}(x_1,x_2).\end{aligned}
[/latex]

Übung 9.6: Deformation der Metrik

Sei [latex](X,\mathrm {d})[/latex] ein metrischer Raum. Zeigen Sie, dass

[latex]
\begin{aligned}[]\mathrm {d}_{1/2}(x,y)=\sqrt {\mathrm {d}(x,y)} \mbox { und }\tilde {\mathrm {d}}(x,y) = \frac {\mathrm {d}(x,y)}{1+\mathrm {d}(x,y)}\end{aligned}
[/latex]

für [latex]x,y \in X[/latex] zwei Metriken [latex]d_{\frac {1}{2}}[/latex], [latex]\tilde {d}[/latex] auf [latex]X[/latex] definieren.

9.1.2 – Konvergenz von Folgen

Zu Beginn von Kapitel 5 haben wir bereits Folgen in beliebigen Mengen definiert. Wir möchten nun Folgen in metrischen Räumen betrachten und für diese genauso wie für Folgen in [latex]\mathbb {R}[/latex] oder [latex]\mathbb {C}[/latex] einen Konvergenzbegriff einführen.

Definition 9.7: Konvergenz

Sei [latex](X,\mathrm {d})[/latex] ein metrischer Raum und [latex](x_n)_n[/latex] eine Folge in [latex]X[/latex]. Wir sagen, dass [latex](x_n)_n[/latex] gegen einen Punkt [latex]x_0 \in X[/latex] konvergiert, falls es für jedes [latex]\varepsilon > 0[/latex] ein [latex]N \in \mathbb {N}[/latex] gibt, so dass [latex]\mathrm {d}(x_n,x_0) Grenzwert der Folge [latex](x_n)_n[/latex]. Wir sagen, dass eine Folge [latex](x_n)_n[/latex] mit [latex]x_n\in X[/latex] für alle [latex]n\in \mathbb {N}[/latex] in [latex]X[/latex] konvergent ist falls es ein [latex]x_0\in X[/latex] mit [latex]\lim _{n \to \infty } x_n = x_0[/latex] gibt.

Wir bemerken zuerst, dass [latex]x_n \to x_0[/latex] für [latex]n \to \infty[/latex] für eine Folge [latex](x_n)_n[/latex] und einen Punkt [latex]x_0 \in X[/latex] in einem metrischen Raum [latex](X,\mathrm {d})[/latex] per Definition zur Konvergenz [latex]\mathrm {d}(x_n,x_0) \to 0[/latex] in [latex]\mathbb {R}[/latex] für [latex]n \to \infty[/latex] äquivalent ist (wieso?).

Des Weiteren ist der Grenzwert einer Folge eindeutig bestimmt, wenn er existiert, was mit dem gleichen Argument wie für reelle Folgen gezeigt wird. (Wie?

Falls sowohl [latex]x_0[/latex] als auch [latex]x_0'[/latex] Grenzwerte einer Folge [latex](x_n)_n[/latex] in [latex]X[/latex] sind, dann gilt [latex]\mathrm {d}(x_0,x_0')\leq \mathrm {d}(x_0,x_n)+\mathrm {d}(x_n,x_0')[/latex] für alle [latex]n\in \mathbb {N}[/latex]. Da aber [latex]\mathrm {d}(x_0,x_n)+\mathrm {d}(x_n,x_0')[/latex] gegen Null strebt für [latex]n\to \infty[/latex], folgt [latex]\mathrm {d}(x_0,x_0')=0[/latex] und daher [latex]x_0=x_0'[/latex] auf Grund der Definitheit der Metrik.

) Man darf somit in der Tat von dem Grenzwert sprechen.

Wir bemerken auch, dass Definition 9.7 kompatibel ist mit dem Konvergenzbegriff für normierte Räume (Definition 5.91) in folgendem Sinne. Sei [latex]V[/latex] ein Vektorraum über [latex]\mathbb {R}[/latex] und [latex]\| {\cdot }\|[/latex] eine Norm auf [latex]V[/latex]. Ist [latex](v_n)_n[/latex] eine Folge in [latex]V[/latex], dann konvergiert [latex](v_n)_n[/latex] gegen ein [latex]v_0\in V[/latex] genau dann bezüglich der Norm [latex]\| {\cdot }\|[/latex], wenn [latex](v_n)_n[/latex] bezüglich der induzierten Metrik gegen [latex]v_0[/latex] konvergiert.

Definition 9.8: Offene Bälle und Umgebungen

Sei [latex](X,\mathrm {d})[/latex] ein metrischer Raum. Für ein [latex]\varepsilon > 0[/latex] und einen Punkt [latex]x_0 \in X[/latex] nennt man

[latex]
\begin{aligned}[]B_\varepsilon (x_0) = \left \lbrace {x \in X} \mid {\mathrm {d}(x,x_0) [/latex]

den offenen Ball mit Radius [latex]\varepsilon[/latex] um [latex]x_0[/latex] oder auch die [latex]\varepsilon[/latex]–Umgebung von [latex]x_0[/latex]. Eine Umgebung von [latex]x_0 \in X[/latex] ist eine Teilmenge [latex]V \subseteq X[/latex], die eine [latex]\varepsilon[/latex]-Umgebung von [latex]x_0[/latex] für ein [latex]\varepsilon >0[/latex] enthält.

Wichtige Übung 9.9: Offene Bälle

Beschreiben Sie die offenen Bälle in folgenden metrischen Räumen.

  • [latex](X,\mathrm {d}_{\rm {diskret}})[/latex] für eine Menge [latex]X[/latex] und die diskrete Metrik [latex]\mathrm {d}_{\rm {diskret}}[/latex] auf [latex]X[/latex].
  • [latex][0,1]^2[/latex] mit der Manhattanmetrik.
  • [latex]\mathbb {C}[/latex] mit der französischen Eisenbahnmetrik.
  • [latex]\mathbb {R}^2[/latex] mit der Metrik [latex]\mathrm {d}_\infty[/latex] gegeben durch [latex]\mathrm {d}_\infty ((x_1,y_1),(x_2,y_2)) = \max \left \lbrace {|x_1-x_2|,|y_1-y_2|} \right \rbrace[/latex] für [latex](x_1,y_1),(x_2,y_2)\in \mathbb {R}^2[/latex].

Die obige Definition von Umgebungen erlaubt nun eine alternative Formulierung von Konvergenz: Eine Folge [latex](x_n)_n[/latex] in einem metrischen Raum [latex](X,\mathrm {d})[/latex] konvergiert genau dann gegen [latex]x_0 \in X[/latex], wenn für jede Umgebung [latex]V \subseteq X[/latex] von [latex]x_0[/latex] fast alle (das heisst, alle bis auf endlich viele) Folgenglieder von [latex](x_n)_n[/latex] in [latex]V[/latex] liegen (wieso?).

Folgende Übung zeigt, dass der Konvergenzbegriff von der Wahl der Metrik abhängt.

Übung 9.10: Manhattan und SNCF sind sehr verschieden

Sei [latex]X = [0,1]^2[/latex]. Finden Sie eine Folge in [latex]X[/latex], die zwar bezüglich der Manhattanmetrik, aber nicht bezüglich der französischen Eisenbahnmetrik konvergiert (wobei wir [latex]\mathbb {R}^2[/latex] mit [latex]\mathbb {C}[/latex] und damit [latex]X[/latex] mit einer Teilmenge von [latex]\mathbb {C}[/latex] identifizieren).

9.1.3 – Der Raum der stetigen Funktionen

Wir kennen bereits einige Normen auf endlich-dimensionalen Vektorräumen und werden im Verlaufe dieses Kapitels weitere kennenlernen. Für die Analysis sind allerdings nicht nur endlich-dimensionale normierte Vektorräume interessant, sondern oft auch unendlich-dimen­sion­ale. Häufig (zum Beispiel bei der Diskussion von Differentialgleichungen) werden dabei sogenannte Funktionenräume untersucht.

Beispiel 9.11: Supremumsnorm und Gleichmässige Konvergenz

Sei [latex]K = [a,b][/latex] ein kompaktes Intervall mit [latex]a

[latex]
\begin{aligned}[]\| {f}\| _\infty = \sup _{x \in K} |f(x)| = \max _{x \in K} |f(x)|\end{aligned}
[/latex]

für [latex]f \in C(K)[/latex]. Dann ist [latex]\| {\cdot }\| _\infty[/latex] eine Norm auf [latex]C(K)[/latex]. Es gilt Definitheit, denn für alle [latex]f \in C(K)[/latex] ist [latex]\| {f}\| _\infty \geq 0[/latex] per Definition von [latex]\| {\cdot }\| _\infty[/latex] und [latex]\| {f}\| _\infty = 0[/latex] genau dann, wenn [latex]f =0[/latex]. Für [latex]\alpha \in \mathbb {C}[/latex] und [latex]f \in C(K)[/latex] gilt des Weiteren

[latex]
\begin{aligned}[]\| {\alpha f}\| _\infty = \max _{x \in K} |\alpha | |f(x)| = |\alpha | \max _{x \in K} |f(x)| = |\alpha | \| {f}\| _\infty\end{aligned}
[/latex]

und somit Homogenität von [latex]\| {\cdot }\| _\infty[/latex]. Schlussendlich gilt für [latex]f_1,f_2 \in C(K)[/latex] auch

[latex]
\begin{aligned}[]\| {f_1+f_2}\| _\infty &= \max _{x \in K} |f_1(x) + f_2(x)|\\ &\leq \max _{x \in K} \big (|f_1(x)|+|f_2(x)|\big )\\ &\leq \max _{x \in K} |f_1(x)|+ \max _{x \in K}|f_2(x)|= \| {f_1}\| _\infty +\| {f_2}\| _\infty ,\end{aligned}
[/latex]

womit wir die Dreiecksungleichung bewiesen haben und gezeigt haben, dass [latex]\| {\cdot }\| _\infty[/latex] eine Norm auf [latex]V[/latex] ist.

Wir beschreiben nun Konvergenz in [latex]C(K)[/latex] mit der von [latex]\| {\cdot }\| _\infty[/latex] induzierten Metrik. Sei [latex](f_n)_n[/latex] eine Folge in [latex]C(K)[/latex] und [latex]f_0\in C(K)[/latex]. Die Funktionenfolge [latex](f_n)_n[/latex] konvergiert bezüglich der von [latex]\| {\cdot }\| _\infty[/latex] induzierten Metrik genau dann gegen [latex]f_0[/latex], wenn [latex](f_n)_n[/latex] gleichmässig gegen [latex]f_0[/latex] konvergiert. In der Tat, falls [latex]\| {f_n-f_0}\| _\infty \to 0[/latex] für [latex]n \to \infty[/latex] und [latex]\varepsilon > 0[/latex] ist, dann existiert ein [latex]N[/latex] mit [latex]\| {f_n-f_0}\| _\infty

Übung 9.12: [latex]1[/latex]-Norm und Konvergenz im Mittel

Sei [latex]K = [a,b][/latex] ein kompaktes Intervall mit [latex]a

[latex]
\begin{aligned}[]\| {f}\| _1 = \int _a^b |f(x)| \thinspace {\rm {d}} x\end{aligned}
[/latex]

für [latex]f \in V[/latex].

  1. Zeigen Sie, dass [latex]\| {\cdot }\| _1[/latex] eine Norm auf [latex]V[/latex] ist. Konvergenz in [latex]V[/latex] bezüglich [latex]\| {\cdot }\| _1[/latex] nennt man Konvergenz im Mittel.
  2. Sei [latex](f_n)_n[/latex] eine Folge in [latex]V[/latex]. Zeigen Sie, dass Konvergenz [latex]f_n \to f[/latex] für [latex]n \to \infty[/latex] bezüglich [latex]\| {\cdot }\| _\infty[/latex] für ein [latex]f \in V[/latex] auch die Konvergenz [latex]f_n \to f[/latex] für [latex]n \to \infty[/latex] bezüglich [latex]\| {\cdot }\| _1[/latex] impliziert.
  3. Finden Sie eine Folge [latex](f_n)_n[/latex] in [latex]V[/latex] mit [latex]\| {f_n}\| _1 \to 0[/latex] für [latex]n \to \infty[/latex] und [latex]\| {f_n}\| _\infty =1[/latex] für alle [latex]n \in \mathbb {N}[/latex].

9.1.4 – Ein kurzer Überblick

Wir fassen die behandelten Begriffe nochmals in einem Diagram zusammen.

image

Wir werden uns in diesem zweiten Semester vor allem mit [latex]\mathbb {R}^d[/latex] für [latex]d \geq 2[/latex] (mehrdimensionale Analysis) beschäftigen; allerdings werden wir auch [latex]C([a,b])[/latex] mit [latex]\| {\cdot }\| _\infty[/latex] wie in Beispiel 9.11 oder gewisse Teilmengen von [latex]\mathbb {R}^d[/latex] verwenden. Insbesondere bieten metrische Räume den für uns geeigneten allgemeinen Rahmen.

9.1.5 – Euklidische Norm

Bei Betrachtung von [latex]\mathbb {R}^d[/latex] (und Teilmengen von [latex]\mathbb {R}^d[/latex]) werden wir meist die Norm [latex]\| {\cdot }\| _2[/latex] (und die von der Norm [latex]\| {\cdot }\| _2[/latex] auf einer Teilmenge induzierte Metrik) verwenden. Der Grund dafür ist naheliegend, denn diese Norm hat eine klare geometrische Bedeutung, die im Laufe des Semesters zunehmend an Bedeutung gewinnen wird. In der Tat stellt die Definition

[latex]
\begin{aligned}[]\| {v}\| _2=\sqrt {v_1^2+\cdots +v_d^2}\end{aligned}
[/latex]

für [latex]v\in \mathbb {R}^d[/latex] eine mehrdimensionale Verallgemeinerung des Satzes von Pythagoras (siehe die entsprechende Übung in Abschnitt 1.7.6) dar. Des Weiteren können wir für zwei nicht-verschwin­dende Vektoren [latex]v,w\in \mathbb {R}^d\setminus \{ 0\}[/latex] die Cauchy-Schwarz-Ungleichung in Proposition 5.86 als die Aussage interpretieren, dass sich das innere Produkt in der Form

[latex]
\begin{aligned}[]\left \langle {{v}}, {{w}} \right \rangle =\| {{v}}\| \| {{w}}\| \cos \varphi\end{aligned}
[/latex]

schreiben lässt, wobei wir [latex]\varphi =\arccos \frac {\left \langle {{v}}, {{w}} \right \rangle }{\| {{v}}\| \| {{w}}\| }\in [0,\pi ][/latex] als den Winkel zwischen den Vektoren [latex]v[/latex] und [latex]w[/latex] bezeichnen. Wir empfehlen Ihnen, diese Definition an einigen expliziten Beispielen durchzuspielen und sich davon zu überzeugen, dass diese Definition des Winkels den Ihnen intuitiv bekannten Eigenschaften genügt.

Schlussendlich werden wir uns in diesem Semester auch intensiv mit Volumen von Teilmengen von [latex]\mathbb {R}^d[/latex] beschäftigen. Auch hier spielt die euklidischen Norm und das innere Produkt eine fundamentale Rolle, da wir das Volumen des «[latex]d[/latex]-dimensionalen Einheitswürfel» , der von [latex]d[/latex] orthogonalen Vektoren der Länge [latex]1[/latex] aufgespannt wird, als [latex]1[/latex] definieren werden.

9.1.6 – Wie sehen metrische Räume aus?

Da metrische Räume sozusagen geometrische Objekte darstellen, drängt sich vielleicht die Frage im Titel des Unterabschnittes auf. Doch ist diese Frage genauso wenig sinnvoll wie zum Beispiel die Frage «Welche Eigenschaften haben chemische Elemente?» . In beiden Fällen hängt die Antwort stark vom betrachteten Spezialfall ab. Zum Beispiel inkludiert die Frage «Wie sehen metrische Räume aus?» auch die Frage «Wie sehen Teilmengen von [latex]\mathbb {R}^d[/latex] aus?» , denn jede Teilmenge von [latex]\mathbb {R}^d[/latex] kann als eigenständiger metrischer Raum (mit der induzierten Metrik) betrachtet werden. Insbesondere hat auf Grund ihrer Allgemeinenheit diese Frage kaum eine vernünftige Antwort.

Damit Sie sich von der Vielfalt der Antwortmöglichkeiten ein besseres Bild machen können, betrachten wir in den folgenden Applets offene Bälle in einigen wenigen metrische Räumen.

Applet 9.13: Bälle in einigen metrischen Räumen

Die folgenden Apps sollten helfen die Vielfalt der Möglichkeiten für die Gestalt von Bällen in metrischen Räumen visualisieren.

9.2 – Topologische Grundbegriffe

9.2.1 – Offene und abgeschlossene Teilmengen

Aus Abschnitt 2.4 kennen wir bereits die offenen Teilmengen von [latex]\mathbb {R}[/latex]. Dies sind im Wesentlichen jene Teilmengen [latex]U \subseteq \mathbb {R}[/latex], so dass für jedes [latex]x \in U[/latex] Punkte «nahe genug an [latex]x[/latex]» wieder in [latex]U[/latex] liegen. Wir führen nun das analoge Konzept für metrische Räume ein.

Definition 9.14: Offene oder abgeschlossene Teilmengen

Sei [latex](X,\mathrm {d})[/latex] ein metrischer Raum. Eine Teilmenge [latex]O \subseteq X[/latex] heisst offen (in [latex]X[/latex]), falls es zu jedem Punkt in [latex]O[/latex] einen offenen Ball um diesen Punkt gibt, der in [latex]O[/latex] liegt. Eine Teilmenge [latex]A \subseteq X[/latex] heisst abgeschlossen (in [latex]X[/latex]), falls ihr Komplement [latex]X \setminus A[/latex] offen ist.

Die leere Menge und die ganze Menge [latex]X[/latex] sind stets sowohl offene als auch abgeschlossene Teilmengen eines metrischen Raumes [latex](X,\mathrm {d})[/latex]. Es kann aber auch weitere (sogenannte abgeschloffene) Teilmengen von [latex]X[/latex] geben, die sowohl offen als auch abgeschlossen sind (siehe Abschnitt 9.2.3).

Beispiel 9.15: Offene oder abgeschlossene Teilmengen

  1. Sei [latex](X,\mathrm {d})[/latex] ein metrischer Raum. Sei [latex]x_0 \in X[/latex] ein Punkt und [latex]r > 0[/latex] ein Radius. Der Ball [latex]B_r(x_0) = \left \lbrace {x \in X} \mid {\mathrm {d}(x,x_0) 0[/latex]. Der Ball mit Radius [latex]\varepsilon[/latex] um [latex]x[/latex] ist damit in [latex]B_r(x_0)[/latex] enthalten, denn es gilt für [latex]y \in B_\varepsilon (x)[/latex] per Definition [latex]\mathrm {d}(y,x)
  2. Sei [latex]A = \left \lbrace {x_1,\ldots ,x_n} \right \rbrace \subseteq X[/latex] eine endliche Teilmenge eines metrischen Raumes [latex](X,\mathrm {d})[/latex]. Dann ist [latex]A[/latex] abgeschlossen. Sei [latex]x \in X \setminus A[/latex] und sei [latex]r = \min \left \lbrace {\mathrm {d}(x,x_1),\ldots ,\mathrm {d}(x,x_n)} \right \rbrace > 0[/latex]. Dann ist der Ball [latex]B_r(x)[/latex] in [latex]X \setminus A[/latex] enthalten, denn für [latex]k \in \left \lbrace {1,\ldots ,n} \right \rbrace[/latex] gilt [latex]\mathrm {d}(x,x_k) \geq r[/latex] und somit [latex]x_k \not \in B_r(x)[/latex]. Also ist [latex]X\setminus A[/latex] offen und [latex]A[/latex] abgeschlossen.
  3. Wir betrachten [latex]X=[-1,1][/latex] als Teilraum von [latex]\mathbb {R}[/latex]. Die Teilmenge [latex](0,1][/latex] von [latex]X[/latex] ist offen, da [latex](0,1][/latex] aus den Punkten in [latex][-1,1][/latex] mit Abstand kleiner [latex]1[/latex] von [latex]1[/latex] besteht. Anders ausgedrückt ist [latex](0,1][/latex] der Ball von Radius [latex]1[/latex] um [latex]1[/latex] im metrischen Raum [latex]X=[-1,1][/latex] und somit nach (i) offen. Betrachtet man stattdessen die Teilmenge [latex][-1,0) \cup (0,1] = [-1,1] \setminus \left \lbrace {0} \right \rbrace[/latex], so ist diese nach (ii) offen, da [latex]\left \lbrace {0} \right \rbrace[/latex] eine abgeschlossene Teilmenge von [latex][-1,1][/latex] ist und ihr Komplement somit offen ist. Wir bemerken, dass für diese Diskussion die Definition des Grundraum [latex]X=[-1,1][/latex] mehrmals verwendet wurde (siehe auch Lemma 9.20 und die anschliessende Diskussion).
  4. Wir betrachten den Raum [latex]X = \left \lbrace {\frac {1}{n}} \mid {n\in \mathbb {N}}\right \rbrace \cup \left \lbrace {0} \right \rbrace[/latex] als Teilraum von [latex]\mathbb {R}[/latex].
    • Die Teilmenge [latex]Y_1 = \left \lbrace {\frac {1}{m}} \mid {m\geq n}\right \rbrace \cup \left \lbrace {0} \right \rbrace[/latex] von [latex]X[/latex] ist für jedes feste [latex]n \in \mathbb {N}[/latex] offen, da [latex]X \setminus Y_1[/latex] endlich ist. Sie ist auch abgeschlossen, da es zu jedem [latex]y \in X\setminus Y_1[/latex] ein [latex]\varepsilon > 0[/latex] mit [latex]\frac {1}{n}
    • Die Teilmenge [latex]Y_2 = \left \lbrace {\frac {1}{m}} \mid {m\geq n}\right \rbrace[/latex] von [latex]X[/latex] ist nicht abgeschlossen. Tatsächlich ist [latex]X \setminus Y_2[/latex] nicht offen, da zu jedem [latex]\varepsilon > 0[/latex] ein [latex]m \in \mathbb {N}[/latex] mit [latex]|\frac {1}{m}-0|

Übung 9.16: Abgeschlossener Ball

Sei [latex](X,\mathrm {d})[/latex] ein metrischer Raum. Zeigen Sie, dass der abgeschlossene Ball [latex]\left \lbrace {x \in X} \mid {\mathrm {d}(x,x_0) \leq r}\right \rbrace[/latex] mit Radius [latex]r > 0[/latex] um [latex]x_0 \in X[/latex] tatsächlich eine abgeschlossene Teilmenge von [latex]X[/latex] ist.

image

Abbildung 9.1 – Offene Teilmengen kann man sich als jene Teilmengen vorstellen, die keinen Punkt ihres Randes enthalten und gewissermassen dick sind (wobei wir auf die Definition des Rand noch verzichten wollen). Dies ist im linken Bild illustriert, wo [latex]O[/latex] eine offene Menge (ein Quadrat) im [latex]\mathbb {R}^2[/latex] darstellt und die gestrichelte Linie die Abwesenheit der Randpunkte andeutet. Fügt man Punkte des Randes hinzu, so ist die Menge im Allgemeinen nicht mehr offen (mittleres Bild). Fügt man den kompletten Rand hinzu, so erhält man eine abgeschlossene Menge (rechtes Bild). Abgeschlossene Mengen brauchen im Allgemeinen nicht «dick» zu sein; beispielsweise sind Geraden im [latex]\mathbb {R}^2[/latex] abgeschlossen.

Lemma 9.17: Topologie

Sei [latex](X, \mathrm {d})[/latex] ein metrischer Raum. Dann gilt

  • Der Durchschnitt endlich vieler offener Mengen ist offen.
  • Die Vereinigung beliebig vieler offener Mengen ist offen.
  • Der Durchschnitt beliebig vieler abgeschlossener Mengen ist abgeschlossen.
  • Die Vereinigung endlich vieler abgeschlossener Mengen ist abgeschlossen.

Die Familie der offenen Teilmengen eines metrischen Raumes [latex](X,\mathrm {d})[/latex] wird die von der Metrik induzierte Topologie auf [latex]X[/latex] genannt.

Nach Lemma 9.17 ist also beispielsweise

[latex]
\begin{aligned}[]\bigcup _{n\in \mathbb {Z}}(2n,2n+1) = \ldots \cup (-2,-1) \cup (0,1) \cup (2,3) \cup (4,5) \cup \ldots\end{aligned}
[/latex]

eine offene Teilmenge von [latex]\mathbb {R}[/latex] mit der Euklidschen Metrik, da für jedes [latex]n \in \mathbb {Z}[/latex] das Intervall [latex](2n,2n+1)[/latex] der Ball um [latex]2n+\frac {1}{2}[/latex] ist und somit offen ist.

Beweis

Seien [latex]O_1,\ldots ,O_n \subseteq X[/latex] offen und sei [latex]x \in \bigcap _{k=1}^n O_k[/latex]. Dann gibt es für jedes [latex]k\in \left \lbrace {1,\ldots ,n} \right \rbrace[/latex] einen Radius [latex]\varepsilon _k > 0[/latex] mit [latex]B_{\varepsilon _k}(x) \subseteq O_k[/latex], da [latex]x[/latex] in [latex]O_k[/latex] liegt und [latex]O_k[/latex] offen ist per Annahme. Für den Radius [latex]\varepsilon = \min _{k=1,\ldots ,n } \varepsilon _k > 0[/latex] gilt nun [latex]B_\varepsilon (x) \subseteq B_{\varepsilon _k}(x) \subseteq O_k[/latex] für jedes [latex]k\in \left \lbrace {1,\ldots ,n} \right \rbrace[/latex] und somit [latex]B_\varepsilon (x) \subseteq \bigcap _{k=1}^n O_k[/latex]. Da [latex]x \in \bigcap _{k=1}^n O_k[/latex] beliebig ist, schliessen wir, dass [latex]\bigcap _{k=1}^n O_k[/latex] offen ist.

Sei nun [latex]\mathcal {I}[/latex] eine beliebige Menge und für jedes [latex]\alpha \in \mathcal {I}[/latex] eine offene Teilmenge [latex]O_\alpha[/latex] von [latex]X[/latex] gegeben. Für jedes [latex]x \in \bigcup _{\alpha \in \mathcal {I}} O_\alpha[/latex] existiert ein [latex]\beta \in \mathcal {I}[/latex] mit [latex]x \in O_\beta[/latex]. Da [latex]O_\beta \subseteq X[/latex] per Annahme offen ist, existiert ein Radius [latex]\varepsilon >0[/latex] mit [latex]B_\varepsilon (x) \subseteq O_\beta \subseteq \bigcup _{\alpha \in \mathcal {I}} O_\alpha[/latex]. Dies beweist, dass [latex]\bigcup _{\alpha \in \mathcal {I}} O_\alpha[/latex] offen ist.

Die beiden Aussagen über abgeschlossene Teilmengen ergeben sich nun aus den ersten zwei Punkten und den Gesetzen von De Morgan. Sind beispielsweise [latex]A_1,\ldots ,A_n \subseteq X[/latex] abgeschlossen, dann ist für jedes [latex]k \in \left \lbrace {1,\ldots ,n} \right \rbrace[/latex] die Teilmenge [latex]O_k = X \setminus A_k[/latex] offen und somit ist nach dem ersten Punkt [latex]\bigcap _{k=1}^n O_k[/latex] offen. Also ist [latex]X \setminus \bigcap _{k=1}^n O_k = \bigcup _{k=1}^n A_k[/latex] abgeschlossen. ∎

Betrachten wir [latex]X=\mathbb {R}^d[/latex] für [latex]d \geq 1[/latex] und verschiedene, von (uns bekannten) Normen induzierte Metriken auf [latex]\mathbb {R}^d[/latex], so erhalten wir auf Grund der Normäquivalenz jeweils dieselbe induzierte Topologie. Diese nennt man auch die Standardtopologie auf [latex]\mathbb {R}^d[/latex].

Übung 9.18: Äquivalente Normen

Sei [latex]\mathrm {d}_\infty[/latex] die von der Maximumsnorm auf [latex]\mathbb {R}^2[/latex] induzierte Metrik, [latex]\mathrm {d}_1[/latex] die von der [latex]1[/latex]-Norm auf [latex]\mathbb {R}^2[/latex] induzierte Metrik und [latex]\mathrm {d}_2[/latex] die Euklidsche Metrik. Zeigen Sie, dass die von [latex]\mathrm {d}_\infty ,\mathrm {d}_1[/latex] respektive [latex]\mathrm {d}_2[/latex] induzierten Topologien gleich sind. Formulieren und beweisen Sie die analoge, verallgemeinerte Aussage für äquivalente Normen auf einem Vektorraum.

Lemma 9.19: Charakterisierung offener und abgeschlossener Mengen durch Konvergenz

Sei [latex](X,\mathrm {d})[/latex] ein metrischer Raum.

  • Eine Teilmenge [latex]O \subseteq X[/latex] ist genau dann offen, wenn für jede konvergente Folge in [latex]X[/latex] mit Grenzwert in [latex]O[/latex] fast alle Folgenglieder in [latex]O[/latex] liegen.
  • Eine Teilmenge [latex]A \subseteq X[/latex] ist genau dann abgeschlossen, wenn für jede konvergente Folge [latex](x_n)_n[/latex] in [latex]X[/latex] mit [latex]x_n \in A[/latex] für alle [latex]n \in \mathbb {N}[/latex] auch der Grenzwert in [latex]A[/latex] liegt.
Beweis

Sei [latex]O \subseteq X[/latex] eine offene Teilmenge von [latex]X[/latex] und [latex](x_n)_n[/latex] eine Folge in [latex]X[/latex] mit Grenzwert [latex]x_0[/latex] in [latex]O[/latex]. Dann ist [latex]O[/latex] eine Umgebung von [latex]x_0[/latex], da [latex]O[/latex] offen ist und somit liegen fast alle Folgenglieder von [latex](x_n)_n[/latex] in [latex]O[/latex].

Sei nun [latex]O \subseteq X[/latex] eine nicht offene Teilmenge. Dann gibt es einen Punkt [latex]x_0 \in O[/latex] mit [latex]B_\varepsilon (x_0) \setminus O \neq \emptyset[/latex] für jedes [latex]\varepsilon > 0[/latex]. (Wieso?

Dies ist einfach die Negation der Definition 9.14 von Offenheit.

) Für jedes [latex]n \in \mathbb {N}[/latex] und [latex]\varepsilon = \tfrac {1}{n}[/latex] finden wir somit ein [latex]x_n \in B_{1/n}(x_0) \setminus O[/latex]. Die Folge [latex](x_n)_n[/latex] in [latex]X\setminus O[/latex] ist nun konvergent mit Grenzwert [latex]x_0 \in O[/latex], da [latex]\mathrm {d}(x_n,x_0)

Sei [latex]A \subseteq X[/latex] abgeschlossen und sei [latex](x_n)_n[/latex] eine konvergente Folge in [latex]X[/latex] mit [latex]x_n \in A[/latex] für alle [latex]n \in \mathbb {N}[/latex]. Sei [latex]x_0[/latex] der Grenzwert der Folge [latex](x_n)_n[/latex]. Dann ist [latex]O = X \setminus A[/latex] offen und kann nicht den Grenzwert [latex]x_0[/latex] von [latex](x_n)_n[/latex] enthalten, da sonst fast alle Folgenglieder der Folge [latex](x_n)_n[/latex] in [latex]O[/latex] liegen müssten. Also ist der Grenzwert [latex]x_0[/latex] in [latex]A[/latex].

Sei schlussendlich [latex]A \subseteq X[/latex] nicht abgeschlossen. Dann ist [latex]O = X \setminus A[/latex] nicht offen und es existiert nach obigem Argument eine Folge [latex](x_n)_n[/latex] in [latex]A=X\setminus O[/latex] mit Grenzwert [latex]x_0 \in O[/latex]. ∎

Für einen metrischen Raum [latex](X,\mathrm {d})[/latex] und eine Teilmenge [latex]Y\subseteq X[/latex] haben wir bereits erwähnt, dass wir [latex]Y[/latex] oft auch als einen metrischen Raum betrachten indem wir die Metrik von [latex]X[/latex] einfach auf [latex]Y[/latex] einschränken. Insbesondere erhalten wir damit eine Definition von offenen Teilmengen von [latex]Y[/latex]. Ebenso könnten wir aber auch zuerst offene Teilmengen [latex]O\subseteq X[/latex] definieren und anschliessend dies verwenden um offene Teilmenge von [latex]Y[/latex] als Durchschnitte [latex]O\cap Y[/latex] zu definieren. Dies führt allerdings zu demselben Begriff von offenen Teilmengen von [latex]Y[/latex].

Lemma 9.20: Relativ offene oder abgeschlossene Teilmengen

Sei [latex](X,\mathrm {d})[/latex] ein metrischer Raum und [latex]Y\subseteq X[/latex] ein Teilraum. Eine Teilmenge von [latex]Y[/latex] ist offen (bezüglich der induzierten Metrik) genau dann wenn sie die Form [latex]O\cap Y[/latex] hat, wobei [latex]O\subseteq X[/latex] eine offene Teilmenge in [latex]X[/latex] ist. Ebenso ist eine Teilmenge von [latex]Y[/latex] abgeschlossen genau dann wenn sie die Form [latex]A\cap Y[/latex] hat, wobei [latex]A\subseteq X[/latex] eine abgeschlossene Teilmenge ist.

Wir sagen in diesem Zusammenhang auch, dass [latex]O\cap Y[/latex] in [latex]Y[/latex] offen oder relativ offen und [latex]A\cap Y[/latex] in [latex]Y[/latex] abgeschlossen oder relativ abgeschlossen ist. Wenn [latex]Y[/latex] eine offene Teilmenge von [latex]X[/latex] ist, so ist eine relativ offene Teilmenge von [latex]Y[/latex] auch in [latex]X[/latex] offen. Dies gilt analog ebenso für abgeschlossene und relativ abgeschlossene Teilmengen.

Man glaubt vielleicht zuerst, dass obiges ein rein formales Wortspiel ist und eigentlich wenig Zweck erfüllt. Das dem nicht so ist, sollte klar sein, wenn Sie Teilmengen von der Kugeloberfläche [latex]\mathbb {S}^2=\left \lbrace {v\in \mathbb {R}^3} \mid {\| {v}\| =1}\right \rbrace[/latex] betrachten und entscheiden wollen, ob der Ball [latex]\left \lbrace {v\in \mathbb {S}^2} \mid {\| {v-e_1}\|

Beweis von Lemma 9.20

Sei [latex]O\subseteq X[/latex] offen und [latex]y_0\in O\cap Y[/latex]. Dann gibt es ein [latex]\varepsilon >0[/latex] mit

[latex]
\begin{aligned}[]B_\varepsilon ^X(y_0)=\left \lbrace {x\in X} \mid {\mathrm {d}(x,y_0)[/latex]

Dies impliziert aber

[latex]
\begin{aligned}[]B_\varepsilon ^Y(y_0)=\left \lbrace {y\in Y} \mid {\mathrm {d}(y,y_0)[/latex]

womit [latex]O\cap Y[/latex] eine offene Teilmenge von [latex]Y[/latex] ist.

Sei nun umgekehrt [latex]O_Y\subseteq Y[/latex] eine offene Teilmenge. Dann existiert für jedes [latex]y\in O_Y[/latex] ein [latex]\varepsilon _y>0[/latex] mit [latex]B_{\varepsilon _y}^Y(y)\subseteq O_Y[/latex]. Wir definieren [latex]O=\bigcup _{y\in O_Y}B_{\varepsilon _y}^X(y)[/latex]. Da [latex]O[/latex] eine Vereinigung von offenen Bällen ist, folgt, dass [latex]O[/latex] eine offene Teilmenge von [latex]X[/latex] ist. Wir zeigen nun, dass [latex]O_Y=O\cap Y[/latex] erfüllt ist. Für [latex]y\in O_Y[/latex] gilt [latex]y\in B_{\varepsilon _y}^X(y)\subseteq O[/latex] und damit [latex]y\in O\cap Y[/latex]. Falls umgekehrt [latex]y\in O\cap Y[/latex] ist, dann existiert ein [latex]y'\in O_Y[/latex] mit [latex]y\in B_{\varepsilon _{y'}}^X(y')[/latex]. Daraus folgt aber [latex]y\in B_{\varepsilon _{y'}}^Y(y')\subseteq O_Y[/latex]. Dies zeigt, dass [latex]O_Y=O\cap Y[/latex] ist.

Die Aussagen über abgeschlossene Teilmengen folgen aus obigem durch Komplementbildung. ∎

Übung 9.21: Beispiele relativ offener Teilmengen

  1. Beschreiben Sie die in [latex]Y = \left \lbrace {(x,y)\in \mathbb {R}^2} \mid {y\geq 0}\right \rbrace[/latex] offenen Teilmengen, welche Punkte der Achse [latex]\left \lbrace {(x,y)} \mid {y=0}\right \rbrace[/latex] enthalten.
  2. Sei [latex]X = \left \lbrace {(x,y) \in \mathbb {R}^2} \mid {xy=0}\right \rbrace[/latex]. Charakterisieren Sie die in [latex]X[/latex] offenen Teilmengen.

Wir werden nun die Begriffe der offenen und abgeschlossenen Teilmengen mit dem Supremum von Teilmengen von [latex]\mathbb {R}[/latex] in Verbindung bringen.

Lemma 9.22: Supremum von abgeschlossenen und offenen Teilmengen in [latex]\mathbb {R}[/latex]

Für eine abgeschlossene nicht-leere von oben beschränkte Teilmenge [latex]A\subseteq \mathbb {R}[/latex] gilt [latex]\sup A=\max A\in A[/latex]. Für eine offene nicht-leere von oben beschränkte Teilmenge [latex]O\subseteq \mathbb {R}[/latex] gilt [latex]\sup O\notin O[/latex]. Dies gilt analog für von unten beschränkte Teilmengen und das Infimum.

Beweis

Für jedes [latex]n\in \mathbb {N}[/latex] existiert nach Satz 2.60 ein [latex]x_n\in A[/latex] mit [latex]\sup A-\frac 1n9.19, dass [latex]\sup A=\max A\in A[/latex].

Falls [latex]\sup O\in O[/latex] wäre, so gäbe es ein [latex]\varepsilon >0[/latex] mit [latex]B_\varepsilon (\sup O)\subseteq O[/latex]. Insbesondere gilt aber damit [latex]\sup O+\varepsilon /2\in O[/latex] und dies widerspricht der Definition des Supremums in Satz 2.60. Daher muss [latex]\sup O\notin O[/latex] gelten. ∎

Definition 9.23: Inneres, Rand und Abschluss

Sei [latex](X,\mathrm {d})[/latex] ein metrischer Raum und [latex]Y \subseteq X[/latex] eine Teilmenge. Ein Punkt [latex]x \in Y[/latex] heisst innerer Punkt von [latex]Y[/latex], falls es ein [latex]\varepsilon > 0[/latex] mit [latex]B_\varepsilon (x) \subseteq Y[/latex] gibt. Die Menge aller inneren Punkte

[latex]
\begin{aligned}[]Y^{\circ } = \left \lbrace {x \in Y} \mid {\exists \varepsilon > 0: B_\varepsilon (x) \subseteq Y}\right \rbrace\end{aligned}
[/latex]

wird das Innere von [latex]Y[/latex] genannt. Ein Punkt [latex]x \in X[/latex] ist ein Randpunkt von [latex]Y[/latex], falls zu jedem [latex]\varepsilon > 0[/latex] beide Durchschnitte [latex]B_\varepsilon (x) \cap Y[/latex] und [latex]B_\varepsilon (x) \cap (X \setminus Y)[/latex] nichtleer sind. Die Menge der Randpunkte

[latex]
\begin{aligned}[]\partial Y = \left \lbrace {x \in X} \mid {B_\varepsilon (x) \cap Y \neq \emptyset \neq B_\varepsilon (x) \cap (X \setminus Y) \text { für alle } \varepsilon > 0}\right \rbrace\end{aligned}
[/latex]

wird als der Rand von [latex]Y[/latex] bezeichnet. Der Abschluss einer Menge wird durch [latex]\overline {Y}=Y\cup \partial Y[/latex] definiert.

Ein Punkt [latex]x[/latex] ist also ein Randpunkt einer Teilmenge [latex]Y \subseteq X[/latex], falls er sowohl von Punkten in der Menge [latex]Y[/latex] als auch von Punkten ausserhalb von [latex]Y[/latex] «approximiert» werden kann. Beispielsweise ist der Rand des Einheitsballes [latex]B_1(0) \subseteq \mathbb {R}^2[/latex] gerade der Einheitskreis und der Abschluss ist der abgeschlossene Einheitsball [latex]\left \lbrace {v \in \mathbb {R}^2} \mid {\| {v}\| \leq 1}\right \rbrace[/latex].

Wichtige Übung 9.24: Eigenschaften des Abschluss und des Inneren

Sei [latex](X,\mathrm {d})[/latex] ein metrischer Raum und sei [latex]Y \subseteq X[/latex] eine Teilmenge.

  1. Zeigen Sie, dass [latex]Y^\circ[/latex] eine offene Teilmenge von [latex]X[/latex] ist und jede offene Teilmenge [latex]U \subseteq Y[/latex] enthält.
  2. Zeigen Sie, dass [latex]\overline {Y}[/latex] eine abgeschlossene Teilmenge von [latex]X[/latex] ist und in jeder abgeschlossenen Teilmenge [latex]A \supseteq Y[/latex] enthalten ist.

9.2.2 – Häufungspunkte und Dichtheit

Wir übertragen einige weitere Begriffe auf beliebige metrische Räume.

Definition 9.25: Häufungspunkte und Dichtheit

Sei [latex](X,\mathrm {d})[/latex] ein metrischer Raum. Ein Punkt [latex]x_0 \in X[/latex] ist ein Häufungspunkt einer Folge [latex](x_n)_n[/latex] in [latex]X[/latex], falls es eine konvergente Teilfolge [latex](x_{n_k})[/latex] gibt mit [latex]\lim _{k \to \infty }x_{n_k} = x_0[/latex]. Ein Punkt [latex]x_0 \in X[/latex] ist ein Häufungspunkt einer Teilmenge [latex]D \subseteq X[/latex], falls für jedes [latex]\varepsilon > 0[/latex] der Durchschnitt [latex]D \cap (B_{\varepsilon }(x_0) \setminus \left \lbrace {x_0} \right \rbrace )[/latex] nicht-leer ist. Eine Teilmenge [latex]D \subseteq X[/latex] heisst dicht, falls für jedes [latex]x_0 \in X[/latex] und jedes [latex]\varepsilon > 0[/latex] der Durchschnitt [latex]D \cap B_{\varepsilon }(x_0)[/latex] nicht-leer ist.

Beispielsweise wissen wir bereits, dass [latex]1[/latex] ein Häufungspunkt der Folge [latex]((-1)^n)_n[/latex] in [latex]\mathbb {R}[/latex] ist, [latex]0[/latex] ein Häufungspunkt der Teilmenge [latex](0,1][/latex] ist und [latex]\mathbb {Q} \subseteq \mathbb {R}[/latex] dicht ist.

Wichtige Übung 9.26

Sei [latex](X,\mathrm {d})[/latex] ein metrischer Raum.

  1. Zeigen Sie, dass ein Punkt [latex]x_0 \in X[/latex] genau dann ein Häufungspunkt einer Teilmenge [latex]D \subseteq X[/latex] ist, wenn es eine Folge [latex](x_n)_n[/latex] in [latex]D[/latex] gibt mit [latex]x_n \neq x_0[/latex] für alle [latex]n \in \mathbb {N}[/latex] und Grenzwert [latex]\lim _{n \to \infty } x_n = x_0[/latex].
  2. Zeigen Sie, dass eine Teilmenge [latex]D \subseteq X[/latex] genau dann dicht ist, wenn es für jeden Punkt [latex]x_0 \in X[/latex] eine Folge in [latex]D[/latex] gibt mit Grenzwert [latex]x_0[/latex].
  3. Zeigen Sie, dass eine konvergente Folge [latex](x_n)_n[/latex] in [latex]X[/latex] den Grenzwert [latex]\lim _{n \to \infty } x_n[/latex] als einzigen Häufungs­punkt hat. Insbesondere hat die konstante Folge [latex](x_0)_n[/latex] den Punkt [latex]x_0[/latex] als Häufungspunkt.
  4. Zeigen Sie, dass eine endliche Teilmenge von [latex]X[/latex] keinen einzigen Häufungspunkt besitzt. Insbesondere sind die Häufungspunkte einer Folge [latex](x_n)_n[/latex] in [latex]X[/latex] im Allgemeinen nicht dieselben wie jene der Teilmenge [latex]\left \lbrace {x_n} \mid {n\in \mathbb {N}}\right \rbrace[/latex].

Wichtige Übung 9.27: Charakterisierung der Häufungspunkte einer Folge

Zeigen Sie in Analogie zu Proposition 5.23, dass für einen metrischen Raum [latex]X[/latex], eine Folge [latex](x_n)_n[/latex] und einen Punkt [latex]x_0 \in X[/latex] folgende Bedingungen äquivalent sind.

  1. Der Punkt [latex]x_0[/latex] ist ein Häufungspunkt der Folge [latex](x_n)_n[/latex].
  2. Für jedes [latex]\varepsilon > 0[/latex] und jedes [latex]N \in \mathbb {N}[/latex] gibt es ein [latex]n \geq N[/latex] mit [latex]x_n \in B_\varepsilon (x_0)[/latex].

Übung 9.28: Häufungspunkte einer Teilmenge und der Abschluss

Sei [latex](X,\mathrm {d})[/latex] ein metrischer Raum und [latex]Y \subseteq X[/latex] eine Teilmenge.

  1. Zeigen Sie, dass [latex]x\in \overline {Y}[/latex] genau dann wenn es eine Folge in [latex]Y[/latex] mit Grenzwert gleich [latex]x[/latex] gibt.
  2. Sei [latex]x \in X \setminus Y[/latex]. Zeigen Sie, dass [latex]x[/latex] genau dann ein Häufungspunkt von [latex]Y[/latex] ist, wenn [latex]x \in \overline {Y}[/latex].
  3. Zeigen Sie, dass [latex]Y[/latex] genau dann dicht in [latex]X[/latex] ist, wenn [latex]\overline {Y} = X[/latex] gilt.

9.2.3 – Zusammenhang

Wir möchten eine weitere, diesmal neue Eigenschaft von metrischen Räumen einführen, die am Rande schon in Übungen 3.63 und 3.64 angesprochen wurde. Einen metrischen Raum wollen wir zusammenhängend nennen, falls er sich nicht mit offenen Mengen «auseinanderreissen» lässt. Präzis ausgedrückt:

Definition 9.29: Zusammenhang

Sei [latex](X,\mathrm {d})[/latex] ein nicht-leerer metrischer Raum. Wir nennen [latex](X,\mathrm {d})[/latex] zusammenhängend, falls es keine zwei offene nicht-leere Teilmengen [latex]O_1,O_2 \subseteq X[/latex] gibt mit [latex]X = O_1 \sqcup O_2[/latex]. Wir sagen, dass eine Teilmenge [latex]G\subseteq X[/latex] abgeschloffen ist, falls [latex]G[/latex] zugleich offen und abgeschlossen ist.

Alternativ ausgedrückt ist ein metrischer Raum [latex](X,\mathrm {d})[/latex] genau dann zusammenhängend, wenn es ausser der leeren Menge und [latex]X[/latex] keine weiteren abgeschloffenen Teilmengen von [latex]X[/latex] gibt. Man beachte auch, dass der Begriff des Zusammenhangs nur von der Topologie und nicht von der Wahl der Metrik abhängt. Man sagt auch, dass Zusammenhang eine topologische Eigenschaft ist.

Ein unzusammenhängender metrischer Raum lässt sich schnell als disjunkte Vereinigung von offenen Teilmengen eines grösseren Raumes konstruieren. Beispielsweise ist für [latex]z_1,z_2 \in \mathbb {C}[/latex] und [latex]r_1,r_2 > 0[/latex] die Menge [latex]X = B_{r_1}(z_1) \cup B_{r_2}(z_2)[/latex] mit der von der Standardmetrik induzierten Metrik unzusammenhängend, falls [latex]|z_1-z_2| \geq r_1 + r_2[/latex] oder anders ausgedrückt falls die Bälle [latex]B_{r_1}(z_1)[/latex] und [latex]B_{r_2}(z_2)[/latex] disjunkt sind.

image

Allgemeiner nennen wir eine Teilmenge [latex]Y \subseteq X[/latex] eines metrischen Raumes [latex](X,\mathrm {d})[/latex] zusammenhängend, falls der Teilraum [latex]Y[/latex] mit der von [latex]X[/latex] induzierten Metrik (oder Topologie) zusammenhängend ist. Wir charakterisieren nun die zusammenhängenden Teilmengen von [latex]\mathbb {R}[/latex].

Proposition 9.30: Zusammenhang in den reellen Zahlen

Eine nicht-leere Teilmenge [latex]X\subseteq \mathbb {R}[/latex] ist genau dann zusammenhängend, wenn [latex]X[/latex] ein Intervall ist.

Beweis

Angenommen [latex]X \subseteq \mathbb {R}[/latex] ist kein Intervall und seien [latex]a = \inf (X)[/latex] und [latex]b = \sup (X)[/latex]. Falls [latex](a,b) \subseteq X[/latex] wäre, dann wäre [latex]X[/latex] doch ein Intervall (wieso?). Also gibt es ein [latex]y \in (a,b) \setminus X[/latex] und nach Definition von [latex]a

[latex]
\begin{aligned}[]x_1 \in O_1&=(-\infty ,y)\cap X\\ x_2 \in O_2&= (y,\infty )\cap X.\end{aligned}
[/latex]

Da [latex]y[/latex] nicht in [latex]X[/latex] liegt, gilt [latex]X = O_1 \sqcup O_2[/latex]. Des Weiteren sind [latex]O_1[/latex] und [latex]O_2[/latex] offene, nicht-leere Teilmengen von [latex]X[/latex] und somit ist [latex]X[/latex] nicht zusammenhängend.

Sei nun [latex]X = I \subseteq \mathbb {R}[/latex] ein Intervall. Angenommen es existiert eine nicht-leere abgeschloffene Teilmenge [latex]Y\subsetneq I[/latex]. Wir wählen [latex]a \in Y[/latex] und [latex]b \in I\setminus Y[/latex]. Ohne Beschränkung der Allgemeinheit können wir annehmen, dass [latex]a

[latex]
\begin{aligned}[]s = \sup (Y \cap [a,b])=\sup (Y \cap (a,b)) \in [a,b] \subseteq I.\end{aligned}
[/latex]

Des Weiteren ist [latex]Y\cap (a,b)[/latex] eine offene Teilmenge von [latex]\mathbb {R}[/latex]. (Wieso?

Dies folgt aus Lemma 9.20. Alternativ existiert für jedes [latex]y_0\in Y\cap (a,b)[/latex] ein [latex]\varepsilon >0[/latex] so dass [latex]I\cap (y_0-\varepsilon ,y_0+\varepsilon )\subseteq Y[/latex] da [latex]Y[/latex] in [latex]I[/latex] offen ist. Wenn wir zusätzlich ohne Beschränkung der Allgemeinheit annehmen, dass [latex]\varepsilon \leq \min (y_0-a,b-y_0)[/latex], dann ergibt sich daraus [latex](y_0-\varepsilon ,y_0+\varepsilon )\subseteq Y\cap (a,b)[/latex].

) Daher zeigt Lemma 9.22, dass [latex]s\notin Y[/latex]. Da [latex]Y\subseteq I[/latex] abgeschlossen ist, folgt zum Beispiel aus Lemma 9.19 dass [latex]Y\cap [a,b][/latex] eine abgeschlossene Teilmenge von [latex]\mathbb {R}[/latex] ist. Daher zeigt Lemma 9.22, dass [latex]s\in Y[/latex] ist. Dieser Widerspruch zeigt also, dass es keine abgeschloffene Teilmengen von [latex]I[/latex] geben kann. ∎

Nun wollen wir eine mögliche Operation auf zusammenhängenden Teilmengen eines metrischen Raumes diskutieren. Eine weitere findet sich in einer Übung in Abschnitt 9.9.3.

Lemma 9.31: Vereinigungen von zusammenhängenden Teilmengen

Sei [latex](X,\mathrm {d})[/latex] ein metrischer Raum und seien [latex]Y_1,Y_2[/latex] zwei zusammenhängende Teilräume. Falls der Schnitt [latex]Y_1 \cap Y_2[/latex] nicht-leer ist, dann ist die Vereingung [latex]Y_1 \cup Y_2[/latex] zusammenhängend.

Beweis

Sei [latex]A[/latex] eine nicht-leere, abgeschlossene und offene Teilmenge von [latex]Y_1 \cup Y_2[/latex]. Dann ist [latex]A \cap Y_j[/latex] eine abgeschlossene und offene Teilmenge von [latex]Y_j[/latex] für [latex]j \in \left \lbrace {1,2} \right \rbrace[/latex]. Da [latex]A[/latex] nicht-leer ist, ist einer dieser beiden Schnitte nicht-leer — sagen wir [latex]A \cap Y_1[/latex] ist nicht-leer. Da [latex]Y_1[/latex] zusammenhängend ist, folgt [latex]A \cap Y_1 = Y_1[/latex] oder äquivalent [latex]Y_1 \subseteq A[/latex]. Da aber [latex]Y_1 \cap Y_2 \neq \emptyset[/latex] folgt [latex]A \cap Y_2 \neq \emptyset[/latex] und somit genauso [latex]A \cap Y_2 = Y_2[/latex] und [latex]Y_2 \subseteq A[/latex]. Zusammenfassend ergibt sich [latex]A = Y_1 \cup Y_2[/latex]. Dies beweist, dass [latex]Y_1 \cup Y_2[/latex] zusammenhängend ist. ∎

Wichtige Übung 9.32

Verallgemeinern Sie Lemma 9.31 für beliebige Vereinigungen.

Lemma 9.31 und Übung 9.32 können beispielsweise angewendet werden, um zu zeigen, dass (endliche) Produkte von zusammenhängenden Räumen zusammenhängend sind (wobei wir noch keine Metriken auf Produkträumen definiert haben). Wir wollen einen Spezialfall davon in folgender Übung diskutieren.

Übung 9.33: Zusammenhang der Ebene

Wir möchten zeigen, dass [latex]\mathbb {R}^2[/latex] zusammenhängend ist (wobei wir [latex]\mathbb {R}^2[/latex] mit der Standardtopologie betrachten). Gehen Sie dazu wie folgt vor:

  1. Betrachten Sie [latex]\mathbb {R}^2[/latex] mit der Maximumsnorm. Zeigen Sie, dass die offenen Teilmengen des Teilraumes [latex]\left \lbrace {x} \right \rbrace \times \mathbb {R}[/latex] für [latex]x \in \mathbb {R}[/latex] (respektive [latex]\mathbb {R} \times \left \lbrace {y} \right \rbrace[/latex] für [latex]y \in \mathbb {R}[/latex]) von der Form [latex]\left \lbrace {x} \right \rbrace \times U[/latex] für [latex]U \subseteq \mathbb {R}[/latex] offen (respektive [latex]U \times \left \lbrace {y} \right \rbrace[/latex] für [latex]U \subseteq \mathbb {R}[/latex] offen) sind.
  2. Zeigen Sie, dass die Teilräume [latex]\left \lbrace {x} \right \rbrace \times \mathbb {R}[/latex] für [latex]x \in \mathbb {R}[/latex] und [latex]\mathbb {R} \times \left \lbrace {y} \right \rbrace[/latex] für [latex]y \in \mathbb {R}[/latex] zusammenhängend sind.
  3. Verwenden Sie Lemma 9.31 und Übung 9.32 um zu zeigen, dass [latex]\mathbb {R}^2[/latex] zusammenhängend ist.

Hinweis.

Zu (iii): Betrachten Sie zuerst die Vereinigung einer horizontalen Linie mit einer vertikalen Linie.

9.3 – Stetigkeit

9.3.1 – Definition und Charakterisierungen

Wir möchten nun den Stetigkeitsbegriff auf metrische Räume verallgemeinern. Wie wir sehen werden, lässt sich dieser ausschliesslich mittels dem Begriff der Topologie (also mittels der Kollektion offener Mengen) charakterisieren. Wir möchten Stetigkeit allerdings zuerst in Analogie zu Definition 3.46 (respektive der Charakterisierung in Proposition 5.17) einführen.

Definition 9.34: Stetigkeit bei einem Punkt

Seien [latex](X,\mathrm {d}_X),(Y,\mathrm {d}_Y)[/latex] zwei metrische Räume und sei [latex]f: X \to Y[/latex] eine Funktion. Wir sagen, dass [latex]f[/latex] bei [latex]x_0 \in X[/latex] [latex]\mathbf {\varepsilon }[/latex]–[latex]\mathbf {\delta }[/latex]-stetig ist, falls für alle [latex]\varepsilon >0[/latex] ein [latex]\delta > 0[/latex] existiert, so dass für alle [latex]x \in B_\delta (x_0)[/latex] auch [latex]f(x) \in B_{\varepsilon }(f(x_0))[/latex] gilt. Wir sagen, dass [latex]f[/latex] bei [latex]x_0 \in X[/latex] folgenstetig ist, falls für jede konvergente Folge [latex](x_n)_n[/latex] in [latex]X[/latex] mit Grenzwert [latex]\lim _{n \to \infty }x_n = x_0[/latex] die Folge [latex](f(x_n))_n[/latex] konvergiert und Grenzwert [latex]\lim _{n \to \infty }f(x_n) = f(x_0)[/latex] hat.

Wie in obiger Definition und vielleicht schon in den letzten zwei Abschnitten ersichtlich wurde, ist eine explizite Bezeichnung [latex]\mathrm {d}[/latex] der Metrik in [latex](X,\mathrm {d})[/latex] nicht immer notwendig (da man zum Beispiel oft stattdessen nur Bälle betrachtet) und führt nur zu zusätzlicher Notation. Wir werden deswegen in Zukunft schlicht [latex]X[/latex] als metrischen Raum bezeichnen, wobei die Metrik also implizit ist, keinen «Namen» hat und wenn nötig einfach mit [latex]\mathrm {d}(\cdot ,\cdot )[/latex] bezeichnet wird.

Lemma 9.35: Stetigkeit bei einem Punkt

Seien [latex]X[/latex] und [latex]Y[/latex] zwei metrische Räume, [latex]f:X \to Y[/latex] eine Funktion und [latex]x_0 \in X[/latex] ein Punkt. Dann ist [latex]f[/latex] genau dann bei [latex]x_0[/latex] [latex]\varepsilon[/latex]–[latex]\delta[/latex]-stetig, wenn [latex]f[/latex] bei [latex]x_0[/latex] folgenstetig ist.

Auf Grund der Aussage des obigen Lemma sagen wir auch kurz, dass [latex]f[/latex] bei [latex]{x_0}[/latex] stetig ist, falls [latex]f[/latex] bei [latex]x_0[/latex] [latex]\varepsilon[/latex]–[latex]\delta[/latex]-stetig ist. Des Weiteren ist [latex]f[/latex] stetig, wenn [latex]f[/latex] bei jedem Punkt in [latex]X[/latex] stetig ist. In Proposition 9.37 werden weitere wichtige Charakterisierungen von Stetigkeit folgen.

Beweis

Angenommen [latex]f[/latex] ist bei [latex]x_0[/latex] [latex]\varepsilon[/latex]–[latex]\delta[/latex]-stetig. Sei [latex](x_n)_n[/latex] eine Folge in [latex]X[/latex] mit Grenzwert [latex]x_0\in X[/latex] und sei [latex]\varepsilon > 0[/latex]. Dann existiert ein [latex]\delta > 0[/latex], so dass [latex]f(x) \in B_\varepsilon (f(x_0))[/latex] für alle [latex]x \in B_\delta (x_0)[/latex]. Da [latex](x_n)_n[/latex] gegen [latex]x_0[/latex] konvergiert, existiert nun ein [latex]N \in \mathbb {N}[/latex] mit [latex]x_n \in B_\delta (x_0)[/latex] für alle [latex]n \geq N[/latex]. Insbesondere gilt für [latex]n \geq N[/latex] also [latex]f(x_n) \in B_{\varepsilon }(f(x_0))[/latex]. Da [latex]\varepsilon > 0[/latex] beliebig war, gilt somit [latex]\lim _{n \to \infty } f(x_n) = f(x_0)[/latex] und [latex]f[/latex] ist bei [latex]x_0[/latex] folgenstetig wie gewünscht.

Angenommen [latex]f[/latex] ist bei [latex]x_0[/latex] nicht [latex]\varepsilon[/latex]–[latex]\delta[/latex]-stetig. Dann existiert ein [latex]\varepsilon > 0[/latex], so dass es für jedes [latex]\delta > 0[/latex] ein [latex]x \in B_\delta (x_0)[/latex] gibt mit [latex]f(x) \not \in B_\varepsilon (f(x_0))[/latex]. Wir wählen nun für jedes [latex]n \in \mathbb {N}[/latex] und [latex]\delta = \frac {1}{n}[/latex] ein solches [latex]x_n \in B_{\frac {1}{n}}(x_0)[/latex]. Die Folge [latex](x_n)_n[/latex] konvergiert somit gegen [latex]x_0[/latex] und es gilt [latex]f(x_n) \not \in B_\varepsilon (f(x_0))[/latex] für alle [latex]n \in \mathbb {N}[/latex]. Insbesondere konvergiert [latex](f(x_n))_n[/latex] nicht gegen [latex]f(x_0)[/latex] und [latex]f[/latex] ist nicht folgenstetig. ∎

Genauso wie für Funktionen auf [latex]\mathbb {R}[/latex] können wir auch in diesem neuen, allgemeinen Rahmen Grenzwerte von Funktionen definieren.

Definition 9.36

Sei [latex]X[/latex] ein metrischer Raum, [latex]D \subseteq X[/latex] eine Teilmenge, [latex]x_0 \in X[/latex] ein Häufungspunkt von [latex]D[/latex] und [latex]f: D \to Y[/latex] eine Abbildung in einen weiteren metrischen Raum [latex]Y[/latex]. Dann ist [latex]y_0 \in Y[/latex] der Grenzwert von [latex]f(x)[/latex] für [latex]x \to x_0[/latex], geschrieben [latex]y_0 = \lim _{x \to x_0}f(x)[/latex], falls gilt

[latex]
\begin{aligned}[]\forall \varepsilon >0\ \exists \delta > 0\ \forall x \in D\cap B_\delta (x_0) \setminus \left \lbrace {x_0} \right \rbrace : f(x) \in B_\varepsilon (y_0).\end{aligned}
[/latex]

Ist [latex]x_0 \in D[/latex], so ist wiederum [latex]f[/latex] genau dann bei [latex]x_0[/latex] stetig, falls der Grenzwert [latex]\lim _{x \to x_0}f(x)[/latex] existiert und gleich [latex]f(x_0)[/latex] ist.

Wie schon erwähnt wurde, wollen wir nun Stetigkeit mit Hilfe der Topologie ausdrücken.

Proposition 9.37: Charakterisierungen der Stetigkeit

Seien [latex]X,Y[/latex] zwei metrische Räume und [latex]f:X \to Y[/latex] eine Funktion. Dann sind folgende Bedingungen äquivalent:

  1. Die Funktion [latex]f[/latex] ist stetig.
  2. Für jedes [latex]x \in X[/latex] ist [latex]f[/latex] bei [latex]x[/latex] [latex]\varepsilon[/latex]–[latex]\delta[/latex]-stetig.
  3. Für jedes [latex]x \in X[/latex] ist [latex]f[/latex] bei [latex]x[/latex] folgenstetig.
  4. Für jede offene Teilmenge [latex]O \subseteq Y[/latex] ist [latex]f^{-1}(O)[/latex] eine offene Teilmenge von [latex]X[/latex].
  5. Für jede abgeschlossene Teilmenge [latex]A \subseteq Y[/latex] ist [latex]f^{-1}(A)[/latex] eine abgeschlossene Teilmenge von [latex]X[/latex].
Beweis

Per Definition der Stetigkeit ist (i) äquivalent zu (ii). Nach Lemma 9.35 sind (ii) und (iii) äquivalent. Wir zeigen als nächsten Schritt die Äquivalenz von (i) und (iv).

Sei [latex]f[/latex] stetig und [latex]O \subseteq Y[/latex] offen. Für [latex]x \in f^{-1}(O)[/latex] ist [latex]f(x)\in O[/latex], wodurch es ein [latex]\varepsilon > 0[/latex] gibt mit [latex]B_\varepsilon (f(x)) \subseteq O[/latex], da [latex]O[/latex] offen ist. Da [latex]f[/latex] bei [latex]x[/latex] [latex]\varepsilon[/latex]–[latex]\delta[/latex]-stetig ist, gibt es ein [latex]\delta >0[/latex] mit [latex]f(B_\delta (x)) \subseteq B_\varepsilon (f(x)) \subseteq O[/latex] und somit [latex]B_\delta (x) \subseteq f^{-1}(O)[/latex]. Da aber [latex]x\in f^{-1}(O)[/latex] beliebig war, ist [latex]f^{-1}(O)[/latex] also offen. Dies beweist (i)[latex]\implies[/latex](iv).

Angenommen [latex]f[/latex] erfüllt die Bedingung in (iv). Sei [latex]x_0 \in X[/latex] und [latex]\varepsilon > 0[/latex]. Wir wissen, dass das Urbild [latex]f^{-1}(B_\varepsilon (f(x_0))) \subseteq X[/latex] den Punkt [latex]x_0[/latex] enthält und offen ist, da der Ball [latex]B_\varepsilon (f(x_0))[/latex] in [latex]Y[/latex] offen ist. Also existiert ein [latex]\delta > 0[/latex] mit [latex]B_\delta (x_0) \subseteq f^{-1}(B_\varepsilon (f(x_0)))[/latex] oder äquivalenterweise [latex]f(B_\delta (x_0)) \subseteq B_\varepsilon (f(x_0))[/latex]. Also ist [latex]f[/latex] bei [latex]x_0[/latex] [latex]\varepsilon[/latex]–[latex]\delta[/latex]-stetig, was die Implikation (iv)[latex]\implies[/latex](i) beweist.

Die Äquivalenz von (iv) und (v) ergibt sich aus den Eigenschaften von Urbildern und der Dualität von offenen und abgeschlossenen Mengen (via der Komplementoperation). Falls [latex]f^{-1}(O)[/latex] für jede offene Menge [latex]O \subseteq Y[/latex] offen ist und [latex]A \subseteq Y[/latex] abgeschlossen ist, dann ist [latex]f^{-1}(A) = X \setminus f^{-1}(Y \setminus A)[/latex] abgeschlossen. Falls [latex]f^{-1}(A)[/latex] für jede abgeschlossene Menge [latex]A \subseteq Y[/latex] abgeschlossen ist und [latex]O \subseteq Y[/latex] offen ist, dann ist [latex]f^{-1}(O) = X \setminus f^{-1}(Y \setminus O)[/latex] offen. ∎

Mit den Kriterien (iv) und (v) in Proposition 9.37 verfügt man auch über ein nützliches Werkzeug, um Teilmengen eines Raumes auf Offenheit und Abgeschlossenheit zu untersuchen. Zum Beispiel gelten für eine stetige Funktion [latex]f:X\to \mathbb {R}[/latex], dass die Teilmenge [latex]\left \lbrace {x\in X} \mid {f(x)>0}\right \rbrace[/latex] offen und die Teilmenge [latex]\left \lbrace {x\in X} \mid {f(x)\geq 0}\right \rbrace[/latex] abgeschlossen ist. Da es leicht ist (siehe Abschnitt 9.3.3), stetige Funktionen zu konstruieren, kann man dadurch für viele Mengen Offenheit oder Abgeschlossenheit zeigen.

Bemerkung

In der Vorlesung «Topologie» im vierten Semester des Mathematikstudiums werden allgemeiner über die Eigenschaften in Lemma 9.17 Kollektionen offener Teilmengen einer Menge betrachtet. Klebt man beispielsweise die Ränder des Quadrats [latex][0,1]^2[/latex] nach vorgegebenen Regeln aneinander, so lässt sich auf dem erhaltenen Quotientenraum eine natürliche Topologie mittels der Topologie auf [latex][0,1]^2[/latex] definieren. Stetigkeit wird dann wie in Proposition 9.37 dadurch definiert, dass Urbilder offener Mengen offen sein sollen.

9.3.2 – Zwei stärkere Stetigkeitsbegriffe

Wie wir bereits gesehen haben, sind manchmal folgende Stetigkeitseigenschaften nützlich.

Definition 9.38

Seien [latex](X,\mathrm {d}_X)[/latex] und [latex](Y,\mathrm {d}_Y)[/latex] zwei metrische Räume und [latex]f:X \to Y[/latex] eine Funktion. Dann heisst [latex]f[/latex] gleichmässig stetig, falls es zu jedem [latex]\varepsilon > 0[/latex] ein [latex]\delta >0[/latex] gibt, so dass für alle [latex]x_1,x_2 \in X[/latex] mit [latex]\mathrm {d}_X(x_1,x_2) Lipschitz-stetig, falls es eine sogenannte Lipschitz-Konstante [latex]L \geq 0[/latex] gibt mit

[latex]
\begin{aligned}[]\mathrm {d}_Y(f(x_1),f(x_2)) \leq L \mathrm {d}_X(x_1,x_2)\end{aligned}
[/latex]

für alle [latex]x_1,x_2 \in X[/latex].

Per Definition sind gleichmässig stetige Funktionen stetig und wie wir schon gesehen haben, sind stetige Funktionen nicht zwingend gleichmässig stetig. Des Weiteren sind Lipschitz stetige Funktionen gleichmässig stetig. (Wieso?

Wir können für jedes [latex]\varepsilon >0[/latex] einfach [latex]\delta =\frac 1{L+1}\varepsilon[/latex] verwenden. (Wobei wir [latex]L+1[/latex] im Zähler verwenden um auch im eigenartigen Fall [latex]L=0[/latex] eine vernünftige Definition für [latex]\delta[/latex] zu erhalten.)

). In Proposition 9.77 werden wir in Analogie zu Satz 3.78 zeigen können, dass unter gewissen, wichtigen Annahmen an [latex]X[/latex] eine stetige Funktion auch gleichmässig stetig ist. Des Weiteren werden wir später sehen, dass eine reellwertige Funktionen auf einer offenen Teilmengen von [latex]\mathbb {R}^d[/latex], die mehrdimensional differenzierbar ist und eine beschränkte Ableitung besitzt, Lipschitz-stetig ist falls der Definitionsbereich noch weitere Annahmen erfüllt.

9.3.3 – Konstruktion von stetigen Funktionen

Wie schon in Abschnitt 3.5 lassen sich für stetigen Funktionen verschiedene Operationen durchführen, die wiederum zu weiteren stetigen Funktionen führen. Die folgende Proposition fasst viele Themen aus dem ersten Semester zusammen und erweitert diese auf den allgemeineren Rahmen dieses Kapitels.

Proposition 9.39: Stetige Funktionen

Seien [latex]X,Y,Z[/latex] metrische Räume.

  1. Falls [latex]f: X \to Y[/latex] und [latex]g:Y \to Z[/latex] stetig sind, dann ist auch [latex]g \circ f: X \to Z[/latex] stetig.
  2. Eine Funktion [latex]f:X\to \mathbb {C}^d[/latex] ist genau dann stetig, wenn die Komponenten
    [latex]
    \begin{aligned}[]f_j=\pi _j\circ f:X\to \mathbb {C}\end{aligned}
    [/latex]

    für [latex]j=1,\ldots ,d[/latex] stetig sind, wobei [latex]\pi _j:(z_1,\ldots ,z_d)^t\in \mathbb {C}^d\mapsto z_j\in \mathbb {C}[/latex] für [latex]j\in \left \lbrace {1,\ldots ,d} \right \rbrace[/latex] die Projektion auf die [latex]j[/latex]-te Koordinate bezeichnet.

  3. Die Addition [latex]+:\mathbb {C}^2\to \mathbb {C}[/latex] ist stetig.
  4. Die Multiplikation [latex]\cdot :\mathbb {C}^2\to \mathbb {C}[/latex] ist stetig.
  5. Die Kehrwertfunktion [latex](\ )^{-1}:z\in \mathbb {C}^\times \mapsto z^{-1}\in \mathbb {C}[/latex] ist stetig.
  6. Polynome in [latex]\mathbb {C}[z][/latex], die komplexe Exponentialabbildung, der reelle Logarithmus, Wurzelfunktionen, die komplexen trigonometrischen Funktionen [latex]\sin[/latex], [latex]\cos[/latex], [latex]\tan[/latex], [latex]\cot[/latex] die komplexen hyperbolischen Funktionen [latex]\sinh[/latex], [latex]\cosh[/latex], [latex]\tanh[/latex], die reellen inversen trigonometrischen Funktionen [latex]\arcsin[/latex], [latex]\arccos[/latex], [latex]\arctan[/latex] und die reellen inversen hyperbolischen Funktionen [latex]\operatorname {arsinh}[/latex], [latex]\operatorname {arcosh}[/latex], [latex]\operatorname {artanh}[/latex] sind auf ihrem jeweiligen Definitionsbereich stetig. Des Weiteren definieren Potenzreihen im Inneren des Konvergenzkreises stetige Funktionen.

Insbesondere können wir mit obiger Proposition die Stetigkeit von beliebig komplizierten Ausdrücken wie zum Beispiel [latex]x\in (0,\infty )\mapsto \sin (\log (x)+x^2-\exp (x+\mathrm {i} \sqrt [3]{x}))[/latex] zeigen. (Wie?

Durch schrittweisen Aufbau der Funktion von Innen nach Aussen: Wegen (ii) und (vi) ist [latex]x\in (0,\infty )\mapsto (x,\sqrt [3]{x})^t[/latex] stetig und damit wegen (iii) auch [latex]x+\sqrt [3]{x}[/latex]. Wegen (i) und (vi) ergibt sich die Stetigkeit von [latex]\exp (x+\mathrm {i}\sqrt [3]{x})[/latex]. Wiederum wegen (i), (ii), (iii), (iv) und (vi) folgt die Stetigkeit von [latex]\log (x)+x^2-\exp (x+\mathrm {i} \sqrt [3]{x})[/latex]. Gemeinsam mit (i) und (vi) ergibt sich nun die Aussage.

)

Beweis

Für (i) sei [latex]O\subseteq Z[/latex] offen, dann ist nach Proposition 9.37(iv) [latex]g^{-1}O\subseteq Y[/latex] offen und damit auch [latex](g\circ f)^{-1}O=f^{-1}(g^{-1}O)\subseteq X[/latex] offen. Da [latex]O\subseteq Z[/latex] eine beliebige offene Teilmenge ist, folgt die Stetigkeit von [latex]g\circ f[/latex] wiederum nach Proposition 9.37(iv).

Die Eigenschaft in (ii) folgt aus der Charakterisierung der Stetigkeit mittels Folgen in Proposition 9.37(iii) und der Charakterisierung von Folgenkonvergenz in [latex]\mathbb {R}^d[/latex] in Lemma 5.93. Die Aussage in (iii), (iv), (v) folgen ebenso aus Proposition 9.37(iii) und Proposition 5.7. Obwohl logisch unnötig geben wir noch direkte Argumente dafür an.

Für (ii) bemerken wir zuerst, dass die Projektion [latex]\pi _j:\mathbb {C}^d\to \mathbb {C}[/latex] wegen [latex]|\pi _j(v-w)|\leq \| v-w\|[/latex] für [latex]v,w\in \mathbb {C}^d[/latex] Lipschitz-stetig mit Lipschitz-Konstante [latex]1[/latex] ist. Daher folgt aus (i) und Stetigkeit von [latex]f:X\to \mathbb {C}^d[/latex], dass auch [latex]\pi _j\circ f:X\to C[/latex] stetig ist für jedes [latex]j\in \{ 1,\ldots ,d\}[/latex]. Sei nun umgekehrt [latex]\pi _j\circ f:X\to C[/latex] für jedes [latex]j\in \{ 1,\ldots ,d\}[/latex] stetig. Sei [latex]x_0\in X[/latex] und [latex]\varepsilon >0[/latex], dann existiert für jedes [latex]j\in \{ 1,\ldots ,d\}[/latex] ein [latex]\delta _j>0[/latex] mit [latex]|\pi _j\circ f(x)-\pi _j\circ f(x_0)|

Für [latex](z_0,w_0)^t,(z,w)^t\in \mathbb {C}^2[/latex] gilt

[latex]
\begin{aligned}[]|(z+w)-(z_0+w_0)|\leq |z-z_0|+|w-w_0|\leq 2\| {(z,w)^t-(z_0,w_0)^t}\| .\end{aligned}
[/latex]

Daher ist die Addition Lipschitz-stetig mit Lipschitz-Konstante [latex]2[/latex], und (iii) folgt.

Für [latex](z_0,w_0)^t,(z,w)^t\in \mathbb {C}^2[/latex] gilt

[latex]
\begin{aligned}[]|zw-z_0w_0|\leq |zw-z_0w|+|z_0w-z_0w_0| =|w||z-z_0|+|z_0||w-w_0|.\end{aligned}
[/latex]

Sei nun [latex]\varepsilon >0[/latex] und

[latex]
\begin{aligned}[]\| {(z,w)^t-(z_0,w_0)^t}\| [/latex]

Dann gilt [latex]|w|\leq |w-w_0|+|w_0|\leq 1+|w_0|[/latex] und daher

[latex]
\begin{aligned}[]|zw-z_0w_0|\leq (1+|w_0|)|z-z_0|+|z_0||w-w_0|[/latex]

Dies zeigt Stetigkeit der Multiplikation in (iv).

Für [latex]z_0\in \mathbb {C}^\times[/latex], [latex]\varepsilon >0[/latex] und [latex]z\in \mathbb {C}[/latex] mit [latex]|z-z_0|

[latex]
\begin{aligned}[]|z|\geq |z_0|-|z-z_0|>\tfrac 12|z_0|\end{aligned}
[/latex]

und daher

[latex]
\begin{aligned}[]|z^{-1}-z_0^{-1}|=|zz_0|^{-1}|z_0-z|\leq 2|z_0|^{-2}\tfrac 12\varepsilon |z_0|^2=\varepsilon .\end{aligned}
[/latex]

Da [latex]\varepsilon >0[/latex] beliebig war, zeigt dies die Stetigkeit der Kehrwertabbildung in (v).

Für die Stetigkeit der Abbildungen in (vi) siehe Korollar 3.52, Satz 6.68, Korollar 5.58, Satz 6.72, Abschnitt 6.7.1, Abschnitt 7.3, Abschnitt 7.4, und Satz 6.56. ∎

Eine wichtige Methode der Definition von Funktion, die nicht von Proposition 9.39 abgedeckt wird, ist die Definition mittels Fallunterscheidungen. Dies führt im Allgemeinen nicht zu stetigen Funktionen (charakteristische Funktionen bilden ein schnelles Gegenbeispiel), doch gibt es auch Situationen, wo eine mittels Fallunterscheidungen definierte Funktion stetig ist.

Übung 9.40: Stetige Funktionen durch Fallunterscheidung

Seien [latex]X,Y[/latex] zwei metrische Räume und seien [latex]A_1,A_2 \subseteq X[/latex] zwei abgeschlossene Teilmengen von [latex]X[/latex] mit [latex]X = A_1 \cup A_2[/latex]. Angenommen [latex]f_1:A_1 \to Y[/latex] und [latex]f_2: A_2 \to Y[/latex] sind zwei stetige Funktionen mit [latex]f_1(x) = f_2(x)[/latex] für alle [latex]x \in A_1 \cap A_2[/latex]. Zeigen Sie, dass die damit wohldefinierte Funktion

[latex]
\begin{aligned}[]f: X \to Y,\ x \mapsto \left \lbrace \begin{array}{cc} f_1(x) & \text {falls } x \in A_1 \\ f_2(x) & \text {falls } x \in A_2\end{array} \right .\end{aligned}
[/latex]

stetig ist.

Wichtige Übung 9.41: Distanzfunktionen

Sei [latex](X,\mathrm {d})[/latex] ein metrischer Raum. In dieser Übung möchten wir den Abstand von Teilmengen von [latex]X[/latex] zu Punkten diskutieren. Zu [latex]x\in X[/latex] und [latex]A \subseteq X[/latex] nicht-leer definieren wir

[latex]
\begin{aligned}[]\mathrm {d}(x,A) = \inf _{a \in A} \mathrm {d}(x,a).\end{aligned}
[/latex]
  1. Zeigen Sie, dass eine nicht-leere Teilmenge [latex]A \subseteq X[/latex] genau dann abgeschlossen ist, wenn für alle [latex]x \in X[/latex] die Eigenschaft [latex]\mathrm {d}(x,A) = 0 \iff x \in A[/latex] gilt.
  2. Zeigen Sie, dass die Funktion [latex]x \in X \mapsto \mathrm {d}(x,A)\in \mathbb {R}[/latex] Lipschitz-stetig mit Lipschitz-Konstante [latex]1[/latex] ist.

Hinweis.

Zu (ii) können Sie Übung 9.5 verwenden.

9.3.4 – Eine Bemerkung zur Stetigkeit

Für [latex]d \geq 2[/latex] gibt es für die Bewegung [latex]x \to x_0[/latex] im [latex]\mathbb {R}^d[/latex] mehr Möglichkeiten als im eindimensionalen [latex]\mathbb {R}[/latex]. In der Definition des Grenzwerts [latex]\lim _{x\to x_0} f(x)[/latex] werden alle mögliche Bewegungen (entlang Geraden, entlang beliebigen Kurven oder auch ohne Formeln) auf [latex]x_0[/latex] zu erlaubt. Wir wollen die Bedeutung dieser Bemerkung mit einigen Beispielen belegen.

Beispiel 9.42

  1. Sei [latex]f: \mathbb {R}^2 \to \mathbb {R}[/latex] gegeben durch
    [latex]
    \begin{aligned}[]f(x,y) = \left \lbrace \begin{array}{cc} \frac {xy}{x^2+y^2} & \text {falls } (x,y)^t \neq (0,0)^t\\ 0 & \text {falls } (x,y)^t = (0,0)^t\end{array} \right .\end{aligned}
    [/latex]

    für [latex](x,y)^t \in \mathbb {R}^2[/latex]. Dann ist [latex]f[/latex] auf der [latex]x[/latex]-Achse und ebenso auf der [latex]y[/latex]-Achse identisch Null. Hingegen ist [latex]f[/latex] auf einer Geraden [latex]y = ax[/latex] zu [latex]a \neq 0[/latex] gleich [latex]\frac {a}{1+a^2}[/latex]. Daher ist [latex]f[/latex] bei [latex](0,0)^t[/latex] nicht stetig.

  2. Sei [latex]f: \mathbb {R}^2 \to \mathbb {R}[/latex] gegeben durch
    [latex]
    \begin{aligned}[]f(x,y) = \left \lbrace \begin{array}{cc} \frac {x^2y}{x^4+y^2} & \text {falls } (x,y)^t \neq (0,0)^t\\ 0 & \text {falls } (x,y)^t = (0,0)^t\end{array} \right .\end{aligned}
    [/latex]

    für [latex](x,y)^t \in \mathbb {R}^2[/latex]. Dann ist [latex]f[/latex] auf der [latex]x[/latex]-Achse und ebenso auf der [latex]y[/latex]-Achse identisch Null. Des Weiteren ist [latex]f[/latex] auf einer Geraden [latex]y = ax[/latex] zu [latex]a \neq 0[/latex] gleich [latex]\frac {ax^3}{x^4+a^2x^2} = \frac {ax}{a^2+x^2}[/latex] und damit entlang dieser Gerade stetig. Dennoch ist [latex]f[/latex] bei [latex](0,0)^t[/latex] nicht stetig, da für [latex]y = x^2[/latex] und [latex]x \neq 0[/latex]

    [latex]
    \begin{aligned}[]f(x,y)= f(x,x^2) = \frac {x^4}{2x^4} = \frac {1}{2}\end{aligned}
    [/latex]

    ist.

Bei der Definition der Stetigkeit einer Funktion [latex]f: U \to \mathbb {R}[/latex] auf einer offenen Menge [latex]U \subseteq \mathbb {R}^d[/latex] bei einem Punkt [latex]x_0 \in U[/latex] oder auch der Definition des Grenzwerts [latex]\lim _{x \to x_0 } f(x)[/latex] darf man also nebst der Annahme, dass [latex]x[/latex] hinreichend nahe an [latex]x_0[/latex] liegt, keine weiteren Annahmen über die genauere Lage von [latex]x[/latex] zu [latex]x_0[/latex] treffen.

9.3.5 – Stetigkeit und Zusammenhang

Stetige Abbildungen verhalten sich gut bezüglich zusammenhängenden Mengen.

Proposition 9.43: Bilder zusammenhängender Mengen unter stetigen Abbildungen

Seien [latex]X,Y[/latex] metrische Räume und [latex]f: X \to Y[/latex] stetig. Falls [latex]X[/latex] zusammenhängend ist, dann ist das Bild [latex]f(X)[/latex] ein zusammenhängender Teilraum von [latex]Y[/latex].

Beispielsweise ist also [latex]\mathbb {S}^1 = \left \lbrace {z \in \mathbb {C}} \mid {|z|=1}\right \rbrace \subseteq \mathbb {C}[/latex] zusammenhängend, da die stetige Abbildung [latex]x \in \mathbb {R} \mapsto \mathrm {e}^{2\pi \mathrm {i} x} \in \mathbb {S}^1[/latex] surjektiv ist.

Beweis

Wir können ohne Beschränkung der Allgemeinheit annehmen, dass [latex]f:X \to Y[/latex] surjektiv ist, indem wir [latex]f:X \to Y[/latex] durch die stetige, surjektive Abbildung [latex]X \to f(X),\ x \mapsto f(x)[/latex] ersetzen. Angenommen [latex]Y[/latex] ist nicht zusammenhängend. Dann existiert eine offene und abgeschlossene Teilmenge [latex]A \subseteq Y[/latex] mit [latex]A \neq \emptyset[/latex] und [latex]A \neq Y[/latex]. Das Urbild [latex]f^{-1}(A)[/latex] von [latex]A[/latex] unter [latex]f[/latex] ist somit nach Proposition 9.37 ebenfalls offen und abgeschlossen. Da aber [latex]X[/latex] zusammenhängend ist, muss entweder [latex]f^{-1}(A) = \emptyset[/latex] oder [latex]f^{-1}(A) = X[/latex]. Dann gilt aber [latex]A = \emptyset[/latex] respektive [latex]A = Y[/latex] wegen Surjektivität von [latex]f[/latex], was der Annahme an [latex]A[/latex] widerspricht. ∎

Eine direkte Konsequenz von Proposition 9.43 kennen wir bereits aus Kapitel 3.

Korollar 9.44: Zwischenwertsatz

Sei [latex]I\subseteq \mathbb {R}[/latex] ein Intervall, [latex]f: I \to \mathbb {R}[/latex] eine stetige Funktion und [latex]a,b\in I[/latex]. Für jedes [latex]c\in \mathbb {R}[/latex] zwischen [latex]f(a)[/latex] und [latex]f(b)[/latex] gibt es ein [latex]x\in I[/latex] zwischen [latex]a[/latex] und [latex]b[/latex], so dass [latex]f(x) = c[/latex] gilt.

Beweis

Ohne Beschränkung der Allgemeinheit können wir [latex]a 9.43 auf die stetige Abbildung [latex]f|_{[a,b]}:[a,b] \to \mathbb {R}[/latex] an. Demnach ist [latex]f([a,b])[/latex] zusammenhängend, da nach Proposition 9.30 das Intervall [latex][a,b][/latex] in [latex]\mathbb {R}[/latex] zusammenhängend ist. Wieder nach Proposition 9.30 muss [latex]f([a,b]) \subseteq \mathbb {R}[/latex] ein Intervall sein. Da [latex]f(a),f(b)\in f([a,b])[/latex] gilt, liegen alle Werte zwischen [latex]f(a)[/latex] und [latex]f(b)[/latex] im Bild von [latex]f|_{[a,b]}[/latex]. ∎

Übung 9.45

Zeigen Sie folgende Verallgemeinerung des Zwischenwertsatzes: Sei [latex]X[/latex] ein zusammenhängender metrischer Raum und [latex]f: X \to \mathbb {R}[/latex] eine stetige Abbildung. Seien [latex]a,b \in X[/latex]. Dann existiert für jedes [latex]c \in \mathbb {R}[/latex] zwischen [latex]f(a)[/latex] und [latex]f(b)[/latex] ein [latex]x \in X[/latex] mit [latex]f(x) = c[/latex].

Nebst einem besser strukturierten Beweis des Zwischenwertsatzes liefert Proposition 9.43 eine alternative Auffassung von «Zusammenhang» .

Definition 9.46: Wege und Wegzusammenhang

Sei [latex]X[/latex] ein nicht-leerer metrischer Raum.

  • Ein Weg in [latex]X[/latex] ist eine stetige Abbildung [latex]\gamma :[a,b] \to X[/latex] auf einem nicht-leeren Intervall [latex][a,b]\subseteq \mathbb {R}[/latex] mit Startpunkt [latex]\gamma (a)[/latex] und Endpunkt [latex]\gamma (b)[/latex]. Dabei sagen wir auch, dass [latex]\gamma[/latex] ein Weg von [latex]\gamma (a)[/latex] nach [latex]\gamma (b)[/latex] ist.
  • Wir nennen [latex]X[/latex] wegzusammenhängend, falls für je zwei Punkte [latex]x,y\in X[/latex] zwei reelle Zahlen [latex]a

Wir können ohne Beschränkung der Allgemeinheit in obiger Definition auch als Definitionsbereich der Wege immer das Intervall [latex][0,1][/latex] wählen. Denn für beliebige reelle Zahlen [latex]a

Lemma 9.47

Jeder wegzusammenhängende metrische Raum ist zusammenhängend.

Beweis

Sei [latex]X[/latex] ein nicht-leerer wegzusammenhängender metrischer Raum und sei [latex]x \in X[/latex] fix. Da [latex]X[/latex] wegzusammenhängend ist, existiert für jedes [latex]y \in X[/latex] ein Weg [latex]\gamma _y:[0,1]\to X[/latex] von [latex]x[/latex] nach [latex]y[/latex]. Nach Proposition 9.43 ist [latex]\gamma _y([0,1])[/latex] für jedes [latex]y \in X[/latex] zusammenhängend. Nach Wahl unserer Wege haben wir

[latex]
\begin{aligned}[]X = \bigcup _{y \in X} \gamma _y([0,1]).\end{aligned}
[/latex]

Da aber [latex]x = \gamma _y(0) \in \gamma _y([0,1])[/latex] liegt für jedes [latex]y \in X[/latex], ist der Schnitt [latex]\bigcap _{y \in X} \gamma _y([0,1])[/latex] nicht-leer. Nach der Verallgemeinerung von Lemma 9.31 in Übung 9.32 ist also [latex]X = \bigcup _{y \in X} \gamma _y([0,1])[/latex] zusammenhängend. In der Tat, ist [latex]A \subseteq X[/latex] offen und abgeschlossen und ohne Beschränkung der Allgemeinheit [latex]x \in A[/latex], dann ist [latex]A \cap \gamma _y([0,1])[/latex] offen und abgeschlossen in [latex]\gamma _y([0,1])[/latex] und somit [latex]\gamma _y([0,1]) \subseteq A[/latex] für alle [latex]y \in X[/latex]. ∎

Umgekehrt muss ein zusammenhängender Raum nicht wegzusammenhängend sein, wie folgende Übung zeigt.

Übung 9.48: Zusammenhang v.s. Wegzusammenhang

Zeigen Sie, dass der Teilraum

[latex]
\begin{aligned}[]X = \left \lbrace {0} \right \rbrace \times [-1,1] \sqcup \left \lbrace {(t,\sin (\tfrac {1}{t})) \in \mathbb {R}^2} \mid {t > 0}\right \rbrace \subseteq \mathbb {R}^2\end{aligned}
[/latex]

zusammenhängend, aber nicht wegzusammenhängend ist.

Hinweis.

Für den Zusammenhang können Sie per Widerspruch annehmen, dass [latex]X \subseteq O_1\cup O_2[/latex] für zwei disjunkte, nicht-leere, offene Teilmengen von [latex]\mathbb {R}^2[/latex]. Begründen Sie dann, dass [latex]\left \lbrace {0} \right \rbrace \times [-1,1][/latex] als auch [latex]\left \lbrace {(t,\sin (\tfrac {1}{t})) \in \mathbb {R}^2} \mid {t > 0}\right \rbrace[/latex] in einer dieser beiden Mengen liegen müssen und zeigen sie anschliessend, dass sie in der selben Menge liegen.

Um zu zeigen, dass [latex]X[/latex] nicht wegzusammenhängend ist, sei per Widerspruch [latex]\gamma[/latex] einen Weg in [latex]X[/latex] von [latex](1,\sin (1))[/latex] nach [latex](0,1)[/latex] und sei [latex]t_0 = \inf \left \lbrace {t} \mid {\gamma (t) \in \left \lbrace {0} \right \rbrace \times [-1,1]}\right \rbrace[/latex]. Betrachten Sie nun [latex]\gamma[/latex] in einer kleinen linksseitigen Umgebung [latex](t_0-\varepsilon ,t_0][/latex] von [latex]t_0[/latex].

Für gewisse Räume ist die Implikation «Zusammenhang [latex]\implies[/latex] Wegzusammenhang» trotzdem richtig.

Proposition 9.49

Sei [latex]O \subseteq \mathbb {R}^d[/latex] für [latex]d \geq 1[/latex] eine nicht-leere, offene Teilmenge. Dann ist [latex]O[/latex] genau dann wegzusammenhängend, wenn [latex]O[/latex] zusammenhängend ist.

Beweis

Ist [latex]O[/latex] wegzusammenhängend, so ist [latex]O[/latex] auch zusammenhängend nach Lemma 9.47.

Sei nun [latex]O[/latex] zusammenhängend und [latex]x_0 \in O[/latex] ein fester Punkt. Wir definieren die Teilmenge

[latex]
\begin{aligned}[]G = \left \lbrace {x \in O} \mid {\text {es gibt einen Weg in O von }x_0 \text { nach } x}\right \rbrace\end{aligned}
[/latex]

und wollen zeigen, dass [latex]G[/latex] sowohl offen als auch abgeschlossen ist.

Sei [latex]x \in G[/latex] und [latex]\gamma :[0,1] \to O[/latex] ein Weg von [latex]x_0[/latex] nach [latex]x[/latex]. Da [latex]O[/latex] offen ist, gibt es ein [latex]\varepsilon > 0[/latex] mit [latex]B_\varepsilon (x) \subseteq O[/latex]. Für ein [latex]y \in B_\varepsilon (x)[/latex] liegt der gerade Weg [latex]t \in [0,1] \mapsto (1-t)x + ty[/latex], der [latex]x[/latex] mit [latex]y[/latex] verbindet, in [latex]O[/latex]. «Klebt» man nun beide Wege zusammen, so erhält man den Weg

[latex]
\begin{aligned}[]t \in [0,2] \mapsto \left \lbrace \begin{array}{cc} \gamma (t) & \text {falls } 0 \leq t \leq 1 \\ (2-t)x + (t-1)y & \text {falls } 1 [/latex]

von [latex]x_0[/latex] nach [latex]y[/latex] (Stetigkeit folgt direkt oder aus Übung 9.40). Also liegt [latex]y \in G[/latex] und, da [latex]y[/latex] beliebig war, [latex]B_\varepsilon (x) \subseteq G[/latex]. Wir haben somit gezeigt, dass [latex]G[/latex] offen ist.

Mit einem ähnlichen Argument zeigt man, dass [latex]O \setminus G[/latex] offen ist. Ist [latex]x \not \in G[/latex] und [latex]\varepsilon > 0[/latex] mit [latex]B_\varepsilon (x) \subseteq O[/latex], so liegen alle Punkte in [latex]B_\varepsilon (x)[/latex] auch nicht in [latex]G[/latex]. Denn wäre [latex]y \in G \cap B_\varepsilon (x)[/latex], so könnte man durch eine Verkettung von Wegen wie oben auch [latex]x[/latex] mit [latex]x_0[/latex] durch einen Weg verbinden. Also ist [latex]B_\varepsilon (x) \subseteq O \setminus G[/latex] und [latex]O \setminus G[/latex] ist offen.

Da [latex]O[/latex] aber zusammenhängend ist, muss entweder [latex]G[/latex] oder [latex]O \setminus G[/latex] leer sein. Da [latex]x_0 \in G[/latex] liegt, ist also [latex]O\setminus G[/latex] leer und [latex]G = O[/latex]. Per Definition von [latex]G[/latex] lässt sich also jeder Punkt in [latex]O[/latex] durch einen Weg mit [latex]x_0[/latex] verbinden. Sind [latex]y,y' \in O[/latex] zwei beliebige Punkte, so ist die Verkettung eines Weges von [latex]y[/latex] nach [latex]x_0[/latex] mit einem Weg von [latex]x_0[/latex] nach [latex]y'[/latex] ein Weg von [latex]y[/latex] nach [latex]y'[/latex]. Also ist [latex]O[/latex] wegzusammenhängend. ∎

Übung 9.50: Verkettung von Wegen

Wir möchten hier die oben verwendete Verkettung nochmals im Allgemeinen behandeln. Sei [latex]X[/latex] ein metrischer Raum und sei [latex]\gamma :[0,1] \to X[/latex] ein Weg von [latex]x_0\in X[/latex] nach [latex]x_1 \in X[/latex].

  1. (Umkehren von Wegen) Zeigen Sie, dass [latex]t \in [0,1] \mapsto \gamma (1-t)[/latex] ein Weg von [latex]x_1[/latex] nach [latex]x_0[/latex] ist.
  2. (Verketten von Wegen) Sei [latex]\tilde {\gamma }:[0,1] \to X[/latex] ein Weg von [latex]x_1[/latex] nach [latex]x_2 \in X[/latex]. Zeigen Sie, dass
    [latex]
    \begin{aligned}[]t \in [0,1] \mapsto \left \lbrace \begin{array}{cc} \gamma (2t) & \text {falls } 0 \leq t \leq \tfrac {1}{2} \\ \tilde {\gamma }(2t-1) & \text {falls } \tfrac {1}{2} [/latex]

    einen Weg von [latex]x_0[/latex] nach [latex]x_2[/latex] definiert.

  3. Wiederholen Sie damit den Beweis von Proposition 9.49.

9.4 – Vollständigkeit

Definition 9.51: Cauchy-Folgen und Vollständigkeit

Sei [latex](X,\mathrm {d})[/latex] ein metrischer Raum. Eine Folge [latex](x_n)_n[/latex] in [latex]X[/latex] heisst Cauchy-Folge, falls es zu jedem [latex]\varepsilon > 0[/latex] ein [latex]N \in \mathbb {N}[/latex] gibt, so dass für alle [latex]m,n\geq N[/latex] die Abschätzung [latex]\mathrm {d}(x_n,x_m) vollständig, falls jede Cauchy-Folge in [latex]X[/latex] konvergiert.

Genauso wie für den Konvergenzbegriff ist der Begriff einer Cauchy-Folge in einem normierten Raum (siehe Abschnitt 5.7.4) kompatibel mit obiger Definition.

Wie dies schon für die reellen Zahlen erwähnt wurde, ist Vollständigkeit gewissermassen jene Eigenschaft, die analytische Methoden erlaubt. Dies ist beispielsweise auch für [latex]\mathbb {R}^d[/latex] mit [latex]d \geq 1[/latex] wahr.

Proposition 9.52: Vollständigkeit in endlich dimensionalen Vektorräumen

Sei [latex]d \geq 1[/latex]. Jede abgeschlossene Teilmenge von [latex]\mathbb {R}^d[/latex] ist vollständig.

Man beachte, dass beliebige Teilmengen von [latex]\mathbb {R}^d[/latex] nicht zwingend vollständig sind; beispielsweise ist [latex](\frac {1}{n})_n[/latex] eine Cauchy-Folge in [latex]X = (0,1)[/latex], konvergiert aber gegen [latex]0[/latex] in [latex]\mathbb {R}[/latex] und hat somit keinen Grenzwert in [latex]X[/latex].

Beweis

Nach Satz 5.95 ist [latex]\mathbb {R}^d[/latex] vollständig. Sei nun [latex]A \subseteq \mathbb {R}^d[/latex] eine abgeschlossene Teilmenge und sei [latex]({a}_n)_n[/latex] eine Cauchy-Folge in [latex]A[/latex]. Dann besitzt die Folge [latex]({a}_n)_n[/latex] einen Grenzwert [latex]{b}\in \mathbb {R}^d[/latex]. Nach Lemma 9.19 muss aber [latex]{b}[/latex] in [latex]A[/latex] liegen, womit die Cauchy-Folge [latex]({a}_n)_n[/latex] also in [latex]A[/latex] konvergiert. Somit ist [latex]A[/latex] vollständig. ∎

Übung 9.53: Vollständigkeit und Abgeschlossenheit

Sei [latex](X,\mathrm {d})[/latex] ein metrischer Raum und [latex]A \subseteq X[/latex] eine Teilmenge.

  1. Angenommen [latex]X[/latex] ist vollständig und [latex]A[/latex] ist abgeschlossen. Zeigen Sie, dass der Teilraum [latex]A[/latex] auch vollständig ist.
  2. Angenommen [latex]A[/latex] ist vollständig. Zeigen Sie, dass [latex]A \subseteq X[/latex] abgeschlossen ist.

Genauso wie bei der Diskussion von Konvergenz wollen wir einen Vektorraum [latex]V[/latex] über [latex]\mathbb {R}[/latex] schlicht vollständig bezüglich einer Norm [latex]\| {\cdot }\|[/latex] nennen, falls er bezüglich der induzierten Metrik vollständig ist. Wie in Abschnitt 5.7.4 ist Vollständigkeit stabil bezüglich Normäquivalenz: Ist [latex]V[/latex] ein Vektorraum über [latex]\mathbb {R}[/latex] und sind [latex]\| {\cdot }\| _1[/latex] und [latex]\| {\cdot }\| _2[/latex] zwei äquivalente Normen auf [latex]V[/latex], so ist [latex]V[/latex] genau dann bezüglich [latex]\| {\cdot }\| _1[/latex] vollständig, wenn [latex]V[/latex] bezüglich [latex]\| {\cdot }\| _2[/latex] vollständig ist.

Bemerkung

Ein Banachraum ist per Definition ein vollständiger normierter Vektorraum. Interessant sind Banachräume unter anderem deswegen, weil sie (und Abbildungen auf ihnen) in der Praxis häufig vorkommen und oft relativ gute Eigenschaften besitzen. Für eine detaillierte Behandlung von Banachräumen verweisen wir auf die Vorlesung der Funktionalanalysis im dritten Studienjahr. Wird die Norm in einem Banachraum noch zusätzlich von einem inneren Produkt induziert, so spricht man von einem Hilbertraum. Hilberträume spielen unter anderem in der vollständigen Behandlung von Fourier-Reihen (siehe Appendix) eine fundamentale Rolle.

9.4.1 – Der Banachsche Fixpunktsatz

Sogenannte Fixpunktsätze beschreiben, wann es Punkte gibt, die unter einer Abbildung oder einer anderen Operation festgehalten werden. Sie sind insbesondere oft dann nützlich, wenn es darum geht, Existenzsätze zu beweisen. Beispielsweise werden wir den folgenden Fixpunktsatz für den Beweis der Existenz von Lösungen zu Differentialgleichungen verwenden.

Satz 9.54: Banachscher Fixpunktsatz

Sei [latex](X,\mathrm {d})[/latex] ein nicht-leerer, vollständiger metrischer Raum. Sei [latex]T: X \to X[/latex] eine Lipschitz-Kontraktion, das heisst, eine Abbildung mit der Eigenschaft

[latex]
\begin{aligned}[]\mathrm {d}(T(x_1),T(x_2)) \leq \lambda \mathrm {d}(x_1,x_2)\end{aligned}
[/latex]

für alle [latex]x_1,x_2 \in X[/latex] und für eine fixe Lipschitz-Konstante [latex]\lambda Fixpunkt von [latex]T[/latex]).

Wir möchten an dieser Stelle anmerken, dass die getroffenen Annahmen in Satz 9.54 notwendig sind — siehe Übung 9.55. Der Banachsche Fixpunktsatz besitzt auch durchaus alltägliche Anwendungen. Beispielsweise zeigt er, dass wenn ein Tourist an der ETH eine Stadtkarte von Zürich zu Boden fallen lässt, ein Punkt auf sich selbst zu liegen kommt (wieso?).

Beweis

Wir zeigen zuerst die behauptete Eindeutigkeit. Seien also [latex]x_0,x_0' \in X[/latex] zwei Fixpunkte von [latex]T[/latex]. Dann gilt

[latex]
\begin{aligned}[]\mathrm {d}(x_0,x_0') = \mathrm {d}(T(x_0),T(x_0')) \leq \lambda \mathrm {d}(x_0,x_0'),\end{aligned}
[/latex]

was wegen [latex]\lambda

Für die Existenz wählen wir ein beliebiges [latex]x_1 \in X[/latex] und definieren rekursiv [latex]x_2 = T(x_1)[/latex], [latex]x_3 = T(x_2)[/latex] und allgemein [latex]x_{n+1} = T(x_n)[/latex] für alle [latex]n \in \mathbb {N}[/latex] (die Folge [latex](T^{\, \circ \, {n} }(x_1))_n[/latex] nennt sich auch die Bahn von [latex]x_1[/latex] unter [latex]T[/latex]). Wir möchten nun zeigen, dass die Folge [latex](x_n)_n[/latex] konvergiert. In der Tat ist der Grenzwert [latex]x_0[/latex] dann ein Fixpunkt, denn es gilt

[latex]
\begin{aligned}[]x_0 = \lim _{n \to \infty }x_n = \lim _{n \to \infty }x_{n+1} = \lim _{n \to \infty }T(x_n) = T(\lim _{n \to \infty }x_n ) = T(x_0),\end{aligned}
[/latex]

da [latex]T:X \to X[/latex] Lipschitz-stetig und somit auch stetig ist.

Wir bemerken zuerst, dass [latex]\mathrm {d}(x_2,x_3) = \mathrm {d}(T(x_1),T(x_2)) \leq \lambda \mathrm {d}(x_1,x_2)[/latex] und allgemeiner
[latex]
\begin{aligned}[]\label{eq:metr-banachproof} \mathrm {d}(x_n,x_{n+1}) \leq \lambda ^{n-1} \mathrm {d}(x_1,x_2)\end{aligned}
[/latex]
für alle [latex]n \in \mathbb {N}[/latex]. In der Tat folgt Ungleichung (9.1) mittels Induktion [latex]n[/latex]. Für [latex]n=1[/latex] ist die Ungleichung tautologisch erfüllt. Falls (9.1) bereits für [latex]n[/latex] gilt, dann folgt

[latex]
\begin{aligned}[]\mathrm {d}(x_{n+1},x_{n+2}) = \mathrm {d}(T(x_{n}),T(x_{n+1})) \leq \lambda \mathrm {d}(x_n,x_{n+1}) \leq \lambda ^{n} \mathrm {d}(x_1,x_2)\end{aligned}
[/latex]

nach Konstruktion, der Voraussetzung im Satz und der Induktionsverankerung.

Wir behaupten nun, dass Ungleichung (9.1) impliziert, dass [latex](x_n)_n[/latex] eine Cauchy-Folge in [latex]X[/latex] bildet. Daraus folgt mit der vorausgesetzten Vollständigkeit, dass ein Grenzwert [latex]x_0[/latex] der Folge [latex](x_n)_n[/latex] existiert, was wie oben erklärt dann den Beweis abschliesst. Sei [latex]\varepsilon > 0[/latex]. Dann existiert ein [latex]N \in \mathbb {N}[/latex] mit [latex]\frac {\lambda ^{N-1}}{1-\lambda }\mathrm {d}(x_1,x_2) m \geq N[/latex] folgt damit

[latex]
\begin{aligned}[]\mathrm {d}(x_m,x_n) &\leq \mathrm {d}(x_m,x_{m+1}) + \mathrm {d}(x_{m+1},x_{m+2}) + \ldots + \mathrm {d}(x_{n-1},x_n)\\ &\leq \lambda ^{m-1}\mathrm {d}(x_1,x_2) + \lambda ^m \mathrm {d}(x_1,x_2) + \ldots + \lambda ^{n-2} \mathrm {d}(x_1,x_2)\\ &\leq \sum _{k=N}^\infty \lambda ^{k-1} \mathrm {d}(x_1,x_2) = \frac {\lambda ^{N-1}}{1-\lambda }\mathrm {d}(x_1,x_2) [/latex]

was den Beweis abschliesst. ∎

Wir möchten kurz anmerken, dass der obige Ansatz einen Fixpunkt zu finden, rein geometrisch begründet ist. Ist ein Fixpunkt [latex]x_0 \in X[/latex] gegeben, so zieht die Abbildung [latex]T[/latex] alle Punkte näher an [latex]x_0[/latex] heran, denn es gilt

[latex]
\begin{aligned}[]\mathrm {d}(T(x),x_0) = \mathrm {d}(T(x),T(x_0)) \leq \lambda \mathrm {d}(x,x_0).\end{aligned}
[/latex]

Da also unter Iterieren von [latex]T[/latex] eine Folge entsteht, die [latex]x_0[/latex] immer näher kommt, ist das Vorgehen, den Fixpunkt [latex]x_0[/latex] als Grenzwert einer Bahn zu konstruieren, sehr sinnvoll.

Übung 9.55: Notwendigkeit der Annahmen in Satz 9.54

  • Finden Sie einen nicht-vollständigen metrischen Raum [latex]X[/latex] und eine Lipschitz-Kontra­ktion [latex]T: X \to X[/latex], die keinen Fixpunkt besitzt.
  • Finden Sie einen vollständigen metrischen Raum [latex](X,\mathrm {d})[/latex] und eine Isometrie (das heisst, eine Abbildung [latex]T: X \to X[/latex] mit [latex]\mathrm {d}(T(x_1),T(x_2)) = \mathrm {d}(x_1,x_2)[/latex]), die keinen Fixpunkt besitzt.

9.4.2 – Wahrscheinlichkeitsmatrizen*

Wir wollen als erste Anwendung des Banachschen Fixpunktsatzes die Existenz eines «sta­tio­nären Wahrscheinlichkeitsvektors» zu einer «positiven Wahrscheinlichkeitsmatrix» zeigen.

Definition 9.56

Sei [latex]d \geq 1[/latex]. Eine [latex]d \times d[/latex]-Matrix [latex]A \in \operatorname {Mat}_{d,d}(\mathbb {R})[/latex] heisst nicht-negativ (positiv), falls [latex]A_{k \ell } \geq 0[/latex] ([latex]A_{k \ell } >0[/latex]) für alle [latex]k,\ell \in \left \lbrace {1,\ldots ,d} \right \rbrace[/latex] gilt. Eine nicht-negative Matrix [latex]P \in \operatorname {Mat}_{d,d}(\mathbb {R})[/latex] ist eine Wahrscheinlichkeitsmatrix, falls [latex]\sum _{k=1}^d P_{k\ell } =1[/latex] für alle [latex]\ell \in \left \lbrace {1,\ldots ,d} \right \rbrace[/latex] gilt. Ein Vektor [latex]{a} \in \mathbb {R}^d[/latex] ist ein Wahrscheinlichkeitsvektor, falls [latex]a_j \geq 0[/latex] für alle [latex]j \in \left \lbrace {1,\ldots ,d} \right \rbrace[/latex] und [latex]\sum _{j=1}^d a_j = 1[/latex]. Ein Wahrscheinlichkeitsvektor [latex]{a} \in \mathbb {R}^d[/latex] heisst stationär bezüglich einer Wahrscheinlichkeitsmatrix [latex]P \in \operatorname {Mat}_{d,d}(\mathbb {R})[/latex], falls [latex]P {a} = {a}[/latex] gilt (also falls [latex]{a}[/latex] ein Eigenvektor von [latex]P[/latex] zum Eigenwert [latex]1[/latex] ist).

In Anwendungen von obigen Begriffen und dem Resultat unten ist [latex]\left \lbrace {1,\ldots ,d} \right \rbrace[/latex] eine Abzähl­ung von verschiedenen Zuständen eines gegebenen Systems. Die Wahrscheinlichkeitsmatrix [latex]P[/latex] gibt zu Zuständen [latex]k,\ell \in \left \lbrace {1,\ldots ,d} \right \rbrace[/latex] die Wahrscheinlichkeit [latex]P_{k\ell } \in [0,1][/latex] eines Übergangs vom Zustand [latex]\ell[/latex] zum [latex]k[/latex] an. Des Weiteren gibt ein stationärer Wahrscheinlichkeitsvektor (einen solchen wollen wir finden) eine «zeitunabhängige Wahrscheinlichkeitsverteilung» über die verschiedenen Zustände an.

Zum Beispiel könnten für [latex]d = 5[/latex] die Zustände das Winterwetter in Zürich angeben: Sonne, Nebel, Wolken, Regen, Schneefall. Die Matrix [latex]P[/latex] beschreibt wie gesagt die Wahrscheinlichkeiten der Übergänge. Beispielsweise gibt [latex]P_{11}[/latex] die Wahrscheinlichkeit an, dass ausgehend von einem sonnigen Tag der nächste Tag wieder sonnig ist; [latex]P_{21}[/latex] gibt die Wahrscheinlichkeit an, dass ausgehend von einem sonnigen Tag der nächste Tag neblig ist und so weiter. Unser Beweis des Resultats weiter unten gemeinsam mit dem Banachschen Fixpunktsatz hat in diesem Zusammenhang die folgende Interpretation. Ausgehend von einem sonnigen Tag gibt also die erste Spalte von [latex]P[/latex] den Wahrscheinlichkeitsvektor an, der das Wetter am nächsten Tag beschreibt. Wendet man nochmals [latex]P[/latex] an, erhält man den Wahrscheinlichkeitsvektor, der das Wetter am nächsten Tag beschreibt. Diese Vektoren stabilisieren sich zu einem stationären Wahrscheinlichkeitsvektor. Dieser Vektor ist derselbe, wenn wir stattdessen von einem Tag mit Nebel, Wolken, Regen oder Schneefall ausgehen.

Korollar 9.57

Sei [latex]d \geq 1[/latex] und [latex]P \in \operatorname {Mat}_{d,d}(\mathbb {R})[/latex] eine positive Wahrscheinlichkeitsmatrix. Dann existiert ein eindeutig bestimmter stationärer Wahrscheinlichkeitsvektor zu [latex]P[/latex], der auch [latex]a_j > 0[/latex] für alle [latex]j \in \left \lbrace {1,\ldots ,d} \right \rbrace[/latex] erfüllt.

Dieser Satz ist ein Spezialfall des Satzes von Perron-Frobenius, welcher ähnliche Existenzaussagen für allgemeinere, nicht-negative Matrizen trifft (siehe Übung 9.58).

Beweis

Wir betrachten den Teilraum

[latex]
\begin{aligned}[]X = \left \lbrace {{a} \in \mathbb {R}^d} \mid {{a} \text { ist ein Wahrscheinlichkeitsvektor}}\right \rbrace .\end{aligned}
[/latex]

Dieser ist abgeschlossen: Falls [latex]({a}_n)_n[/latex] eine Folge in [latex]X[/latex] ist und [latex]{a}_n \to {a} \in \mathbb {R}^d[/latex] gilt, dann ist wegen [latex]\pi _j({a}_n) \geq 0[/latex] für [latex]j \in \left \lbrace {1,\ldots ,d} \right \rbrace[/latex] auch [latex]\pi _j({a}) \geq 0[/latex] auf Grund der Stetigkeit von [latex]\pi _j[/latex]. Des Weiteren ist [latex]\sum _{j=1}^d \pi _j({a}_n) = 1[/latex] und somit auch [latex]\sum _{j=1}^d \pi _j({a}) = 1[/latex] nach Stetigkeit der Abbildung [latex]{b} \mapsto \sum _{j=1}^d \pi _j({b})[/latex]. Also ist [latex]X[/latex] abgeschlossen nach der Charakterisierung in Lemma 9.19.

Nach Proposition 9.52 ist [latex]X[/latex] also auch vollständig, wobei wir nach Normäquivalenz wahlweise die Euklidsche Norm [latex]\| {\cdot }\| _2[/latex] oder die [latex]1[/latex]-Norm [latex]\| {\cdot }\| _1[/latex] verwenden dürfen. Wir werden die Norm [latex]\| {\cdot }\| _1[/latex] und ihre induzierte Metrik [latex]\mathrm {d}_1[/latex] verwenden.

Wir identifizieren die Matrix [latex]P[/latex] mit der zugehörigen linearen Abbildung [latex]P: \mathbb {R}^d \to \mathbb {R}^d[/latex] und definieren [latex]T= P|_X: X \to X[/latex]. Dies ist in der Tat wohldefiniert: Für [latex]{a}\in X[/latex] gilt

[latex]
\begin{aligned}[](P{a})_k = \sum _{\ell =1}^d P_{k\ell }a_\ell \geq 0\end{aligned}
[/latex]

für alle [latex]k\in \left \lbrace {1,\ldots ,d} \right \rbrace[/latex] sowie

[latex]
\begin{aligned}[]\sum _{k=1}^d (P{a})_k = \sum _{k=1}^d \sum _{\ell =1}^d P_{k\ell }a_\ell = \sum _{\ell =1}^d a_\ell \bigg (\sum _{k=1}^d P_{k\ell }\bigg ) = \sum _{\ell =1}^d a_\ell = 1\end{aligned}
[/latex]

und somit [latex]P{a} = T({a}) \in X[/latex].

Wir möchten nun zeigen, dass [latex]T[/latex] eine Lipschitz-Kontraktion von [latex]X[/latex] ist. Sind [latex]{a},{a'} \in X[/latex], dann gilt

[latex]
\begin{aligned}[]\mathrm {d}_1(P{a},P{a'}) = \| {P{a}-P{a'}}\| _1 = \| {P{y}}\| _1\end{aligned}
[/latex]

für [latex]{y} = {a}-{a'} \in Y = \left \lbrace {{y} \in \mathbb {R}^d} \mid {\sum _{\ell =1}^d y_\ell =0}\right \rbrace[/latex]. Man beachte dabei, dass [latex]P(Y) \subseteq Y[/latex] gilt, da für alle [latex]y \in Y[/latex] die Identität [latex]\sum _{k=1}^d (P{y})_k = \sum _{k=1}^d \sum _{\ell =1}^d P_{k\ell }y_\ell = \sum _{\ell =1}^d y_\ell \sum _{k=1}^d P_{k\ell } = \sum _{\ell =1}^d y_\ell = 0[/latex] erfüllt ist. Wir möchten nun zeigen, dass [latex]\| {P{y}}\| _1 \leq (1-\eta ) \| {y}\| _1[/latex] für alle [latex]{y} \in Y[/latex] und [latex]\eta = \min _{k,\ell }P_{k\ell } > 0[/latex] gilt, womit nach obigem [latex]T[/latex] eine Lipschitz-Kontraktion mit Lipschitz-Konstante [latex]\lambda = 1-\eta

Sei nun [latex]{y} \in Y[/latex]. Wir definieren die Indexmengen

[latex]
\begin{aligned}[]L^+ = \big \lbrace \ell \in \left \lbrace {1,\ldots ,d} \right \rbrace \ |\ y_\ell > 0\big \rbrace , \quad L^- = \big \lbrace \ell \in \left \lbrace {1,\ldots ,d} \right \rbrace \ |\ y_\ell [/latex]

und [latex]K^+ = \big \lbrace \ell \in \left \lbrace {1,\ldots ,d} \right \rbrace \ |\ (P{y})_\ell > 0\big \rbrace[/latex]. Dann gilt

[latex]
\begin{aligned}[]\| {{y}}\| _1 = \sum _{\ell =1}^d |y_\ell | = \sum _{\ell \in L^+} y_\ell - \sum _{\ell \in L^-}y_\ell = 2\sum _{\ell \in L^+} y_\ell = 2\sum _{\ell \in L^-} |y_\ell |\end{aligned}
[/latex]

auf Grund von [latex]\sum _{\ell =1}^d y_\ell = 0[/latex] und aus demselben Grund

[latex]
\begin{aligned}[]\| {P{y}}\| _1 = 2 \sum _{k \in K^+} (P{y})_k.\end{aligned}
[/latex]

Daher ist

[latex]
\begin{aligned}[]\| {P{y}}\| _1 &= 2 \sum _{k \in K^+} \sum _{\ell =1}^d P_{k\ell }y_\ell = 2 \sum _{k \in K^+} \left ( \sum _{\ell \in L^+} P_{k\ell }y_\ell + \sum _{\ell \in L^-} P_{k\ell }y_\ell \right ) \\ &\leq 2 \sum _{\ell \in L^+} y_\ell \sum _{k \in K^+} P_{k\ell } - 2 \sum _{k \in K^+} \sum _{\ell \in L^-}\eta |y_\ell | \\ &\leq 2 \sum _{\ell \in L^+} y_\ell - |K^+|\ \eta \| {{y}}\| _1 \leq (1-\eta ) \| {{y}}\| _1,\end{aligned}
[/latex]

was wir zeigen wollten. Somit ist [latex]T[/latex] eine Lipschitz-Kontraktion. Nach dem Banachschen Fixpunktsatz (Satz 9.54) hat [latex]T[/latex] also einen eindeutigen Fixpunkt [latex]{a} \in X[/latex].

Wir müssen noch zeigen, dass [latex]a_k > 0[/latex] für alle [latex]k \in \left \lbrace {1,\ldots ,d} \right \rbrace[/latex] gilt. In der Tat ist

[latex]
\begin{aligned}[]a_k = (P {a})_k = \sum _{\ell =1}^d P_{k\ell } a_\ell\end{aligned}
[/latex]

genau dann gleich null, wenn [latex]P_{k\ell } a_\ell = 0[/latex] und somit, da [latex]P[/latex] positiv ist, [latex]a_\ell = 0[/latex] für alle [latex]\ell[/latex] in [latex]\left \lbrace {1,\ldots ,d} \right \rbrace[/latex] gilt. Dies ist aber nicht möglich, da [latex]{a}[/latex] ein Wahrscheinlichkeitsvektor ist. ∎

Übung 9.58: Verallgemeinerungen

  1. Sei [latex]P \in \operatorname {Mat}_{d,d}(\mathbb {R})[/latex] eine Wahrscheinlichkeitsmatrix mit einer positiven Potenz, zum Beispiel [latex]P = \left (\begin{smallmatrix}\frac {1}{2} & 1 \\ \frac {1}{2} & 0\end{smallmatrix} \right )[/latex]. Zeigen Sie, dass [latex]P[/latex] ebenso die Konklusionen in Korollar 9.57 erfüllt und beweisen Sie, dass alle von [latex]1[/latex] verschiedenen Eigenwerte von [latex]P[/latex] Absolutbetrag kleiner [latex]1[/latex] haben.
  2. Sei [latex]P \in \operatorname {Mat}_{d,d}(\mathbb {R})[/latex] eine Wahrscheinlichkeitsmatrix mit der Eigenschaft, dass es für jedes Indexpaar [latex]k,\ell \in \left \lbrace {1,\ldots ,d} \right \rbrace[/latex] ein [latex]n \in \mathbb {N}[/latex] gibt mit [latex](P^n)_{k,\ell } > 0[/latex], zum Beispiel [latex]P = \left (\begin{smallmatrix}0 & 1 \\ 1 & 0\end{smallmatrix} \right )[/latex]. Zeigen Sie, dass [latex]P[/latex] ebenso die Konklusionen in Korollar 9.57 erfüllt und beweisen Sie, dass alle von [latex]1[/latex] verschiedenen Eigenwerte von [latex]P[/latex] Absolutbetrag kleiner [latex]1[/latex] haben oder eine Wurzel aus [latex]1[/latex] sind.
  3. Finden Sie weitere Beispiele von Wahrscheinlichkeitsmatrizen, wo andere Phänomene eintreten und obige Aussagen nicht immer gelten.

9.5 – Kompaktheit

Wir haben in Abschnitt 3.8 die fundamentalen Eigenschaften Beschränktheit, Existenz eines Maximums (und Minimums) und gleichmässige Stetigkeit von stetigen Funktionen auf kompakten Intervallen bewiesen. Diese Eigenschaften waren dann grundlegend für die Riemann-Integrierbarkeit von stetigen Funktionen (siehe Satz 4.42), den Satz von Rolle und den Mittelwertsatz (Satz 7.28 und Theorem 7.29), und damit für die gesamte Differential- und Integralrechnung des ersten Semesters. Des Weiteren hatten wir in Satz 5.44 gesehen, dass jede Folge in einem kompakten Intervall eine konvergente Teilfolge besitzt. Wir wollen diese und auch noch weitere Eigenschaften im grösseren Rahmen der metrischen Räume untersuchen und werden feststellen, dass viele dieser Eigenschaften äquivalent sind.

9.5.1 – Definitionen

Definition 9.59: Folgenkompaktheit und Beschränktheit

Sei [latex]X[/latex] ein metrischer Raum. Wir sagen, dass [latex]X[/latex] folgenkompakt ist, falls jede Folge in [latex]X[/latex] eine konvergente Teilfolge (mit Grenzwert in [latex]X[/latex]) besitzt. Ein Teilraum [latex]K \subseteq X[/latex] ist folgenkompakt, falls er als eigenständiger metrischer Raum folgenkompakt ist. Eine Teilmenge [latex]B \subseteq X[/latex] ist beschränkt, falls es ein Punkt [latex]x_0 \in X[/latex] und einen Radius [latex]R > 0[/latex] gibt, so dass [latex]B[/latex] im Ball [latex]B_R(x_0)[/latex] enthalten ist.

Da die Definition der Folgenkonvergenz nur von der Topologie (der Familie der offenen Teilmengen) abhängt, ist Folgenkompaktheit ein topologischer Begriff. Ganz im Gegensatz dazu ist Beschränktheit stark von der Wahl der Metrik abhängig. Betrachtet man beispielsweise [latex]\mathbb {R}[/latex] mit den Metriken [latex]\mathrm {d}[/latex] und [latex]\mathrm {d}'[/latex], wobei [latex]\mathrm {d}[/latex] die Euklidsche Metrik ist und [latex]\mathrm {d}'[/latex] die durch [latex]\mathrm {d}'(x,y) = \min \left \lbrace {\mathrm {d}(x,y),1} \right \rbrace[/latex] für [latex]x,y\in \mathbb {R}[/latex] definierte Metrik ist, so sieht man, dass [latex]\mathbb {R}[/latex] bezüglich [latex]\mathrm {d}'[/latex] beschränkt ist. Allerdings ist die durch [latex]\mathrm {d}'[/latex] gegebene Topologie die Standardtopologie (die durch [latex]\mathrm {d}[/latex] gegebene Topologie) auf [latex]\mathbb {R}[/latex]. (Wieso?

Für [latex]\varepsilon \in (0,1)[/latex] sind die Bälle von Radius [latex]\varepsilon[/latex] bezüglich [latex]\mathrm {d}[/latex] die gleichen, wie die Bälle von Radius [latex]\varepsilon[/latex] bezüglich [latex]\mathrm {d}'[/latex].

)

Wir betrachten nun die Beziehung zwischen den beiden Begriffen «Folgenkompaktheit» und «Beschränktheit» .

Lemma 9.60: Notwendige Eigenschaften

Eine folgenkompakte Teilmenge eines nicht-leeren metrischen Raumes ist abgeschlossen und beschränkt.

Man beachte, dass die Umkehrung des Lemmas falsch ist (bezüglich der Metrik [latex]\mathrm {d}'[/latex] ist [latex]\mathbb {R}[/latex] abgeschlossen und beschränkt, aber nicht folgenkompakt — wieso?).

Beweis

Sei [latex]X[/latex] ein nicht-leerer metrischer Raum und [latex]K \subseteq X[/latex] folgenkompakt. Sei [latex](x_n)_n[/latex] eine Folge in [latex]K[/latex] mit Grenzwert [latex]x\in X[/latex]. Nach Definition 9.59 existiert eine in [latex]K[/latex] konvergente Teilfolge von [latex](x_n)_n[/latex]. Da eine Teilfolge einer konvergenten Folge denselben Grenzwert wie die ursprüngliche Folge besitzt, muss also [latex]x[/latex] in [latex]K[/latex] liegen. Nach der Charakterisierung abgeschlossener Teilmengen aus Lemma 9.19 ist [latex]K[/latex] abgeschlossen.

Sei nun [latex]x_0 \in X[/latex] ein beliebiger Punkt und angenommen [latex]K[/latex] ist nicht beschränkt. Dann gibt es für jedes [latex]n \in \mathbb {N}[/latex] ein [latex]x_n \in K[/latex] mit [latex]\mathrm {d}(x_n,x_0) \geq n[/latex]. Da [latex]K[/latex] folgenkompakt ist, existiert nun eine in [latex]K[/latex] konvergente Teilfolge [latex](x_{n_\ell })_\ell[/latex] der Folge [latex](x_n)_n[/latex]. Sei [latex]x \in K[/latex] ihr Grenzwert. Insbesondere gibt es zu [latex]\varepsilon = 1[/latex] ein [latex]L \in \mathbb {N}[/latex], so dass [latex]\mathrm {d}(x_{n_\ell },x)

[latex]
\begin{aligned}[]n_\ell \leq \mathrm {d}(x_{n_\ell },x_0) \leq \mathrm {d}(x_{n_\ell },x) + \mathrm {d}(x,x_0) [/latex]

Dies ist ein Widerspruch, da [latex]n_\ell \to \infty[/latex] für [latex]\ell \to \infty[/latex], und somit ist [latex]K[/latex] beschränkt. ∎

Wir möchten uns nun einem fundamentalen, weiteren Kompaktheitsbegriff zuwenden.

Definition 9.61: Überdeckungskompaktheit

Sei [latex]X[/latex] ein metrischer Raum.

  • Eine offene Überdeckung [latex]\mathcal {O}[/latex] von [latex]X[/latex] ist eine Kollektion offener Teilmengen von [latex]X[/latex], für die [latex]X = \bigcup _{O \in \mathcal {O}}O[/latex] gilt. Eine (endliche) Teilüberdeckung einer offenen Überdeckung [latex]\mathcal {O}[/latex] ist eine (endliche) Teilmenge [latex]\mathcal {O}' \subseteq \mathcal {O}[/latex], welche selbst eine offene Überdeckung bildet.
  • Der Raum [latex]X[/latex] ist (abzählbar) überdeckungskompakt, falls für jede (abzählbare) offene Überdeckung eine endliche Teilüberdeckung existiert.
  • Wir sagen, dass [latex]X[/latex] das (abzählbare) Schachtelungsprinzip erfüllt, falls für jede (abzählbare) Kollektion [latex]\mathcal {A}[/latex] abgeschlossener Teilmengen von [latex]X[/latex] mit [latex]A_{1} \cap \ldots \cap A_n \neq \emptyset[/latex] für alle [latex]n\in \mathbb {N}[/latex] und [latex]A_1,\ldots ,A_n \in \mathcal {A}[/latex] (auch endliche Schnitteigenschaft genannt) auch [latex]\bigcap _{A \in \mathcal {A}}A \neq \emptyset[/latex] gilt.

Beispiel 9.62: Offene Überdeckungen und Überdeckungskompaktheit

  1. Jeder endliche metrische Raum [latex]X[/latex] ist überdeckungskompakt. In der Tat ist die Potenzmenge [latex]\mathcal {P}(X)[/latex] endlich und somit jede offene Überdeckung bereits endlich.
  2. Der Teilraum [latex]X = (0,1] \subseteq \mathbb {R}[/latex] ist nicht überdeckungskompakt. Tatsächlich besitzt die offene Überdeckung [latex]\mathcal {O} = \left \lbrace {\left (\frac {1}{n},1\right ]} \mid {n \geq 2}\right \rbrace[/latex] keine endliche Teilüberdeckung. (Wieso?).
  3. Falls wir stattdessen den Teilraum [latex]X = [0,1] \subseteq \mathbb {R}[/latex] betrachten, dann bildet die Kollektion [latex]\mathcal {O} = \left \lbrace {\left (\frac {1}{n},1\right ] \cap X} \mid {n \in \mathbb {N}}\right \rbrace[/latex] keine offene Überdeckung mehr, da [latex]0 \not \in \left (\frac {1}{n},1\right ][/latex] für alle [latex]n \in \mathbb {N}[/latex]. Geben wir zu dieser Kollektion eine weitere offene Menge, die [latex]0[/latex] enthält (wie zum Beispiel [latex][0,\varepsilon ) = (-\varepsilon ,\varepsilon ) \cap X[/latex] für [latex]\varepsilon > 0[/latex]), hinzu, dann erhalten wir eine offene Überdeckung, die eine endliche Teilüberdeckung besitzt (wieso gilt dies im Beispiel?). In der Tat ist [latex][0,1][/latex] überdeckungskompakt (siehe Satz 9.66 weiter unten).

Überdeckungen spielen bei vielen Beweisen eine wichtige Rolle, wie wir später sehen werden. Überdeckungskompakte Räume sind in diesem Zusammenhang einfach zu handhaben, weil man für eine gegebene Überdeckung (nach Annahme) meist nur endlich viele offene Mengen zu «kontrollieren» braucht.

Übung 9.63

Sei [latex]X[/latex] ein metrischer Raum. Zeigen Sie, dass [latex]X[/latex] genau dann überdeckungs­kompakt ist, wenn jede offene Überdeckung von [latex]X[/latex] bestehend aus offenen Bällen eine endliche Teilüberdeckung besitzt.

Definition 9.64: Lebesgue-Zahl

Sei [latex]X[/latex] ein metrischer Raum und [latex]\mathcal {O}[/latex] eine offene Überdeckung von [latex]X[/latex]. Wir sagen, dass eine Zahl [latex]r>0[/latex] eine Lebesgue-Zahl für [latex]\mathcal {O}[/latex] ist, falls es für jedes [latex]x \in X[/latex] eine offene Menge [latex]O \in \mathcal {O}[/latex] gibt mit [latex]B_r(x) \subseteq O[/latex].

Wir bemerken, dass in obiger Definition die gleichmässige Wahl von der Zahl [latex]r[/latex] unabhängig von [latex]x\in X[/latex] entscheidend für die Definition ist. Wir benötigen noch eine weitere Definition, die den Begriff der Beschränkheit auf eine wesentliche Art und Weise verstärkt.

Definition 9.65: Total beschränkt

Sei [latex]X[/latex] ein metrischer Raum. Wir sagen, dass [latex]X[/latex] total beschränkt ist, falls es für jedes [latex]\varepsilon > 0[/latex] endlich viele [latex]x_1,\ldots ,x_n \in X[/latex] gibt mit [latex]X= \bigcup _{j=1}^n B_\varepsilon (x_j)[/latex].

Beispielsweise ist [latex][0,1][/latex] total beschränkt und [latex]\mathbb {R}[/latex] nicht total beschränkt (wieso?).

9.5.2 – Äquivalenzen

Der folgende Satz stellt obige Begriffe in direkte Beziehung zu einander.

Satz 9.66: Kompaktheit

Sei [latex]X[/latex] ein metrischer Raum. Dann sind folgende neun Eigenschaften äquivalent. Wenn diese erfüllt sind, so nennen wir [latex]X[/latex] einen kompakten metrischen Raum.

  1. Jede unendliche Teilmenge von [latex]X[/latex] besitzt einen Häufungspunkt.
  2. [latex]X[/latex] ist folgenkompakt (das heisst, jede Folge in [latex]X[/latex] hat eine in [latex]X[/latex] konvergente Teilfolge).
  3. [latex]X[/latex] ist abzählbar überdeckungskompakt (das heisst, für jede Folge [latex]O_1,O_2,\ldots[/latex] offener Teilmengen in [latex]X[/latex] mit [latex]X=\bigcup _{n\in \mathbb {N}}O_n[/latex] gibt es ein [latex]N\in \mathbb {N}[/latex] mit [latex]X=\bigcup _{n=1}^NO_n[/latex]).
  4. Jede stetige, komplexwertige Funktion auf [latex]X[/latex] ist beschränkt.
  5. Jede stetige, reellwertige Funktion auf [latex]X[/latex] nimmt ein Maximum und ein Minimum an.
  6. Jede offene Überdeckung von [latex]X[/latex] besitzt eine Lebesgue-Zahl und [latex]X[/latex] ist total beschränkt.
  7. [latex]X[/latex] ist überdeckungskompakt.
  8. [latex]X[/latex] erfüllt das Schachtelungsprinzip.
  9. [latex]X[/latex] ist total beschränkt und vollständig.

*Die Bedingungen (6)-(7) wurden in der Vorlesung nicht besprochen, sind daher auch nicht Prüfungsstoff. Proposition 9.67 wurde hingegen bewiesen und war auch mehrmals in den folgenden Kapiteln von Relevanz.

Beweis der Äquivalenz von (1)-(5)

Wir beweisen hier die Implikationen (1) [latex]\implies[/latex] (2) [latex]\implies[/latex] (3) [latex]\implies[/latex] (4) [latex]\implies[/latex] (5) [latex]\implies[/latex] (1).

Für die Implikation [latex](1) \implies (2)[/latex] betrachten wir eine Folge [latex](x_n)_n[/latex] in [latex]X[/latex] und die Bildmenge [latex]D = \left \lbrace {x_n} \mid {n \in \mathbb {N}}\right \rbrace[/latex]. Falls [latex]D[/latex] endlich ist, so hat [latex](x_n)_n[/latex] eine konstante und insbesondere konvergente Teilfolge. Wir nehmen nun also an, dass [latex]D[/latex] unendlich ist, womit [latex]D[/latex] nach [latex](1)[/latex] einen Häufungspunkt [latex]x_0 \in X[/latex] besitzt. Nach dem Analogon von Proposition 5.23 für metrische Räume — Übung 9.27 — reicht es zu zeigen, dass für [latex]\varepsilon > 0[/latex] und [latex]N \in \mathbb {N}[/latex] ein [latex]n\geq N[/latex] mit [latex]x_n \in B_{\varepsilon }(x_0)[/latex] existiert. Sei [latex]\varepsilon ' >0[/latex] so gewählt, dass

[latex]
\begin{aligned}[]B_{\varepsilon '}(x_0)\setminus \left \lbrace {x_0} \right \rbrace \subseteq B_{\varepsilon }(x_0) \setminus \left \lbrace {x_1,\ldots ,x_N} \right \rbrace .\end{aligned}
[/latex]

Da [latex]x_0[/latex] ein Häufungspunkt ist, gibt es ein [latex]n \in \mathbb {N}[/latex] mit [latex]x_n \in B_{\varepsilon '}(x_0)\setminus \left \lbrace {x_0} \right \rbrace \subseteq B_{\varepsilon }(x_0)[/latex], womit insbesondere [latex]n > N[/latex] sein muss.

Wir zeigen [latex](2) \implies (3)[/latex]. Sei also [latex]\mathcal {O} = \left \lbrace {O_1,O_2,\ldots } \right \rbrace[/latex] eine abzählbare offene Überdeckung von [latex]X[/latex]. Angenommen [latex]\mathcal {O}[/latex] besitzt keine endliche Teilüberdeckung. Dann gibt es für jedes [latex]n \in \mathbb {N}[/latex] einen Punkt [latex]x_n \in X \setminus \bigcup _{j=1}^n O_n[/latex]. Nach Vorraussetzung hat die Folge [latex](x_n)_n[/latex] eine konvergente Teilfolge [latex](x_{n_k})_k[/latex]; sei [latex]x_0 \in X[/latex] deren Grenzwert. Nach Konstruktion ist [latex]x_n \in X \setminus O_\ell[/latex] für alle [latex]n\geq \ell[/latex] und aus Lemma 9.19 folgt [latex]x_0 \in X \setminus O_\ell[/latex] für alle [latex]\ell \in \mathbb {N}[/latex]. Dies widerspricht aber der Annahme, dass [latex]\mathcal {O}[/latex] eine offene Überdeckung von [latex]X[/latex] ist.

Für die Implikation [latex](3) \implies (4)[/latex] sei [latex]f:X \to \mathbb {C}[/latex] eine stetige Funktion. Es gilt [latex]\mathbb {C} = \bigcup _{n\in \mathbb {N}}B_n(0)[/latex] und somit

[latex]
\begin{aligned}[]X = f^{-1}(\mathbb {C}) = \bigcup _{n\in \mathbb {N}} f^{-1}(B_n(0)).\end{aligned}
[/latex]

Nach Lemma 9.17 ist [latex]\mathcal {O} = \left \lbrace {f^{-1}(B_n(0))} \mid {n\in \mathbb {N}}\right \rbrace[/latex] eine abzählbare offene Überdeckung, womit nach (3) ein [latex]n\in \mathbb {N}[/latex] mit [latex]f^{-1}(B_n(0)) = \bigcup _{k=1}^n f^{-1}(B_k(0)) = X[/latex] existiert. Es gilt also [latex]|f(x)|

Der Beweis der Implikation [latex](4) \implies (5)[/latex] ist analog zum Beweis von Korollar 3.72; wir geben ihn hier der Vollständigkeit halber an. Sei [latex]f: X \to \mathbb {R}[/latex] stetig. Wir setzen [latex]M = \sup f(X)[/latex] und nehmen per Widerspruch an, dass [latex]f[/latex] sein Maximum nicht annimmt. Damit ist die Funktion

[latex]
\begin{aligned}[]g: X \to \mathbb {R},\ x \mapsto \frac {1}{M-f(x)}\end{aligned}
[/latex]

wohldefiniert und stetig. Nach Annahme in (4) gibt es nun ein [latex]S>0[/latex] mit [latex]g(x) \leq S[/latex] oder äquivalenterweise [latex]f(x) \leq M-\frac {1}{S}[/latex] für alle [latex]x\in X[/latex]. Dies widerspricht aber der Definition von [latex]M[/latex] als Supremum.

Wir zeigen nun [latex](5) \implies (1)[/latex]. Sei [latex]D \subseteq X[/latex] eine Teilmenge. Wir nehmen an, dass [latex]D[/latex] keine Häufungspunkte besitzt, und möchten nachweisen, dass [latex]D[/latex] endlich ist. Dazu möchten wir zuerst zeigen, dass es einen Radius [latex]r>0[/latex] gibt, so dass für [latex]x \in D[/latex] der Ball [latex]B_r(x)[/latex] keinen weiteren Punkt von [latex]D[/latex] enthält. Hierfür betrachten wir die Funktion [latex]\eta :X \to \mathbb {R}_{\geq 0}[/latex] gegeben durch

[latex]
\begin{aligned}[]\eta (x) = \sup \left \lbrace {\delta \geq 0} \mid {|B_\delta (x) \cap D| \leq 1}\right \rbrace\end{aligned}
[/latex]

für [latex]x \in X[/latex]. Da [latex]D[/latex] keine Häufungspunkte besitzt, können wir für jedes [latex]x \in X[/latex] ein [latex]\delta >0[/latex] finden mit [latex]|B_\delta (x) \cap D| \leq 1[/latex]. Somit ist [latex]\eta (x)>0[/latex] für alle [latex]x \in X[/latex]. Wir bemerken noch, dass daher

[latex]
\begin{aligned}[]\left \lbrace {\delta \geq 0} \mid {|B_\delta (x) \cap D| \leq 1}\right \rbrace =[0,\eta (x)]\end{aligned}
[/latex]

für alle [latex]x\in X[/latex] gilt. (Wieso?

Falls [latex]|B_{\eta (x)}(x) \cap D| \geq 2[/latex] wäre, dann gäbe es [latex]y_1\neq y_2[/latex] in [latex]B_{\eta (x)}(x) \cap D[/latex], die beide Abstand kleiner als [latex]\eta (x)[/latex] zu [latex]x[/latex] hätten, und daher wäre auch [latex]|B_{\delta }(x) \cap D|\geq 2[/latex] für alle [latex]\delta \in (\max (\mathrm {d}(x,y_1),\mathrm {d}(x,y_2)),\eta (x))[/latex].

)

Wir behaupten nun, dass [latex]\eta :X \to \mathbb {R}_{>0}[/latex] Lipschitz-stetig mit Lipschitz-Konstante [latex]1[/latex] (und insbesondere stetig) ist. Daraus folgt, dass [latex]r = \min \eta (X)[/latex] grösser Null ist und die oben gewünschte Eigenschaft erfüllt. Seien [latex]x_1,x_2 \in X[/latex]. Falls [latex]\varepsilon = \eta (x_1) - \mathrm {d}(x_1,x_2) > 0[/latex] ist, gilt [latex]B_\varepsilon (x_2) \subseteq B_{\eta (x_1)}(x_1)[/latex] auf Grund der Dreiecksungleichung, womit [latex]\eta (x_2) \geq \varepsilon = \eta (x_1) -\mathrm {d}(x_1,x_2)[/latex]. Falls hingegen [latex]\varepsilon = \eta (x_1) -\mathrm {d}(x_1,x_2)\leq 0[/latex] ist, gilt dies ebenfalls. Es gilt also in jedem Fall [latex]\eta (x_1) - \eta (x_2) \leq \mathrm {d}(x_1,x_2)[/latex]. Auf Grund der Symmetrie zwischen [latex]x_1[/latex] und [latex]x_2[/latex] schliessen wir also auf [latex]|\eta (x_1) - \eta (x_2)| \leq \mathrm {d}(x_1,x_2)[/latex] wie behauptet.

Wir nehmen nun zusätzlich an, dass [latex]D[/latex] unendlich ist und werden einen Widerspruch ableiten. Sei also [latex](x_n)_n[/latex] eine Folge in [latex]D[/latex] mit [latex]x_n \neq x_m[/latex] für alle [latex]n \neq m[/latex]. Zu [latex]n \in \mathbb {N}[/latex] und [latex]x \in X[/latex] setzen wir

[latex]
\begin{aligned}[]f(x) = \left \lbrace \begin{array}{cl} n \bigl (\tfrac 14r-\mathrm {d}(x,x_n)\bigr ) & \text { falls } x \in B_{r/4}(x_n) \text { für ein }n\in \mathbb {N} \text { und}\\ 0 & \text { sonst.}\end{array} \right .\end{aligned}
[/latex]

Nach Wahl von [latex]r[/latex] gibt es für ein gegebenes [latex]x\in X[/latex] höchstens ein [latex]n\in \mathbb {N}[/latex] mit [latex]x_n\in B_{r/2}(x)[/latex], womit insbesondere die Funktion [latex]f:X \to \mathbb {R}_{\geq 0}[/latex] wohldefiniert ist. Des Weiteren ist [latex]f[/latex] stetig bei jedem [latex]x\in X[/latex], denn falls es kein [latex]n\in \mathbb {N}[/latex] mit [latex]x_n\in B_{r/2}(x)[/latex] gibt, so ist [latex]f(x)=0=f(y)[/latex] für alle [latex]y\in B_{r/4}(x)[/latex], und falls es doch ein eindeutig bestimmtes [latex]n\in N[/latex] mit [latex]x_n\in B_{r/2}(x)[/latex] gibt, so ist [latex]f(y)=\max (0, n (\tfrac 14r-\mathrm {d}(y,x_n)))[/latex] für alle [latex]y\in B_{r/4}(x)[/latex] und damit ebenso bei [latex]x[/latex] stetig. Es gilt aber [latex]f(x_n) = \frac 14 n r[/latex] für alle [latex]n \in \mathbb {N}[/latex], womit [latex]f[/latex] unbeschränkt ist, was [latex](5)[/latex] widerspricht. Dieser Widerspruch zeigt, dass eine unendliche Teilmenge einen Häufungspunkt besitzen muss. ∎

Proposition 9.67: Lebesgue-Zahl

Sei [latex]X[/latex] ein folgenkompakter metrischer Raum. Dann hat jede offene Überdeckung eine Lebesgue-Zahl.

Beweis

Sei [latex]\mathcal {O}[/latex] eine offene Überdeckung von [latex]X[/latex], für welche wir eine Lebesgue-Zahl finden wollen. Dazu betrachten wir die Hilfsfunktion [latex]\eta : X \to \mathbb {R}_{>0}[/latex] gegeben durch

[latex]
\begin{aligned}[]\eta (x) = \sup \left \lbrace {\delta >0} \mid {\exists O \in \mathcal {O}:B_\delta (x) \subseteq O}\right \rbrace\end{aligned}
[/latex]

Wir behaupten, dass diese Lipschitz-stetig ist mit Lipschitz-Konstante [latex]1[/latex]. Seien also [latex]x_1,x_2 \in X[/latex] und sei [latex]\delta 0[/latex] ist, gibt es nach Wahl von [latex]\delta[/latex] ein [latex]O \in \mathcal {O}[/latex] mit [latex]B_\varepsilon (x_2) \subseteq B_\delta (x_1) \subseteq O[/latex]. Somit ist [latex]\eta (x_2) \geq \varepsilon = \delta - \mathrm {d}(x_1,x_2)[/latex], was auch für [latex]\varepsilon \leq 0[/latex] erfüllt ist. Nach Übergang zum Supremum ist [latex]\eta (x_2) \geq \eta (x_1) - \mathrm {d}(x_1,x_2)[/latex] oder in anderen Worten [latex]\mathrm {d}(x_1,x_2) \geq \eta (x_1)-\eta (x_2)[/latex]. Aus Symmetriegründen folgt daraus [latex]\mathrm {d}(x_1,x_2) \geq |\eta (x_1)-\eta (x_2)|[/latex] wie gewünscht.

Nach Annahme an [latex]X[/latex] (und der schon bewiesenen Äquivalenz der Eigenschaften (1)-(5) in Satz 9.66) existiert somit ein [latex]x_0 \in X[/latex] mit [latex]\eta (x_0) = \min \eta (X)[/latex]. Wir zeigen nun, dass [latex]r_{\text {Leb}} = \tfrac {1}{2} \eta (x_0)>0[/latex] eine Lebesgue-Zahl für [latex]\mathcal {O}[/latex] darstellt. In der Tat gilt für jedes [latex]x \in X[/latex] die Ungleichung [latex]r_{\text {Leb}}

Beweis der restlichen Äquivalenzen in Satz 9.66*

Wir beginnen mit dem Beweis von [latex](5) \implies (6)[/latex]. Nach obiger Proposition hat jede offene Überdeckung eine Lebesgue-Zahl. Es bleibt noch zu zeigen, dass [latex]X[/latex] total beschränkt ist. Dazu dürfen wir auf Grund der Äquivalenz von (5) und (2) verwenden, dass [latex]X[/latex] folgenkompakt ist. Sei [latex]\varepsilon > 0[/latex] und angenommen [latex]X[/latex] lässt sich nicht durch endlich viele Bälle vom Radius [latex]\varepsilon > 0[/latex] überdecken. Sei [latex]x_1 \in X[/latex] beliebig. Wir wählen rekursiv

[latex]
\begin{aligned}[]x_n \in X \setminus \left (B_\varepsilon (x_1) \cup \ldots \cup B_\varepsilon (x_{n-1}) \right )\end{aligned}
[/latex]

für alle [latex]n \geq 2[/latex] (was nach der indirekten Annahme jeweils möglich ist). Somit erhalten wir eine Folge [latex](x_n)_n[/latex] mit [latex]\mathrm {d}(x_m,x_n) \geq \varepsilon[/latex] für alle [latex]m,n \in \mathbb {N}[/latex] mit [latex]m \neq n[/latex]. Sei [latex](x_{n_k})_k[/latex] eine konvergente Teilfolge von [latex](x_n)_n[/latex]. Da aber jede konvergente Folge auch eine Cauchy-Folge ist, erhalten wir mit [latex]\mathrm {d}(x_{n_k},x_{n_\ell })\geq \varepsilon[/latex] für [latex]k\neq \ell[/latex] einen Widerspruch. Also lässt sich [latex]X[/latex] doch durch endlich viele Bälle vom Radius [latex]\varepsilon > 0[/latex] überdecken. Da [latex]\varepsilon > 0[/latex] beliebig war, ist [latex]X[/latex] total beschränkt.

Für die Implikation [latex](6) \implies (7)[/latex] sei [latex]\mathcal {O}[/latex] eine beliebige offene Überdeckung von [latex]X[/latex] und [latex]r>0[/latex] eine Lebesgue-Zahl für [latex]\mathcal {O}[/latex] nach [latex](6)[/latex]. Des Weiteren existieren nach totaler Beschränktheit [latex]x_1,\ldots ,x_n\in X[/latex] mit [latex]X = \bigcup _{k=1}^n B_r(x_k)[/latex]. Für jeden solchen Ball [latex]B_r(x_k)[/latex] existiert nach Definition der Lebesgue-Zahl ein [latex]O_k\in \mathcal {O}[/latex] mit [latex]B_r(x_k) \subseteq O_k[/latex]. Damit gilt also [latex]X = \bigcup _{k=1}^n O_k[/latex], womit [latex]\left \lbrace {O_1,\ldots ,O_n} \right \rbrace[/latex] eine endliche Teilüberdeckung von [latex]\mathcal {O}[/latex] darstellt.

Wir zeigen nun [latex](7) \implies (8)[/latex]. Sei also [latex]\mathcal {A}[/latex] eine Kollektion abgeschlossener Teilmengen mit [latex]\bigcap _{A\in \mathcal {A}} A = \emptyset[/latex]. Wir wollen zeigen, dass endlich viele [latex]A_1,\ldots ,A_N \in \mathcal {A}[/latex] existieren mit [latex]\bigcap _{n=1}^N A_n = \emptyset[/latex]. Nach Definition von [latex]\mathcal {A}[/latex] ist [latex]\mathcal {O} = \left \lbrace {X \setminus A} \mid {A \in \mathcal {A}}\right \rbrace[/latex] eine offene Überdeckung von [latex]X[/latex], womit nach (7) [latex]O_1 = X \setminus A_1[/latex], …, [latex]O_N = X \setminus A_N[/latex] in [latex]\mathcal {O}[/latex] existieren mit [latex]X = \bigcup _{n=1}^N O_n[/latex]. In anderen Worten ist [latex]\bigcap _{n=1}^N A_n = \emptyset[/latex] wie gewünscht.

Für die Implikationen [latex](8) \implies (9)[/latex] beweisen wir zuerst, dass [latex]X[/latex] total beschränkt ist. Sei [latex]\varepsilon > 0[/latex] beliebig und [latex]\mathcal {A} = \left \lbrace {X \setminus B_\varepsilon (x)} \mid {x \in X}\right \rbrace[/latex]. Dann ist [latex]\bigcap _{A\in \mathcal {A}} A = \emptyset[/latex], womit nach dem Schachtelungsprinzip [latex]x_1,\ldots ,x_n \in X[/latex] mit [latex]\bigcap _{k=1}^n (X \setminus B_\varepsilon (x_k)) = \emptyset[/latex] oder in anderen Worten mit [latex]\bigcup _{k=1}^n B_\varepsilon (x_k) = X[/latex] existieren müssen.

Um die Vollständigkeit zu verifizieren, betrachten wir eine Cauchy-Folge [latex](x_n)_n[/latex] und nehmen an, dass [latex](x_n)_n[/latex] nicht konvergiert. Da eine Cauchy-Folge genau dann konvergiert, wenn sie eine konvergente Teilfolge besitzt, ist für jedes [latex]n \in \mathbb {N}[/latex] die Teilmenge [latex]D_n = \left \lbrace {x_k} \mid {k \geq n}\right \rbrace[/latex] abgeschlossen. Weiter gilt für alle [latex]n \in \mathbb {N}[/latex]

[latex]
\begin{aligned}[]\bigcap _{k=1}^n D_k = D_n \neq \emptyset ,\end{aligned}
[/latex]

womit nach dem Schachtelungsprinzip [latex]\bigcap _{k\in \mathbb {N}} D_k \neq \emptyset[/latex], was einen Widerspruch darstellt.

Für die verbleibende Implikation [latex](9) \implies (1)[/latex] sei [latex]D \subseteq X[/latex] eine unendliche Teilmenge. Wir möchten eine Cauchy-Folge [latex](x_n)_n[/latex] in [latex]D[/latex] mit paarweise verschiedenen Folgenglieder konstruieren, welche dann nach [latex](9)[/latex] konvergieren muss. Der Grenzwert von [latex](x_n)_n[/latex] ist ein Häufungspunkt von [latex]D[/latex]. (Wieso?). Da [latex]X[/latex] total beschränkt ist, gibt es eine endliche Teilmenge [latex]F_1 \subseteq X[/latex] mit

[latex]
\begin{aligned}[]X = \bigcup _{y \in F_1} B_1(y).\end{aligned}
[/latex]

Sei [latex]y_1 \in X[/latex] mit [latex]|B_1(y_1) \cap D|= \infty[/latex] und sei [latex]D_1 = B_1(y_1) \cap D[/latex]. Im nächsten Schritt schreiben wir [latex]X = \bigcup _{y \in F_2} B_{\frac {1}{2}}(y)[/latex] für eine endliche Teilmenge [latex]F_2 \subseteq X[/latex], wählen ein [latex]y_2 \in F_2[/latex] mit [latex]|B_{\frac {1}{2}}(y_2) \cap D_1|= \infty[/latex] und setzen [latex]D_2 = B_{\frac {1}{2}}(y_2) \cap D_1[/latex]. Fährt man so fort, erhält man eine absteigende Folge

[latex]
\begin{aligned}[]D_1 \supseteq D_2 \supseteq D_3 \supseteq \ldots\end{aligned}
[/latex]

von Teilmengen von [latex]D[/latex] mit [latex]|D_k| = \infty[/latex] sowie [latex]D_k \subseteq B_{\frac {1}{k}}(y_k)[/latex] für alle [latex]k\in \mathbb {N}[/latex]. Für jedes [latex]n \in \mathbb {N}[/latex] sei nun [latex]x_n \in D_n[/latex] ein beliebiger Punkt. Die resultierende Folge [latex](x_n)_n[/latex] ist eine Cauchy-Folge, da für [latex]n \geq m[/latex] gilt [latex]x_n,x_m \in B_{\frac {1}{n}}(y_n)[/latex], womit [latex]\mathrm {d}(x_n,x_m)

Der Beweis der Überdeckungskompaktheit eines kompakten Intervalles in [latex]\mathbb {R}[/latex] ist nicht wesentlich einfacher als die allgemeine Äquivalenz zur Folgenkompaktheit in Satz 9.66. Trotzdem mag es hilfreich sein, diesen Fall getrennt zu untersuchen, da dadurch die Ähnlichkeiten zwischen totaler Beschränktheit und dem Intervallschachtelungsprinzip aufgezeigt werden können.

Übung 9.68

Sei [latex][a,b] \subseteq \mathbb {R}[/latex] ein nicht-leeres, kompaktes Intervall. Verwenden Sie das Intervallschachtelungsprinzip, um die Überdeckungskompaktheit von [latex][a,b][/latex] zu beweisen.

Hinweis.

Zerlegen Sie [latex][a,b][/latex] als [latex][a,b] = \left [a, \frac {a+b}{2}\right ] \cup \left [\frac {a+b}{2},b\right ][/latex] und betrachten Sie eines der beiden Intervalle abhängig von einer offenen Überdeckung von [latex][a,b][/latex], die keine endliche Teilüber­deckung besitzt.

Übung 9.69: Alternative Beweise

Beweisen Sie Lemma 9.60 unter Verwendung von Überdeckungskompaktheit.

9.5.3 – Endlich-dimensionale Vektorräume

Wie wir bei Lemma 9.60 schon gesehen haben, ist eine abgeschlossene und beschränkte Teilmenge eines metrischen Raumes nicht zwingend kompakt. Betrachtet man aber [latex]\mathbb {R}[/latex] oder allgemeiner [latex]\mathbb {R}^d[/latex] mit der Euklidschen Metrik, dann stimmt auch die Umkehrung. Wenn nichts anderes angeben ist, so statten wir [latex]\mathbb {R}^d[/latex] für [latex]d \geq 1[/latex] mit der Euklidschen Metrik aus.

Satz 9.70: Heine-Borel

Eine Teilmenge [latex]K \subseteq \mathbb {R}^d[/latex] für [latex]d \geq 1[/latex] ist genau dann kompakt, wenn sie abgeschlossen und beschränkt ist.

Beweis

Ist [latex]K[/latex] kompakt, dann ist [latex]K[/latex] nach Lemma 9.60 abgeschlossen und beschränkt.

Für die Umkehrung zeigen wir, dass [latex]K[/latex] folgenkompakt ist. Sei also [latex](x_n)_n[/latex] eine Folge in [latex]K[/latex]. Da [latex]K[/latex] beschränkt ist, ist die Folge beschränkt und besitzt nach Theorem 5.94 eine konvergente Teilfolge [latex](x_{n_k})_{k}[/latex]. Der Grenzwert [latex]x \in \mathbb {R}^d[/latex] dieser Teilfolge muss in [latex]K[/latex] liegen, da [latex]K[/latex] abgeschlossen ist (Lemma 9.19). ∎

Wie nützlich die Aussage von Satz 9.70 gemeinsam mit Satz 9.66(5) sein kann, illustriert beispielsweise die folgende Übung. In dieser möchten wir zeigen, dass je zwei Normen auf [latex]\mathbb {R}^d[/latex] äquivalent sind, wie schon in Abschnitt 5.7.2 versprochen wurde.

Wichtige Übung 9.71: Alle Normen auf [latex]\mathbb {R}^d[/latex] sind äquivalent

Sei [latex]d \geq 1[/latex] und seien [latex]\| {\cdot }\| ,\| {\cdot }\| '[/latex] zwei Normen auf [latex]\mathbb {R}^d[/latex]. Wir möchten in folgenden Schritten zeigen, dass [latex]\| {\cdot }\|[/latex] und [latex]\| {\cdot }\| '[/latex] äquivalent sind.

  1. Erklären Sie, wieso Sie annehmen können, dass [latex]\| {\cdot }\| ' = \| {\cdot }\| _1[/latex] die Einsnorm ist.
  2. Finden Sie eine Konstante [latex]C > 0[/latex] mit [latex]\| {{x}}\| \leq C \| {{x}}\| _1[/latex] für alle [latex]{x} \in \mathbb {R}^d[/latex]. Schliessen Sie daraus auch, dass [latex]\| {\cdot }\| : \mathbb {R}^d \to \mathbb {R}[/latex] eine bezüglich der euklidischen Metrik [latex]\mathrm {d}_2[/latex] stetige Abbildung ist.
  3. Sei [latex]A = \left \lbrace {{x} \in \mathbb {R}^d} \mid {\| {{x}}\| _1 = 1}\right \rbrace[/latex]. Zeigen Sie, dass die Einschränkung von [latex]\| {\cdot }\|[/latex] auf [latex]A[/latex] ein Minimum annimmt.
  4. Verwenden Sie (iii) um eine Konstante [latex]C' > 0[/latex] mit [latex]C'\| {{x}}\| _1 \leq \| {{x}}\|[/latex] für alle [latex]{x} \in \mathbb {R}^d[/latex] zu finden und schliessen Sie damit auf die zu beweisende Aussage.

Hinweis.

Zu (i): Normäquivalenz bildet eine Äquivalenzrelation — siehe Übung 5.90. Für (ii) stellen Sie [latex]{x} \in \mathbb {R}^d[/latex] in einer Basis dar und verwenden Sie die Dreiecksungleichung. Punkt (iii) folgt aus Stetigkeit von [latex]\| {\cdot }\|[/latex] und Kompaktheit von der Menge [latex]A[/latex].

9.5.4 – Bilder von kompakten Mengen

Eine weitreichende Verallgemeinerung der Existenz eines Maximums und eines Minimums stetiger, reellwertiger Funktionen auf kompakten Räumen (in Satz 9.66(5)) ist das folgende Resultat.

Satz 9.72: Kompaktes Bild

Seien [latex]X,Y[/latex] metrische Räume und sei [latex]f:X \to Y[/latex] eine stetige Abbildung. Falls [latex]X[/latex] kompakt ist, so ist auch [latex]f(X)[/latex] kompakt.

Beweis

Gegeben eine Folge [latex](y_n)_n[/latex] in [latex]f(X)[/latex] schreiben wir [latex]f(x_n) = y_n[/latex] für [latex]n \in \mathbb {N}[/latex] und [latex]x_n \in X[/latex]. Dann existiert eine konvergente Teilfolge [latex](x_{n_k})_k[/latex] von [latex](x_n)_n[/latex] mit Grenzwert [latex]x_0[/latex], da [latex]X[/latex] kompakt ist. Wegen Folgenstetigkeit von [latex]f[/latex] konvergiert also die Teilfolge [latex](f(x_{n_k}))_k = (y_{n_k})_k[/latex] gegen [latex]f(x_0)[/latex]. Die Folge [latex](y_n)_n[/latex] war aber beliebig. Somit ist [latex]f(X)[/latex] kompakt. ∎

Übung 9.73

Beweisen Sie Satz 9.72 jeweils mit den Charakterisierungen (5) und (7) von Kompaktheit aus Satz 9.66.

9.5.5 – Operatornorm von Matrizen

Als eine weitere Anwendung von Satz 9.66 möchten wir in diesem Abschnitt eine natürliche Norm auf dem Vektorraum [latex]\operatorname {Mat}_{m,n}(\mathbb {R})[/latex] der reellen [latex]m\times n[/latex]-Matrizen definieren. Wir bemerken zuerst, dass [latex]\operatorname {Mat}_{m,n}(\mathbb {R})[/latex] als reeller Vektorraum zu [latex]\mathbb {R}^{mn}[/latex] isomorph ist. Insbesondere kann man mit einer beliebigen Norm auf [latex]\mathbb {R}^{mn}[/latex] eine Norm auf [latex]\operatorname {Mat}_{m,n}(\mathbb {R})[/latex] induzieren (und umgekehrt). Alle solche Normen sind nach Übung 9.71 äquivalent, womit wir eine natürliche induzierte Topologie auf [latex]\operatorname {Mat}_{m,n}(\mathbb {R})[/latex] erhalten. Selbstverständlich kann man auch direkt auf [latex]\operatorname {Mat}_{m,n}(\mathbb {R})[/latex] eine Topologie definieren.

Übung 9.74: Hilbert-Schmidt-Norm

Zeigen Sie, dass [latex]\left \langle {A}, {B} \right \rangle = \operatorname {Tr}(AB^T)[/latex] ein Skalarprodukt auf [latex]\operatorname {Mat}_{n,n}(\mathbb {R})[/latex] definiert. Die induzierte Norm wird die Hilbert-Schmidt-Norm auf [latex]\operatorname {Mat}_{n,n}(\mathbb {R})[/latex]. Identifizieren Sie die dazugehörige Norm auf [latex]\mathbb {R}^{n^2}[/latex].

In unserem Kontext ist vor allem die folgende Norm auf [latex]\operatorname {Mat}_{m,n}(\mathbb {R})[/latex] nützlich.

Definition 9.75: Operatornorm

Die Operatornorm einer Matrix [latex]A \in \operatorname {Mat}_{m,n}(\mathbb {R})[/latex] ist durch

[latex]
\begin{aligned}[]\| {A}\| _{\mathrm {op}} = \sup _{x \in \mathbb {R}^n:\| {x}\| _2 \leq 1} \| {Ax}\| _2\end{aligned}
[/latex]

definiert.

Wir bemerken an dieser Stelle, dass sowohl auf [latex]\mathbb {R}^n[/latex] als auch [latex]\mathbb {R}^m[/latex] in obiger Definition eine andere Norm hätte verwendet werden können, womit man wiederum eine andere Operatornorm erhalten hätte. Allerdings werden wir nur die obige Operatornorm verwenden.

Lemma 9.76: Eigenschaften der Operatornorm

Seien [latex]m,n\in \mathbb {N}[/latex]. Dann definiert [latex]\| {\cdot }\| _\mathrm {op}[/latex] in der Tat eine (wohldefinierte) Norm auf [latex]\operatorname {Mat}_{m,n}(\mathbb {R})[/latex] und erfüllt
[latex]
\begin{aligned}[]\label{eq:metr-opnormeig1} \| {Ax}\| _2 \leq \| {A}\| _\mathrm {op} \| {x}\| _2\end{aligned}
[/latex]
für alle [latex]x \in \mathbb {R}^n[/latex] und alle [latex]A \in \operatorname {Mat}_{m,n}(\mathbb {R})[/latex]. Des Weiteren gilt für [latex]k\in \mathbb {N}[/latex], [latex]A \in \operatorname {Mat}_{m,n}(\mathbb {R})[/latex] und [latex]B \in \operatorname {Mat}_{n,k}(\mathbb {R})[/latex]
[latex]
\begin{aligned}[]\label{eq:metr-opnormeig2} \| {AB}\| _\mathrm {op} \leq \| {A}\| _\mathrm {op} \| {B}\| _\mathrm {op}.\end{aligned}
[/latex]

Beweis

Nach dem Satz von Heine Borel (Satz 9.70) ist [latex]\overline {B_1(0)} = \left \lbrace {x \in \mathbb {R}^n} \mid {\| {x}\| _2 \leq 1}\right \rbrace[/latex] kompakt. Da die Abbildung [latex]x \in \overline {B_1(0)} \mapsto \| {Ax}\| _2[/latex] (als Verknüpfung von stetigen Funktionen) stetig ist, ist sie auf Grund von Satz 9.66(4) beschränkt, womit [latex]\| {A}\| _{\mathrm {op}}

Für die Definitheit sei [latex]A \in \operatorname {Mat}_{m,n}(\mathbb {R})[/latex] mit [latex]\| {A}\| _\mathrm {op} = 0[/latex]. Dann ist [latex]\| {Ae_i}\| _2 \leq \| {A}\| _\mathrm {op} =0[/latex] und somit [latex]Ae_i = 0[/latex] für alle [latex]i \in \left \lbrace {1,\ldots ,n} \right \rbrace[/latex]. Also ist [latex]A = 0[/latex]. Für [latex]\lambda \in \mathbb {R}[/latex] und [latex]A \in \operatorname {Mat}_{m,n}(\mathbb {R})[/latex] ist

[latex]
\begin{aligned}[]\| {\lambda A}\| _\mathrm {op} &= \sup _{x \in \mathbb {R}^n:\| {x}\| _2 \leq 1} \| { \lambda A x}\| _2 = \sup _{x \in \mathbb {R}^n:\| {x}\| _2 \leq 1} |\lambda |\| { A x}\| _2 \\ &= |\lambda | \sup _{x \in \mathbb {R}^n:\| {x}\| _2 \leq 1} \| { A x}\| _2 = |\lambda | \| {A}\| _\mathrm {op}.\end{aligned}
[/latex]

Für [latex]A,B \in \operatorname {Mat}_{m,n}(\mathbb {R})[/latex] und [latex]x \in \overline {B_1(0)}[/latex] gilt

[latex]
\begin{aligned}[]\| {(A+B)x}\| _2 = \| {Ax+Bx}\| _2 \leq \| {Ax}\| _2 + \| {Bx}\| _2 \leq \| {A}\| _\mathrm {op} + \| {B}\| _\mathrm {op}\end{aligned}
[/latex]

und somit nach Übergang zum Supremum [latex]\| {A+B}\| _\mathrm {op} \leq \| {A}\| _\mathrm {op} + \| {B}\| _\mathrm {op}[/latex].

Für die Ungleichung (9.2) stellt man zuerst fest, dass im Falle [latex]x=0[/latex] nichts zu beweisen ist. Ist [latex]x \neq 0[/latex], so gilt

[latex]
\begin{aligned}[]\| {Ax}\| _2 = \big \| A \tfrac {x}{\| {x}\| _2} \big \| \| {x}\| _2 \leq \| {A}\| _\mathrm {op} \| {x}\| _2.\end{aligned}
[/latex]

Für die Ungleichung (9.3) berechnet man zu [latex]x \in \overline {B_1(0)}[/latex] mit (9.2)

[latex]
\begin{aligned}[]\| {ABx}\| _2 \leq \| {A}\| _\mathrm {op} \| {Bx}\| _2 \leq \| {A}\| _\mathrm {op} \| {B}\| _\mathrm {op},\end{aligned}
[/latex]

womit die Behauptung folgt. ∎

Analog lässt sich auch eine Topologie sowie eine Operatornorm auf [latex]\operatorname {Mat}_{m,n}(\mathbb {C})[/latex] durch

[latex]
\begin{aligned}[]\| {A}\| _\mathrm {op} = \sup _{x \in \mathbb {C}^n:\| {x}\| _2 \leq 1} \| {Ax}\| _2\end{aligned}
[/latex]

für [latex]A \in \operatorname {Mat}_{m,n}(\mathbb {C})[/latex] definieren.

9.5.6 – Gleichmässige Stetigkeit und Oszillation

Stetige Abbildungen auf kompakten Mengen haben die (wichtige) Eigenschaft der gleichmässigen Stetigkeit.

Proposition 9.77

Seien [latex]X,Y[/latex] zwei metrische Räume und [latex]f:X \to Y[/latex] eine stetige Funktion. Falls [latex]X[/latex] kompakt ist, so ist [latex]f[/latex] gleichmässig stetig.

Dies ist eine Verallgemeinerung des Satzes von Heine (Satz 3.78), welcher den Spezialfall [latex]X = [a,b] \subseteq \mathbb {R}[/latex] für [latex]a

Definition 9.78

Sei [latex]f: X \to \mathbb {R}[/latex] eine beschränkte, reellwertige Funktion auf einem metrischen Raum [latex]X[/latex]. Für [latex]x \in X[/latex] ist die Oszillation oder Schwankung von [latex]f[/latex] bei [latex]x[/latex] durch

[latex]
\begin{aligned}[]\omega (f,x) = \lim _{\delta \searrow 0} \omega (f,x,\delta )\end{aligned}
[/latex]

definiert, wobei für [latex]\delta > 0[/latex]

[latex]
\begin{aligned}[]\omega (f,x,\delta ) = \sup f(B_{\delta }(x)) - \inf f(B_{\delta }(x)).\end{aligned}
[/latex]

Wir bemerken dazu, dass für [latex]f[/latex] wie oben und für [latex]0

[latex]
\begin{aligned}[]\sup f(B_{\delta _1}(x)) \leq \sup f(B_{\delta _2}(x)),\quad \inf f(B_{\delta _1}(x)) \geq \inf f(B_{\delta _2}(x)),\end{aligned}
[/latex]

und damit auch [latex]\omega (f,x,\delta _1) \leq \omega (f,x,\delta _2)[/latex] impliziert. Insbesondere zeigt dies, dass der Grenzwert für [latex]\delta \searrow 0[/latex] in Definition 9.78 tatsächlich existiert. Des Weiteren bemerken wir, dass [latex]f[/latex] genau dann bei [latex]x \in X[/latex] stetig ist, wenn [latex]\omega (f,x) = 0[/latex] ist. (Wieso?).

Proposition 9.79: Gleichmässig kleine Oszillation

Sei [latex]X[/latex] eine kompakter metrischer Raum und sei [latex]f:X \to \mathbb {R}[/latex] eine beschränkte Funktion. Angenommen es gibt [latex]\eta \geq 0[/latex], so dass [latex]\omega (f,x) \leq \eta[/latex] für alle [latex]x \in X[/latex]. Dann existiert für jedes [latex]\varepsilon > 0[/latex] ein [latex]\delta > 0[/latex], so dass für alle [latex]x \in X[/latex]

[latex]
\begin{aligned}[]\omega (f,x,\delta ) [/latex]

gilt.

Wir bemerken, dass der Fall [latex]\eta = 0[/latex] gerade der Aussage, dass eine stetige Funktion auf einer kompakten Menge gleichmässig stetig ist, entspricht (Proposition 9.77).

Beweis

Angenommen die Aussage der Proposition trifft nicht zu. Sei also [latex]\varepsilon > 0[/latex], so dass für alle [latex]\delta > 0[/latex] ein [latex]x \in X[/latex] mit [latex]\omega (f,x,\delta ) \geq \eta + \varepsilon[/latex] existiert. Zu [latex]m \in \mathbb {N}[/latex] und [latex]\delta _m = \frac {1}{m}[/latex] können wir also [latex]x_m \in X[/latex] mit [latex]\omega (f,x_m,\frac {1}{m}) \geq \eta + \varepsilon[/latex] wählen. Nach Definition von [latex]\omega (f,x_m,\frac {1}{m})[/latex] zeigt dies, dass es auch [latex]x_m^+,x_m^- \in B_{\frac {1}{m}}(x_m)[/latex] mit
[latex]
\begin{aligned}[]\label{eq:metr-proofunifoscillation} f(x_m^+)-f(x_m^-) > \eta + \frac {\varepsilon }{2}\end{aligned}
[/latex]
gibt. Da [latex]X[/latex] als kompakt vorausgesetzt wurde, existiert eine konvergente Teilfolge [latex](x_{m_\ell })[/latex] in [latex]X[/latex]. Sei

[latex]
\begin{aligned}[]x = \lim _{\ell \to \infty } x_{m_\ell } = \lim _{\ell \to \infty } x_{m_\ell }^+ = \lim _{\ell \to \infty } x_{m_\ell }^-\in X\end{aligned}
[/latex]

deren Grenzwert.

Zu jedem [latex]\delta > 0[/latex] gibt es insbesondere ein [latex]\ell[/latex] mit [latex]x_{m_\ell }^+,x_{m_\ell }^- \in B_{\delta }(x)[/latex]. Die Ungleichung (9.4) impliziert [latex]\omega (f,x,\delta ) > \eta +\frac {\varepsilon }{2}[/latex]. Da [latex]\delta > 0[/latex] beliebig war, erhalten wir [latex]\omega (f,x)\geq \eta +\frac {\varepsilon }{2}[/latex], was unserer Voraussetzung widerspricht. ∎

Übung 9.80

Beweisen Sie Proposition 9.79 unter Verwendung einer offenen Überdeckung und der Existenz einer Lebesgue-Zahl in Proposition 9.67.

9.6 – Fundamentalsatz der Algebra

Theorem 9.81: Fundamentalsatz der Algebra

Jedes komplexe Polynom [latex]f \in \mathbb {C}[z][/latex] mit positivem Grad [latex]n = \deg (f) \geq 1[/latex] hat mindestens eine Nullstelle in [latex]\mathbb {C}[/latex]. Des Weiteren lässt sich [latex]f[/latex] als Produkt eines Skalars [latex]a_n \in \mathbb {C}^\times[/latex] und [latex]n[/latex] Linearfaktoren der Form [latex](z-\alpha _j)[/latex] schreiben, das heisst

[latex]
\begin{aligned}[]f = a_n \prod _{j=1}^n (z-\alpha _j)\end{aligned}
[/latex]

mit [latex]\alpha _1,\ldots ,\alpha _n \in \mathbb {C}[/latex].

Es genügt dabei, den ersten Teil des Theorems zu beweisen. Der zweite Teil folgt dann aus dem ersten, Division mit Rest (siehe Abschnitt 3.2) und Induktion. Der Fundamentalsatz der Algebra hat viele verschiedene Beweise; einige davon beruhen auf Funktionentheorie (siehe zweites Studienjahr) oder algebraischer Topologie (siehe drittes oder viertes Jahr des Mathematikstudiums). Wir werden hier das asymptotische Verhalten von Polynomen (Proposition 3.15), den Satz von Heine-Borel (Satz 9.70) und die Polarzerlegung komplexer Zahlen (Lemma 6.81) verwenden.

Beweis

Sei [latex]f \in \mathbb {C}[z][/latex] ein komplexes Polynom mit Grad [latex]n = \deg (f) \geq 1[/latex]. Falls [latex]f(0) = 0[/latex] ist, so haben wir bereits eine Nullstelle gefunden. Also angenommen [latex]M = 2 |f(0)| > 0[/latex]. Nach Proposition 3.15 gibt es daher ein [latex]R \geq 1[/latex], so dass für alle [latex]z \in \mathbb {C}[/latex] mit [latex]|z| \geq R[/latex] auch [latex]|f(z)| \geq M[/latex] gilt. Nach dem Satz von Heine-Borel (Satz 9.70) ist

[latex]
\begin{aligned}[]K = \left \lbrace {z \in \mathbb {C}} \mid {|z| \leq R}\right \rbrace\end{aligned}
[/latex]

eine kompakte Teilmenge. Wir wenden nun die Existenz des Minimalwertes in Satz 9.66(5) auf die stetige Funktion [latex]z \in K \mapsto |f(z)| \in \mathbb {R}[/latex] an und finden ein [latex]z_0 \in K[/latex] mit [latex]|f(z_0)| = \min _{z \in K} |f(z)|[/latex].

Da [latex]|f(0)| = \frac {M}{2}[/latex], gilt die Ungleichung [latex]|f(z_0)| 0[/latex] ist.

Wir schreiben [latex]f[/latex] als Potenzreihe um [latex]z_0[/latex] (für das Polynom folgt die Existenz dieser Darstellung aber auch einfach mittels Division mit Rest und Induktion nach [latex]n=\deg (f)[/latex])

[latex]
\begin{aligned}[]f(z) = \sum _{k=0}^n b_k (z-z_0)^k\end{aligned}
[/latex]

mit [latex]b_0 = f(z_0)[/latex], [latex]b_1 = f'(z_0)[/latex], [latex]\ldots[/latex] , [latex]b_n = \frac {f^{(n)}(z_0)}{n!}\in \mathbb {C}[/latex]. Per Annahme an [latex]z_0[/latex] gilt [latex]b_0 \neq 0[/latex], was wir nun auf einen Widerspruch führen wollen, indem wir einen Punkt [latex]z[/latex] in der Nähe von [latex]z_0[/latex] mit [latex]|f(z)|

[latex]
\begin{aligned}[]f(z) = b_0 + b_\ell (z-z_0)^\ell + \sum _{k=\ell +1}^n b_k (z-z_0)^k = b_0 + b_\ell (z-z_0)^\ell + O(|z-z_0|^{\ell +1})\end{aligned}
[/latex]

für [latex]z \to z_0[/latex]. Wir setzen [latex]z = z_0 + r\mathrm {e}^{\mathrm {i} \varphi }[/latex] für ein festes [latex]\varphi \in \mathbb {R}[/latex] (welches wir später wählen wollen) und ein variierendes [latex]r> 0[/latex], womit

[latex]
\begin{aligned}[]f(z_0 + r \mathrm {e}^{\mathrm {i} \varphi }) = b_0 + b_\ell r^\ell \mathrm {e}^{\mathrm {i} \ell \varphi } + O(r^{\ell +1}) = b_0 \left (1 + \tfrac {b_\ell }{b_0} r^\ell \mathrm {e}^{\mathrm {i} \ell \varphi }\right ) + O(r^{\ell +1})\end{aligned}
[/latex]

für [latex]r \searrow 0[/latex]. Wir schreiben [latex]\tfrac {b_\ell }{b_0} = s \mathrm {e}^{\mathrm {i} \psi }[/latex] für ein [latex]s > 0[/latex] und [latex]\psi \in [0,2\pi )[/latex] und erhalten

[latex]
\begin{aligned}[]f(z_0 + r \mathrm {e}^{\mathrm {i} \varphi }) = b_0 \left (1 + s \mathrm {e}^{\mathrm {i} \psi } r^\ell \mathrm {e}^{\mathrm {i} \ell \varphi } \right ) + O(r^{\ell +1}) = b_0 \left (1 + s r^\ell \mathrm {e}^{\mathrm {i} (\ell \varphi + \psi )} \right ) + O(r^{\ell +1})\end{aligned}
[/latex]

für [latex]r \searrow 0[/latex]. Wir möchten nun den obigen Term bei [latex]r^\ell[/latex] so arrangieren, dass er negativ und reell ist. Dafür setzt man [latex]\varphi = \frac {-\psi +\pi }{\ell }[/latex], so ist der obige Winkel [latex]\ell \varphi + \psi[/latex] gleich [latex]\pi[/latex] und [latex]\mathrm {e}^{\mathrm {i} (\ell \varphi + \psi )} = -1[/latex]. Zusammenfassend gilt

[latex]
\begin{aligned}[]|f(z_0 + r \mathrm {e}^{\mathrm {i} \varphi })| &= \left |b_0 \left (1 - s r^\ell \right )+O(r^{\ell +1})\right | \leq |b_0| \left (1 - s r^\ell \right ) +O(r^{\ell +1})\end{aligned}
[/latex]

für [latex]r \to 0[/latex]. Für genügend kleine [latex]r>0[/latex] ist diese obere Schranke aber kleiner als [latex]|b_0|[/latex], was einen Widerspruch zu [latex]|f(z_0)| = |b_0| = \min _{z \in K} |f(z)|[/latex] darstellt. Dies beweist, dass [latex]f(z_0)=0[/latex] gelten muss. ∎

Auch für reelle Polynome kann ein Fundamentalsatz formuliert werden. Wir bemerken hierfür, dass für jedes [latex]f \in \mathbb {R}[x][/latex] mit einer Nullstelle [latex]\alpha \in \mathbb {C}[/latex] auch [latex]\overline {\alpha }[/latex] eine Nullstelle ist (wieso?). In der Zerlegung von [latex]f[/latex] in Linearfaktoren in Theorem 9.81 sieht man also zu jeder echt komplexen Nullstelle [latex]\alpha \in \mathbb {C} \setminus \mathbb {R}[/latex] den Faktor

[latex]
\begin{aligned}[](x-\alpha ) (x-\overline {\alpha }) = x^2 - (\alpha + \overline {\alpha }) x + |\alpha |^2 = x^2 - 2 \operatorname {Re}(\alpha ) x + |\alpha |^2\end{aligned}
[/latex]

stehen. Wendet man diese Einsicht auf jede echt komplexe Nullstelle von [latex]f[/latex] an, so erhält man, dass [latex]f[/latex] als Produkt von reellen Polynomen vom Grad [latex]1[/latex] und vom Grad [latex]2[/latex] geschrieben werden kann. Wir haben diese Einsichten implizit bereits bei der Besprechung der Integration von rationalen Funktionen verwendet.

Übung 9.82: Endlich-dimensionale Körpererweiterungen von [latex]\mathbb {C}[/latex]

Sei [latex]\mathbb {K} \supseteq \mathbb {C}[/latex] ein Körper, für den die Körperoperationen auf [latex]\mathbb {C}[/latex] mit den üblichen Körperoperationen von [latex]\mathbb {C}[/latex] selbst übereinstimmen (eine Körpererweiterung von [latex]\mathbb {C}[/latex]). Dann lässt sich [latex]\mathbb {K}[/latex] auf natürliche Weise als Vektorraum über [latex]\mathbb {C}[/latex] auffassen. Zeigen Sie, dass der Körper [latex]\mathbb {K}[/latex] automatisch gleich [latex]\mathbb {C}[/latex] selbst ist, falls die Dimension von [latex]\mathbb {K}[/latex] über [latex]\mathbb {C}[/latex] endlich ist.

Hinweis.

Betrachten Sie zu [latex]z \in \mathbb {K}[/latex] Multiplikation mit [latex]z[/latex] auf [latex]\mathbb {K}[/latex].

Übung 9.83: Minimumsprinzip

Sei [latex]f[/latex] eine komplexwertige Funktion auf einer offenen Teilmenge von [latex]\mathbb {C}[/latex], die sich lokal durch Potenzreihen darstellen lässt (eine analytische Funktion). Genauer soll für jedes [latex]x_0 \in U[/latex] ein [latex]R>0[/latex] mit [latex]B_R(x_0) \subseteq U[/latex] existieren, so dass [latex]f[/latex] auf [latex]B_R(x_0)[/latex] gleich einer Potenzreihe um [latex]x_0[/latex] mit Konvergenzradius grösser gleich [latex]R[/latex] ist. Zeigen Sie, dass [latex]f[/latex] kein Minimum annimmt.

Hinweis.

Imitieren Sie den Beweis von Theorem 9.81.

9.7 – Der Raum der stetigen Funktionen

Wie wir bereits erwähnt haben, werden wir den Banachschen Fixpunktsatz unter anderem in der Diskussion von Differentialgleichungen verwenden. Wir wollen hier noch den dafür geeigneten vollständigen metrischen Raum besprechen.

Lemma 9.84: Vektorraum der stetigen Funktionen

Sei [latex]X[/latex] ein metrischer Raum. Dann ist für jedes [latex]d \geq 1[/latex] die Menge

[latex]
\begin{aligned}[]C(X,\mathbb {R}^d) = \left \lbrace {f:X \to \mathbb {R}^d} \mid {f \text { ist stetig}}\right \rbrace\end{aligned}
[/latex]

mit den (punktweisen) Verknüpfungen

[latex]
\begin{aligned}[]f + g: x\in X \mapsto f(x)+g(x),\\ \lambda f: x \in X\mapsto \lambda f(x)\end{aligned}
[/latex]

für [latex]f,g[/latex] in [latex]C(X,\mathbb {R}^d)[/latex] und [latex]\lambda \in \mathbb {R}[/latex] ein Vektorraum.

Übung 9.85

Beweisen Sie Lemma 9.84.

Ist der metrische Raum [latex]X[/latex] in obigem Lemma kompakt, so lässt sich der Vektorraum [latex]C(X,\mathbb {R}^d)[/latex] mit einer Norm ausstatten.

Proposition 9.86: Vollständigkeit des Raumes der stetigen Funktionen

Sei [latex]K[/latex] ein kompakter metrischer Raum. Dann definiert

[latex]
\begin{aligned}[]\| {f}\| _\infty = \sup _{x \in K} \| {f(x)}\| _2=\max _{x \in K} \| {f(x)}\| _2\end{aligned}
[/latex]

eine Norm auf [latex]C(K,\mathbb {R}^d)[/latex], die sogenannte Supremumsnorm, und [latex]C(K,\mathbb {R}^d)[/latex] ist bezüglich dieser Norm vollständig. Des Weiteren konvergiert [latex]f_n\in C(K)[/latex] bezüglich der Norm [latex]\| {\cdot }\| _\infty[/latex] gegen [latex]f\in C(K)[/latex] genau dann, wenn [latex]f_n[/latex] gleichmässig gegen [latex]f[/latex] konvergiert, das heisst wenn es zu jedem [latex]\varepsilon >0[/latex] ein [latex]N\in \mathbb {N}[/latex] gibt so dass für alle [latex]n\in \mathbb {N}[/latex] mit [latex]n\geq N[/latex] und alle [latex]x\in X[/latex] die Abschätzung [latex]\| {f_n(x)-f(x)}\| _2

In Beispiel 9.11 wurde für den Fall, wo [latex]K \subseteq \mathbb {R}[/latex] ein kompaktes Intervall ist und [latex]d=1[/latex] ist, schon gezeigt, dass [latex]\| {\cdot }\| _\infty[/latex] eine Norm definiert. Der allgemeine Fall ist analog. Wir müssen also noch zeigen, dass [latex]C(K,\mathbb {R}^d)[/latex] mit der Supremumsnorm vollständig ist. Auf Grund des Zusammenhangs mit gleichmässiger Konvergenz ist dies aber im Wesentlichen eine Wiederholung von Satz 6.48.

Beweis

Sei [latex](f_n)_n[/latex] eine Cauchy-Folge in [latex]C(K,\mathbb {R}^d)[/latex]. Für [latex]x \in K[/latex] ist dann wegen

[latex]
\begin{aligned}[]\| {f_n(x)-f_m(x)}\| _2 \leq \| {f_m-f_n}\| _\infty\end{aligned}
[/latex]

für alle [latex]m,n\in \mathbb {N}[/latex] die Folge [latex](f_n(x))_n[/latex] eine Cauchy-Folge in [latex]\mathbb {R}^d[/latex]. Da [latex]\mathbb {R}[/latex] vollständig ist (Satz 5.48) existiert also der Grenzwert dieser Folge, welchen wir als [latex]f(x) = \lim _{n \to \infty }f_n(x)[/latex] bezeichnen. Da [latex]x \in K[/latex] beliebig war (und Grenzwerte eindeutig bestimmt sind), definiert dies also eine Funktion [latex]f: K \to \mathbb {R}^d[/latex].

Wir behaupten nun, dass [latex]f_n[/latex] gleichmässig gegen [latex]f[/latex] strebt. Sei also [latex]\varepsilon > 0[/latex]. Dann existiert ein [latex]N \in \mathbb {N}[/latex], so dass [latex]\| {f_m(x)-f_n(x)}\| _2 \leq \| {f_m-f_n}\| _\infty

Zuletzt zeigen wir noch, dass [latex]f \in C(K,\mathbb {R}^d)[/latex] ist. Sei also [latex]\varepsilon > 0[/latex] und [latex]N \in \mathbb {N}[/latex] mit der Eigenschaft [latex]\| {f_N - f}\| _\infty \leq \varepsilon[/latex]. Für jedes [latex]x_0 \in K[/latex] gibt es auf Grund der Stetigkeit von [latex]f_N[/latex] bei [latex]x_0[/latex] ein [latex]\delta > 0[/latex], so dass [latex]\| {f_N(x)-f_N(x_0)}\| _2 \leq \varepsilon[/latex] für alle [latex]x \in K[/latex] mit [latex]\mathrm {d}(x,x_0)

[latex]
\begin{aligned}[]\| {f(x)-f(x_0)}\| _2 \leq \| {f(x)-f_N(x)}\| _2 + \| {f_N(x)-f_N(x_0)}\| _2 + \| {f_N(x_0)-f(x_0)}\| _2 \leq 3 \varepsilon\end{aligned}
[/latex]

Da [latex]\varepsilon > 0[/latex] beliebig war, beweist dies die Stetigkeit von [latex]f[/latex] bei [latex]x_0 \in K[/latex].

Schlussendlich zeigen wir noch, dass [latex]f_n[/latex] für [latex]n\to \infty[/latex] genau dann bezüglich Norm [latex]\| {\cdot }\| _\infty[/latex] gegen [latex]f[/latex] strebt, wenn [latex]f_n[/latex] für [latex]n\to \infty[/latex] gleichmässig gegen [latex]f[/latex] strebt. In der Tat ist die Bedingung [latex]\| {f_n - f}\| _\infty \leq \varepsilon[/latex] zu [latex]\forall x\in X:|f_n(x)-f(x)|\leq \varepsilon[/latex] äquivalent. Setzt man dies in die Definitionen der beiden Konvergenzen ein, so erhält man die behauptete Äquivalenz. ∎

9.8 – Konstruktion der reellen Zahlen*

Wir haben bis jetzt all unsere Diskussionen auf den Axiomen der reellen Zahlen (siehe Abschnitt 2.1) aufgebaut. Unter anderem wurden aus diesen auch die natürlichen, die ganzen und die rationalen Zahlen konstruiert. Die etwas abstrakteren Begriffe dieses Kapitels lassen sich jedoch auch dazu verwenden, eine Konstruktion der reellen Zahlen aus den natürlichen Zahlen anzugeben. Wir wollen dies hier andeuten, ohne aber auf alle Details einzugehen.

Satz 9.87: Existenz der reellen Zahlen

Man kann ein Modell [latex](\mathbb {R},+,\cdot ,\leq )[/latex] der Axiome der reellen Zahlen (in Abschnitt 2.1) unter Verwendung der natürlichen Zahlen (wie in Abschnitt 1.5) definieren.

Beweis

Wie in Abschnitt 1.5.1 besprochen wurde, kann man aus [latex]\mathbb {N}[/latex] zuerst [latex]\mathbb {Z}[/latex] und dann auch den Körper [latex]\mathbb {Q}[/latex] mittels rein algebraischer Überlegungen und geeigneten Äquivalenzrelationen konstruieren. Mit der Ordnung auf den natürlichen Zahlen (definiert für [latex]m,n\in \mathbb {N}[/latex] durch [latex]m\leq n\Leftrightarrow \exists k\in \mathbb {N}_0:m+k=n[/latex]) lässt sich auch eine Ordnung auf [latex]\mathbb {Q}[/latex] definieren, die alle gewünschten Eigenschaften erfüllt. Wir nehmen nun im Folgenden also den Körper [latex]\mathbb {Q}[/latex] und die Relation [latex]\leq _\mathbb {Q}[/latex] als gegeben an und werden die bekannten Eigenschaften (die Axiome (2.2)-() aus Abschnitt 2.1 für [latex](\mathbb {Q},+,\cdot )[/latex] und ihre Konsequenzen) ohne Verweis verwenden.

Definition der reellen Zahlen: Um [latex]\mathbb {R}[/latex] aus [latex]\mathbb {Q}[/latex] zu konstruieren, definieren wir zuerst

[latex]
\begin{aligned}[]V = \left \lbrace {(x_n)_n \in \mathbb {Q}^\mathbb {N}} \mid {(x_n)_n \text { ist eine Cauchy-Folge in }\mathbb {Q}}\right \rbrace ,\end{aligned}
[/latex]

wobei [latex](x_n)_n \in \mathbb {Q}^\mathbb {N}[/latex] eine Cauchy-Folge in [latex]\mathbb {Q}[/latex] ist, falls es für jedes [latex]\varepsilon \in \mathbb {Q}[/latex] mit [latex]\varepsilon >_\mathbb {Q} 0[/latex] ein [latex]N \in \mathbb {N}[/latex] gibt mit [latex]|x_m-x_n| <_ f alle n die menge ist bez der komponentenweisen operationen ein vektorraum und>

[latex]
\begin{aligned}[]V_0 = \left \lbrace {(z_n)_n \in \mathbb {Q}^\mathbb {N}} \mid {\lim _{n\to \infty } z_n = 0}\right \rbrace\end{aligned}
[/latex]

der Nullfolgen in [latex]\mathbb {Q}[/latex] ist ein Teilraum. Wir definieren die Menge der reellen Zahlen

[latex]
\begin{aligned}[]\mathbb {R} = \mathchoice {\text {\raise 0.4ex\hbox {${V}$}\big /\lower 0.4ex\hbox {${V_0}$}}} {{V}/\text {\hspace {-0.3ex}\vspace {-0.5ex}${V_0}$} } {{V}\, /{V_0} } {{V}\, /{V_0} } = \left \lbrace {\vphantom {\mathbb {Q}^\mathbb {N}}[(x_n)_n]} \mid {(x_n)_n \in V}\right \rbrace\end{aligned}
[/latex]

im Sinne der Linearen Algebra, wobei

[latex]
\begin{aligned}[][(x_n)_n] = (x_n)_n + V_0 = \left \lbrace {(x_n+z_n)_n} \mid {(z_n)_n \in \mathbb {Q}^\mathbb {N} \text { mit } \lim _{n \to \infty }z_n = 0}\right \rbrace\end{aligned}
[/latex]

die Nebenklasse von [latex](x_n)_n \in V[/latex] bezeichnet.

Operationen auf den reellen Zahlen: Da [latex]V_0[/latex] ein Teilraum von [latex]V[/latex] ist, ist [latex]\mathbb {R} = \mathchoice {\text {\raise 0.4ex\hbox {${V}$}\big /\lower 0.4ex\hbox {${V_0}$}}} {{V}/\text {\hspace {-0.3ex}\vspace {-0.5ex}${V_0}$} } {{V}\, /{V_0} } {{V}\, /{V_0} }[/latex] ein Vektorraum über [latex]\mathbb {Q}[/latex]. Insbesondere erfüllt [latex]\mathbb {R}[/latex] bezüglich der induzierten Addition die Axiome (2.2)-() der Addition.

Um die Multiplikation auf [latex]\mathbb {R}[/latex] zu definieren, setzen wir analog zur Addition zuerst

[latex]
\begin{aligned}[](x_n)_n \cdot (y_n)_n = (x_ny_n)_n\end{aligned}
[/latex]

für alle [latex](x_n)_n, (y_n)_n \in V[/latex]. Hierbei ist [latex](x_ny_n)_n \in V[/latex] in der Tat wieder eine Cauchy-Folge (überprüfen Sie dies). Des Weiteren gilt für eine Cauchy-Folge [latex](x_n)_n \in V[/latex] und eine Nullfolge [latex](z_n)_n \in V_0[/latex], dass [latex]|x_n|[/latex] beschränkt ist (unabhängig von [latex]n[/latex]) und somit [latex]\lim _{n \to \infty } x_nz_n = 0[/latex] oder äquivalent [latex](x_n)_n \cdot (z_n)_n \in V_0[/latex] gilt. Wir definieren nun eine Multiplikation [latex]\cdot : \mathbb {R} \times \mathbb {R} \to \mathbb {R}[/latex] durch

[latex]
\begin{aligned}[][(x_n)_n]\cdot [(y_n)_n] = [(x_ny_n)_n]\end{aligned}
[/latex]

für [latex][(x_n)_n],[(y_n)_n] \in \mathbb {R}[/latex]. Diese ist in der Tat wohldefiniert, denn es gilt für [latex](z_n)_n,(z_n')_n \in V_0[/latex]

[latex]
\begin{aligned}[](x_n+z_n)_n \cdot (y_n+z_n')_n = (x_ny_n + \underset { \text {eine Nullfolge}}{\underbrace {z_ny_n+x_nz_n'+z_nz_n'}})_n \in (x_ny_n)_n + V_0.\end{aligned}
[/latex]

Da [latex]\mathbb {Q}[/latex] ein Körper ist, kann man nun die weiteren Körperaxiome von [latex](\mathbb {R},+,\cdot )[/latex] überprüfen. Unter anderem zeigt man, dass [latex][(1)_n][/latex] das multiplikative Einselement ist. Wir wollen hier bloss die Existenz einer multiplikativen Inversen (Axiom ()) genauer besprechen.

Existenz einer multiplikativen Inversen: Sei [latex][(x_n)_n] \in \mathbb {R} \setminus \left \lbrace {0} \right \rbrace[/latex]. Dann ist [latex](x_n)_n \in V[/latex] eine Cauchy-Folge in [latex]\mathbb {Q}[/latex], die nicht gegen Null strebt. Insbesondere gibt es ein rationales [latex]\delta >_\mathbb {Q} 0[/latex], so dass [latex]|x_n| \geq _\mathbb {Q} 2 \delta[/latex] für unendlich viele [latex]n \in \mathbb {N}[/latex]. Da [latex](x_n)_n[/latex] eine Cauchy-Folge ist, gibt es auch ein [latex]N \in \mathbb {N}[/latex], so dass [latex]|x_m-x_n|<_ f alle n wir w ein mit _ und erhalten>

[latex]
\begin{aligned}[]|x_m| \geq _\mathbb {Q} |x_n| - |x_m-x_n| \geq _\mathbb {Q} \delta\end{aligned}
[/latex]

für alle [latex]m \geq N[/latex]. Damit definieren wir nun eine Folge [latex](y_m)_m[/latex] durch

[latex]
\begin{aligned}[]y_m = \left \lbrace \begin{array}{cc} 1 & \text {falls } m [/latex]

zu [latex]m \in \mathbb {N}[/latex]. Dies ist eine Cauchy-Folge. In der Tat existiert für jedes rationale [latex]\varepsilon > 0[/latex] ein [latex]N_\varepsilon \in \mathbb {N}[/latex], so dass für [latex]m,n\geq N_\varepsilon[/latex] die Abschätzung [latex]|x_m-x_n| <_ und damit>

[latex]
\begin{aligned}[]|y_m-y_n| = |x_m^{-1} - x_n^{-1}| = |x_m^{-1}||x_n^{-1}| |x_m-x_n|\leq _\mathbb {Q} \delta ^{-2} |x_m-x_n| <_>[/latex]

für alle [latex]m,n \geq \max (N,N_\varepsilon )[/latex] gilt. Nun folgt

[latex]
\begin{aligned}[][(x_n)_n]\cdot [(y_n)_n] = [(x_ny_n)_n] = [(1)_n],\end{aligned}
[/latex]

da [latex](x_ny_n-1)_n \in V_0[/latex] eine Nullfolge ist (sie ist schliesslich konstant Null). Da [latex][(1)_n][/latex] das multiplikative Einselement in [latex]\mathbb {R}[/latex] ist, haben wir die Existenz der multiplikativen Inversen gezeigt. Also ist [latex](\mathbb {R},+,\cdot )[/latex] ein Körper.

Die Ordnung auf den reellen Zahlen: Wir verwenden nun die Ordnung [latex]\leq _\mathbb {Q}[/latex] auf [latex]\mathbb {Q}[/latex], um aus [latex]\mathbb {R}[/latex] einen geordneten Körper zu machen. Für [latex][(x_n)_n],[(y_n)_n]\in \mathbb {R}[/latex] soll per Definition [latex][(x_n)_n]\leq _\mathbb {R} [(y_n)_n][/latex] genau dann gelten, wenn es eine Nullfolge [latex](z_n)_n \in V_0[/latex] gibt mit

[latex]
\begin{aligned}[]x_n \leq _\mathbb {Q} y_n + z_n\end{aligned}
[/latex]

für alle [latex]n \in \mathbb {N}[/latex]. Dies ist in der Tat wohldefiniert und erfüllt die Axiome der Anordnung (die Axiome ()-()) und der Kompatibilität mit der Körperstruktur (Axiome ()-()). Wir beweisen als Beispiel die Linearität der Ordnung [latex]\leq _\mathbb {R}[/latex] (Axiom ()). Seien also [latex][(x_n)_n],[(y_n)_n][/latex] in [latex]\mathbb {R}[/latex].

  • Falls es ein [latex]N \in \mathbb {N}[/latex] gibt, so dass [latex]x_n \leq _\mathbb {Q} y_n[/latex] für alle [latex]n \geq N[/latex] gilt, dann definieren wir eine Nullfolge [latex](z_n)_n \in V_0[/latex] durch
    [latex]
    \begin{aligned}[]z_n = \left \lbrace \begin{array}{cc} x_n-y_n & \text {falls } n [/latex]

    für [latex]n \in \mathbb {N}[/latex]. Wir erhalten also [latex]x_n \leq _\mathbb {Q} y_n + z_n[/latex] für alle [latex]n \in \mathbb {N}[/latex] und damit [latex][(x_n)_n]\leq _\mathbb {R} [(y_n)_n][/latex].

  • Falls [latex]x_n \geq _\mathbb {Q} y_n[/latex] für fast alle [latex]n \in \mathbb {N}[/latex] gilt, so folgt analog [latex][(x_n)_n]\geq _\mathbb {R} [(y_n)_n][/latex].
  • Es verbleibt den Fall zu behandeln, wo [latex]x_n \leq _\mathbb {Q} y_n[/latex] für unendlich viele [latex]n\in \mathbb {N}[/latex] und ebenso [latex]x_n \geq _\mathbb {Q} y_n[/latex] für unendlich viele [latex]n\in \mathbb {N}[/latex] gilt. In diesem Fall behaupten wir, dass [latex]z_n = x_n -y_n[/latex] für [latex]n \in \mathbb {N}[/latex] eine Nullfolge [latex](z_n)_n \in V_0[/latex] definiert und damit [latex][(x_n)_n] = [(y_n)_n][/latex] und insbesondere [latex][(x_n)_n] \leq _\mathbb {R} [(y_n)_n][/latex] gilt. Sei also [latex]\varepsilon >_\mathbb {Q} 0[/latex] rational. Dann existiert ein [latex]N \in \mathbb {N}[/latex], so dass [latex]|x_m-x_n| <_ und f alle n gilt. sei so dass _ y_ dann folgt beliebig class="wp-caption aligncenter">[latex]
    \begin{aligned}[]-2 \varepsilon \leq _\mathbb {Q} x_{m_2}-y_{m_2} -2 \varepsilon \leq _\mathbb {Q} x_n-y_n \leq _\mathbb {Q} x_{m_1} - y_{m_1} + 2 \varepsilon \leq _{\mathbb {Q}} 2\varepsilon .\end{aligned}
    [/latex]

Da [latex]\varepsilon >_\mathbb {Q} 0[/latex] beliebig war, folgt die Behauptung und damit, dass [latex]\leq _\mathbb {R}[/latex] eine lineare Ordnung (Axiom ()) ist.

Zusammenfassend können wir also sagen, dass wir die Axiome (2.2)-() mehr oder weniger direkt von [latex]\mathbb {Q}[/latex] auf die hier aus [latex]\mathbb {Q}[/latex] definierten reellen Zahlen [latex]\mathbb {R}[/latex] übertragen können.

Wir betrachten [latex]\mathbb {Q}[/latex] als eine Teilmenge von [latex]\mathbb {R}[/latex], indem wir [latex]q \in \mathbb {Q}[/latex] mit [latex][(q)_n] \in \mathbb {R}[/latex] identifizieren (die Abbildung [latex]q \in \mathbb {Q} \mapsto [(q)_n] \in \mathbb {R}[/latex] ist injektiv). Da [latex]q \leq _\mathbb {Q} q'[/latex] für [latex]q,q' \in \mathbb {Q}[/latex] genau dann gilt, wenn [latex][(q)_n] \leq _\mathbb {R} [(q')_n][/latex] gilt, ist diese Identifikation mit der von uns definierten Ordnung auf [latex]\mathbb {R}[/latex] kompatibel und wir können zur Vereinfachung der Notation wieder schlicht [latex]\leq[/latex] für [latex]\leq _\mathbb {Q}[/latex] und [latex]\leq _\mathbb {R}[/latex] schreiben. Weiter schreiben wir im Folgenden [latex]\overline {{x}}\in \mathbb {R}[/latex] anstelle von [latex][(x_n)_n] \in \mathbb {R}[/latex].

Wir wollen zuletzt das entscheidende Axiom der Vollständigkeit (Axiom ()) sorgfältiger beweisen, da dieses ja nicht von [latex]\mathbb {Q}[/latex] nicht erfüllt wird.

Das Archimedische Prinzip: Wir behaupten zuerst, dass zu [latex]\overline {{x}} \in \mathbb {R}[/latex] mit [latex]\overline {{x}} > 0[/latex] ein [latex]n\in \mathbb {N}[/latex] mit [latex]0 0[/latex] mit [latex]|x_n| > \delta[/latex] für fast alle [latex]n \in \mathbb {N}[/latex] gibt. Jedes [latex]n \in \mathbb {N}[/latex] mit [latex]\frac {1}{n}

Vollständigkeit von [latex]\mathbb {R}[/latex]: Seien [latex]X,Y \subseteq \mathbb {R}[/latex] zwei nicht-leere Teilmengen, so dass [latex]\overline {{x}} \leq \overline {{y}}[/latex] für alle [latex]\overline {{x}} \in X[/latex] und [latex]\overline {{y}} \in Y[/latex]. Wir wollen nun eine reelle Zahl zwischen [latex]X[/latex] und [latex]Y[/latex] finden indem wir eine Cauchy-Folge in [latex]\mathbb {Q}[/latex] konstruieren.

Sei [latex]\overline {{x}}=[(x_n)_n] \in X[/latex] beliebig. Da [latex](x_n)_n[/latex] eine Cauchy-Folge ist, gibt es ein [latex]N \in \mathbb {N}[/latex], so dass [latex]|x_m-x_n|

[latex]
\begin{aligned}[]a_1\leq Y\end{aligned}
[/latex]

schreiben werden. Genauso kann man unter Verwendung eines [latex]\overline {{y}} \in Y[/latex] ein [latex]b_1 \in \mathbb {Q}[/latex] finden mit [latex]\overline {{x}}\leq b_1[/latex] für alle [latex]\overline {{x}} \in X[/latex], was wir kurz als

[latex]
\begin{aligned}[]X\leq b_1\end{aligned}
[/latex]

schreiben werden.

Wir nehmen nun an, dass wir bereits [latex]a_1,\ldots ,a_m ,b_1,\ldots , b_m \in \mathbb {Q}[/latex] konstruiert haben mit

[latex]
\begin{aligned}[]a_1\leq a_2 \leq \ldots \leq a_m \leq b_m \leq b_{m-1} \leq \ldots \leq b_1,\quad b_k-a_k=\frac {b_1-a_1}{2^{k-1}}\end{aligned}
[/latex]

für alle [latex]k=1,\ldots ,m[/latex] und der Eigenschaft [latex]a_m \leq Y[/latex], [latex]X \leq b_m[/latex]. Wir möchten nun [latex]a_{m+1},b_{m+1} \in \mathbb {Q}[/latex] definieren. Dabei unterscheiden wir drei Möglichkeiten, wie [latex]c_m=\frac {a_m+b_m}2[/latex] gegenüber [latex]X[/latex] und [latex]Y[/latex] stehen kann.

  • Es gibt ein [latex]\overline {{x}} \in X[/latex] mit [latex]c_m
  • Es gibt ein [latex]\overline {{y}} \in Y[/latex] mit [latex]\overline {{y}}
  • Es gilt [latex]\overline {{x}} \leq c_m \leq \overline {{y}}[/latex] für alle [latex]\overline {{x}}\in X[/latex] und [latex]\overline {{y}} \in Y[/latex]. In diesem Fall erfüllt [latex]c_m \in \mathbb {Q}[/latex] bereits die Existenzaussage des Vollständigkeitsaxioms.

Falls obige Rekursion nicht bereits nach endlich vielen Schritten zum Ziel führt (das heisst, immer entweder der erste oder der zweite Fall eintritt), so konstruiert diese stattdessen zwei Folgen [latex](a_n)_n, (b_n)_n[/latex] in [latex]\mathbb {Q}[/latex] mit [latex]a_m \leq b_n[/latex], [latex]a_m \leq \overline {{y}}[/latex], [latex]\overline {{x}} \leq b_n[/latex] sowie [latex]b_n-a_n = \frac {b_1-a_1}{2^{n-1}}[/latex] für alle [latex]m,n\in \mathbb {N}[/latex] und [latex]\overline {{x}} \in X[/latex], [latex]\overline {{y}} \in Y[/latex]. Es folgt insbesondere, dass [latex]z_n = b_n-a_n \in \mathbb {Q}[/latex] für [latex]n \in \mathbb {N}[/latex] eine Nullfolge [latex](z_n)_n[/latex] definiert (wieso?). Für [latex]\varepsilon \in \mathbb {Q}[/latex] mit [latex]\varepsilon > 0[/latex] gibt es also ein [latex]N[/latex], so dass [latex]0

[latex]
\begin{aligned}[]\overline {{c}} = [(a_n)_n] )= [(a_n + z_n)_n] = [(b_n)_n] \in \mathbb {R}\end{aligned}
[/latex]

Wir behaupten nun, dass [latex]\overline {{c}}[/latex] zwischen [latex]X[/latex] und [latex]Y[/latex] liegt.

Für dies bemerken wir, dass [latex]a_m\leq \overline {{c}}[/latex] (da [latex]a_m\leq b_n[/latex] für alle [latex]n\in \mathbb {N}[/latex]) und [latex]\overline {{c}}\leq b_m[/latex] (da ebenso [latex]a_n\leq b_m[/latex] für alle [latex]n\in \mathbb {N}[/latex]) für alle [latex]m\in \mathbb {N}[/latex]. Weiters ist [latex](b_m-a_m)_m[/latex] eine Nullfolge in [latex]\mathbb {Q}[/latex] und auf Grund des Archimedischen Prinzips auch in [latex]\mathbb {R}[/latex]. Daraus folgt

[latex]
\begin{aligned}[]\overline {{c}}=\lim _{m\to \infty }a_m=\lim _{m\to \infty }b_m\end{aligned}
[/latex]

und mit [latex]a_m\leq Y[/latex] und [latex]X\leq b_m[/latex] für alle [latex]m\in \mathbb {N}[/latex] auch [latex]\overline {{c}}\leq Y[/latex] und [latex]X\leq \overline {{c}}[/latex]. Dies beweist die Existenz im Vollständigkeitsaxiom und beendet den Beweis. ∎

9.9 – Weitere Lernmaterialien

9.9.1 – Verwendung des Kapitels

Wie Sie sicher schon bemerkt haben, ist dieses Kapitel vielmehr ein theoretisches Hilfsmittel als ein praktisches Werkzeug für Berechnungen. Die zentralen Begriffe dieses Kapitels sind Konvergenz, Stetigkeit, Kompaktheit und Vollständigkeit, wobei wir diese Begriffe eben für metrische Räume besprochen haben und diese Begriffe damit universeller einsetzbar geworden sind. Insbesondere werden diese Begriffe für die mehrdimensionale Analysis von grundlegender Bedeutung sein. Wir bemerken noch, dass viele der Sätze des ersten Semesters bei Verallgemeinerung ins Mehrdimensionale den Begriff der zusammenhängenden offenen Teilmenge benötigen werden. Des Weiteren haben wir auch die Vollständigkeit des Funktionenraums [latex]C(K,\mathbb {R}^d)[/latex] bewiesen und werden dies später gemeinsam mit dem auch sehr allgemein gehaltenen Banachschen Fixpunktsatz verwenden, um implizite Gleichungen und Differentialgleichungen abstrakt zu lösen.

9.9.2 – Weitere Normen und Metriken

Wir möchten Ihnen in diesem Abschnitt die Gelegenheit geben, andere Normen oder Metriken kennenzulernen, die in gewissen Situationen von Interesse sein können.

Übung: [latex]p[/latex]-Normen

In dieser Übung möchten wir für jedes [latex]p \geq 1[/latex] eine Norm, die sogenannte [latex]p[/latex]-Norm definieren, die für [latex]p=1[/latex] gerade die Einsnorm und für [latex]p=2[/latex] gerade die Euklidsche Norm ist. Wir setzen für [latex]{x} = (x_1,\ldots ,x_d)^t \in \mathbb {R}^d[/latex]

[latex]
\begin{aligned}[]\| {{x}}\| _p = \left (\sum _{k=1}^d |x_k|^p\right )^{\tfrac {1}{p}}.\end{aligned}
[/latex]

Verfizieren Sie zuerst, dass [latex]\| {\cdot }\| _p[/latex] definit und homogen ist. Zeigen Sie dann die Dreiecksungleichung in folgenden Schritten. Sei [latex]q = \frac {p}{p-1}[/latex].

  1. (Young-Ungleichung) Zeigen Sie für [latex]a,b \geq 0[/latex]
    [latex]
    \begin{aligned}[]ab \leq \frac {a^p}{p} + \frac {b^q}{q}.\end{aligned}
    [/latex]
  2. (Hölder-Ungleichung) Zeigen Sie für [latex]{x} = (x_1,\ldots ,x_d)^t[/latex] und [latex]{y} = (y_1,\ldots ,y_d)^t[/latex]
    [latex]
    \begin{aligned}[]\sum _{k=1}^d |x_ky_k| \leq \| {{x}}\| _p \| {{y}}\| _q.\end{aligned}
    [/latex]
  3. (Dreiecksungleichung/Minkowski-Ungleichung) Zeigen Sie für [latex]{x},{y} \in \mathbb {R}^d[/latex]
    [latex]
    \begin{aligned}[]\| {{x}+ {y}}\| _p \leq \| {{x}}\| _p + \| {{y}}\| _p.\end{aligned}
    [/latex]

Hinweis.

Für (i): Betrachten Sie die Funktion [latex]a \mapsto ab - \frac {a^p}{p}[/latex]. Für (ii): Zeigen Sie, dass Sie [latex]\| {{x}}\| _p = \| {{y}}\| _q = 1[/latex] annehmen können und setzen Sie [latex]a = |x_k|[/latex] und [latex]b = |y_k|[/latex] für [latex]k \in \left \lbrace {1,\ldots ,d} \right \rbrace[/latex]. Für (iii): Schreiben Sie

[latex]
\begin{aligned}[]\sum _{k=1}^d |x_k+y_k|^p \leq \sum _{k=1}^d |x_k||x_k+y_k|^{p-1} + \sum _{k=1}^d |y_k||x_k+y_k|^{p-1}.\end{aligned}
[/latex]

Übung: Hexagon-Metrik

Finden Sie eine Norm auf [latex]\mathbb {R}^2[/latex], so dass der Einheitsball [latex]B_1(0)[/latex] bezüglich der induzierten Metrik das reguläre Hexagon mit Eckpunkt [latex](1,0)[/latex] ist. Gibt es eine Norm auf [latex]\mathbb {R}^2[/latex], so dass der Einheitsball durch ein reguläres Pentagon gegeben ist?

Übung: Ultrametriken

Eine Ultrametrik auf einer Menge [latex]X[/latex] ist eine Abbildung [latex]\mathrm {d}:X \times X \to \mathbb {R}_{\geq 0}[/latex], die die gleichen Eigenschaften wie eine Metrik hat, abgesehen davon, dass sie anstelle der Dreiecksungleichung die Ungleichung

[latex]
\begin{aligned}[]\mathrm {d}(x_1,x_3) \leq \max \left \lbrace {\mathrm {d}(x_1,x_2),\mathrm {d}(x_2,x_3)} \right \rbrace\end{aligned}
[/latex]

für alle [latex]x_1,x_2,x_3\in X[/latex] erfüllt.

  1. Zeigen Sie, dass jede Ultrametrik eine Metrik ist.
  2. ([latex]p[/latex]-adische Metrik auf [latex]\mathbb {Z}[/latex]) Sei [latex]p \in \mathbb {Z}[/latex] eine Primzahl. Wir definieren
    [latex]
    \begin{aligned}[]\nu _p(x) = \max \left \lbrace {k} \mid {p^k \text { teilt } x}\right \rbrace\end{aligned}
    [/latex]

    für [latex]x \in \mathbb {Z} \setminus \left \lbrace {0} \right \rbrace[/latex] und [latex]\mathrm {d}_p:\mathbb {Z} \times \mathbb {Z} \to \mathbb {R}_{\geq 0}[/latex] durch

    [latex]
    \begin{aligned}[]\mathrm {d}_p(x,y) = \left \lbrace \begin{array}{cc} p^{-\nu _p(x-y)} & \text {falls } x \neq y \\ 0 & \text {falls } x=y\end{array} \right . .\end{aligned}
    [/latex]

    für [latex]x,y\in \mathbb {Z}[/latex]. Zeigen Sie, dass [latex]\mathrm {d}_p[/latex] eine Ultrametrik auf [latex]\mathbb {Z}[/latex] definiert und beschreiben Sie die Bälle in dieser Metrik.

  3. Sei [latex]X[/latex] eine Menge und [latex]\mathrm {d}[/latex] eine Ultrametrik. Zeigen Sie, dass für alle [latex]r>0[/latex], [latex]x_0 \in X[/latex] und [latex]x\in B_r(x_0)[/latex] der Ball von Radius [latex]r[/latex] um [latex]x[/latex] gleich [latex]B_r(x_0)[/latex] ist. In anderen Worten ist jeder Punkt in einem Ball Zentrum dieses Balles.

Übung: Rangmetrik

Sei [latex]\mathbb {K}[/latex] ein Körper und [latex]X[/latex] die Menge der [latex]m \times n[/latex]-Matrizen über [latex]\mathbb {K}[/latex]. Wir definieren [latex]\mathrm {d}(A,B) = \operatorname {rang}(A-B)[/latex] für [latex]A,B \in X[/latex]. Zeigen Sie, dass [latex]\mathrm {d}[/latex] eine Ultrametrik auf [latex]X[/latex] ist.

Hinweis.

Für die Dreiecksungleichung reicht es [latex]\operatorname {rang}(A+B)\leq \operatorname {rang}(A)+\operatorname {rang}(B)[/latex] für alle [latex]A,B \in X[/latex] zu beweisen.

Übung: Verschiedene Produktmetriken und die Produkttopologie

In dieser Übung möchten wir kartesische Produkte von metrischen Räumen mit Metriken ausstatten, so dass «horizontale und vertikale» Abstände gewissermassen natürlich sind. Seien [latex](X,\mathrm {d}_X)[/latex] und [latex](Y,\mathrm {d}_Y)[/latex] metrische Räume. Für [latex](x_1,y_1),(x_2,y_2) \in X\times Y[/latex] definieren wir

[latex]
\begin{aligned}[]\mathrm {d}_\infty ((x_1,y_1),(x_2,y_2)) &= \max \left \lbrace {\mathrm {d}_X(x_1,x_2),\mathrm {d}_Y(y_1,y_2)} \right \rbrace \\ \mathrm {d}_1((x_1,y_1),(x_2,y_2)) &= \mathrm {d}_X(x_1,x_2) + \mathrm {d}_Y(y_1,y_2)\\ \mathrm {d}_2((x_1,y_1),(x_2,y_2)) &= \sqrt {\mathrm {d}_X(x_1,x_2)^2 + \mathrm {d}_Y(y_1,y_2)^2}.\end{aligned}
[/latex]

Verfizieren Sie, dass [latex]\mathrm {d}_\infty ,\mathrm {d}_1[/latex] und [latex]\mathrm {d}_2[/latex] Metriken auf [latex]X \times Y[/latex] definieren und dass diese Metriken die gleiche Topologie induzieren. Diese Topologie nennt sich die Produkttopologie auf [latex]X \times Y[/latex]. Zeigen Sie, dass eine Teilmenge [latex]U \subseteq X \times Y[/latex] genau dann offen ist, wenn für jedes [latex]x\in U[/latex] offene Teilmenge [latex]U_1 \subseteq X[/latex] und [latex]U_2 \subseteq Y[/latex] mit [latex]x\in U_1 \times U_2 \subseteq U[/latex] existieren.

Übung: Metriken auf Folgenräumen

Für einen beschränkten metrischen Raum [latex](X,\mathrm {d})[/latex] und Punkte [latex](x_n)_n,(y_n)_n \in X^{\mathbb {N}}[/latex] setzen wir

[latex]
\begin{aligned}[]\mathrm {d}'((x_n)_n),(y_n)_n) = \sum _{n=1}^\infty 2^{-n} \mathrm {d}(x_n,y_n).\end{aligned}
[/latex]

Zeigen Sie, dass [latex]\mathrm {d}'[/latex] eine (wohldefinierte) Metrik auf [latex]X^\mathbb {N}[/latex] darstellt. Fast in Analogie zu obiger Übung kann man auch hier die offenen Mengen charakterisieren. Zeigen Sie, dass eine Teilmenge [latex]U \subseteq X^\mathbb {N}[/latex] genau dann offen ist, wenn für alle [latex]x\in U[/latex] offene Mengen [latex]U_1,U_2,\ldots[/latex] in [latex]X[/latex] existieren mit [latex]U_n = X[/latex] für fast alle [latex]n\in \mathbb {N}[/latex] und

[latex]
\begin{aligned}[]x \in \prod _{n \in \mathbb {N}} U_n = \left \lbrace {(y_n)_n\in X^\mathbb {N}} \mid {y_n \in U_n \text { für alle } n \in \mathbb {N}}\right \rbrace \subseteq U.\end{aligned}
[/latex]

Übung

Sei [latex]f:\mathbb {R}_{\geq 0} \to \mathbb {R}_{\geq 0}[/latex] eine monoton wachsende, konkave Funktion mit [latex]f(x) = 0 \iff x = 0[/latex] für alle [latex]x \in \mathbb {R}_{\geq 0}[/latex] und sei [latex](X,\mathrm {d})[/latex] ein metrischer Raum. Zeigen Sie wie folgt, dass [latex]f \circ \mathrm {d}[/latex] eine Metrik auf [latex]X[/latex] definiert.

  1. Beweisen Sie die Ungleichung [latex]f(tx) \geq tf(x)[/latex] für alle [latex]x \in \mathbb {R}_{\geq 0}[/latex] und [latex]t \in [0,1][/latex].
  2. Zeigen Sie, dass für alle [latex]x,y \in \mathbb {R}_{\geq 0}[/latex] die Ungleichung [latex]f(x)+f(y) \geq f(x+y)[/latex] gilt ([latex]f[/latex] ist subadditiv).
  3. Schliessen Sie auf die zu beweisende Aussage.

Geben Sie des Weiteren ein Beispiel einer solchen Abbildung [latex]f[/latex] und der Metrik [latex]f \circ \mathrm {d}[/latex] für [latex]X = \mathbb {R}[/latex] und die Standardmetrik [latex]\mathrm {d}[/latex] auf [latex]\mathbb {R}[/latex] an.

Übung: Hausdorff-Metrik

Sei [latex](X,\mathrm {d})[/latex] ein metrischer Raum. In dieser Übung möchten wir einen Abstand von Teilmengen von [latex]X[/latex] zueinander einführen und verwenden die in Übung 9.41 definierten Distanzfunktionen. Für nicht-leere, abgeschlossene Teilmengen [latex]A,B \subseteq X[/latex] sei

[latex]
\begin{aligned}[]\mathrm {d}(A,B) = \max \left \lbrace {\sup _{a \in A} \mathrm {d}(a,B),\sup _{b \in B} \mathrm {d}(b,A)} \right \rbrace\end{aligned}
[/latex]

Für die Wohldefiniertheit wollen wir hierbei annehmen, dass [latex]\mathrm {d}(x,y) \leq M[/latex] für ein [latex]M > 0[/latex] und alle [latex]x,y \in X[/latex] (siehe Übung 9.6). Zeigen Sie, dass obiges eine Metrik auf der Menge der nicht-leeren abgeschlossenen Teilmengen von [latex]X[/latex] definiert.

9.9.3 – Weitere Übungen

Übung: Teilmengen gegeben durch Gleichungen

  1. Sei [latex]X[/latex] ein metrischer Raum und [latex]\mathcal {I}[/latex] eine Menge. Gegeben sei für jedes [latex]\alpha \in \mathcal {I}[/latex] ein metrischer Raum [latex]Y_\alpha[/latex], eine stetige Abbildung [latex]f_\alpha : X \to Y_\alpha[/latex] und ein Punkt [latex]y_\alpha \in Y_\alpha[/latex]. Zeigen Sie, dass die durch Gleichungen gegebene Teilmenge
    [latex]
    \begin{aligned}[]\left \lbrace {x \in X} \mid {\forall \alpha \in \mathcal {I}: f_\alpha (x) = y_\alpha }\right \rbrace \subseteq X\end{aligned}
    [/latex]

    abgeschlossen ist. Zeigen Sie auch, dass die Teilmenge [latex]\left \lbrace {x \in X} \mid {\forall \alpha \in \mathcal {I}: f_\alpha (x) \neq y_\alpha }\right \rbrace[/latex] offen ist, falls [latex]\mathcal {I}[/latex] eine endliche Menge ist.

Wenden Sie dies nun auf einige anschauliche Beispiele an.

  1. Zeigen Sie, dass die Sphäre [latex]\mathbb {S}^{d-1} = \left \lbrace {{x} = (x_1,\ldots ,x_{d}) \in \mathbb {R}^{d}} \mid {x_1^2+\ldots +x_{d}^2 = 1}\right \rbrace[/latex] für [latex]d \geq 1[/latex] abgeschlossen ist.
  2. Zeigen Sie, dass alle (linearen) Unterräume von [latex]\mathbb {C}^d[/latex] für [latex]d \geq 1[/latex] abgeschlossen sind.
  3. Sei [latex]I \subseteq \mathbb {R}[/latex] ein Intervall und [latex]\gamma : I \to \mathbb {R}^d[/latex] eine stetige Funktion für [latex]d \geq 1[/latex] (ein stetiger Weg in [latex]\mathbb {R}^d[/latex]). Zeigen Sie, dass der Graph von [latex]\gamma[/latex]
    [latex]
    \begin{aligned}[]\left \lbrace {{x} = (t,{v}) \in \mathbb {R}^{d+1}} \mid {\gamma (t) = {v}}\right \rbrace \subseteq \mathbb {R}^{d+1}\end{aligned}
    [/latex]

    (auch die Spur des Weges [latex]\gamma[/latex] genannt) abgeschlossen ist.

Übung: Urysohn’s Lemma

Sei [latex]X[/latex] ein metrischer Raum. Wir möchten zeigen, dass sich disjunkte, abgeschlossene Teilmengen von [latex]X[/latex] mit einer stetigen Funktion «trennen» lassen. Seien also [latex]A,B \subseteq X[/latex] abgeschlossene, nicht-leere, disjunkte Teilmengen. Zeigen Sie, dass eine stetige Funktion [latex]f: X \to [0,1][/latex] mit [latex]f(a) =0[/latex] für alle [latex]a \in A[/latex] und [latex]f(b) = 1[/latex] für alle [latex]b \in B[/latex] existiert. Betrachten Sie dazu die Funktion

[latex]
\begin{aligned}[]x \in X \mapsto \frac {\mathrm {d}(x,A)}{\mathrm {d}(x,A)+\mathrm {d}(x,B)}\end{aligned}
[/latex]

und verifizieren Sie auch, dass diese wohldefiniert ist.

Übung: Fortsetzungssatz von Tietze

Sei [latex]X[/latex] ein metrischer Raum, sei [latex]A \subseteq X[/latex] eine nicht-leere abgeschlossene Teilmenge und sei [latex]f: A \to \mathbb {R}_{\geq 0}[/latex] eine stetige, beschränkte Funktion. Für alle [latex]x\in X[/latex] setzen wir

[latex]
\begin{aligned}[]F(x) = \left \lbrace \begin{array}{cc} f(x) & \text {falls } x \in A \\ \inf \left \lbrace {f(a) + \frac {\mathrm {d}(x,a)}{\mathrm {d}(x,A)}-1} \mid {a \in A}\right \rbrace & \text {falls } x \not \in A\end{array} \right . .\end{aligned}
[/latex]

Zeigen Sie, dass [latex]F:X \to \mathbb {R}_{\geq 0}[/latex] eine stetige Funktion mit [latex]F|_A = f[/latex] ist.

Hinweis.

Um Stetigkeit bei einem [latex]x \not \in A[/latex] zu zeigen, können Sie zuerst beweisen, dass für [latex]a \in A[/latex] beliebig [latex]x' \mapsto \frac {\mathrm {d}(x',a)}{\mathrm {d}(x',A)}[/latex] auf einem kleinen Ball um [latex]x[/latex] Lipschitz-stetig ist, wobei die Lipschitz-Konstante von [latex]a[/latex] unabhängig ist.

Um Stetigkeit bei einem [latex]x \in A[/latex] zu zeigen, können Sie eine Folge [latex](x_n)_n[/latex] in [latex]X \setminus A[/latex] mit [latex]x_n \to a \in A[/latex] betrachten. Argumentieren Sie nun, wieso [latex]\limsup _{n \to \infty } F(x_n) \leq f(a) \leq \liminf _{n \to \infty } F(x_n)[/latex]. Für die zweite Ungleichung können Sie die Folge [latex](x_n)_n[/latex] mit einer Folge [latex](a_n)_n[/latex], welche

[latex]
\begin{aligned}[]f(a_n) + \frac {\mathrm {d}(x_n,a_n)}{\mathrm {d}(x,A)}-1 \leq F(x_n) + \mathrm {d}(x_n,A)\end{aligned}
[/latex]

genügt, vergleichen.

Übung: Stetigkeit der Matrixmultiplikation

Seien [latex]d,d' \geq 1[/latex].

  1. Zeigen Sie, dass [latex]\operatorname {Mat}_{d',d}(\mathbb {C}) \times \mathbb {C}^d,\ (A,{x}) \mapsto A{x}\in \mathbb {C}^{d'}[/latex] stetig ist.
  2. Zeigen Sie, dass die Gruppe der invertierbaren [latex]d\times d[/latex]-Matrizen [latex]\operatorname {GL}_d(\mathbb {C})[/latex] eine offene Teilmenge von [latex]\operatorname {Mat}_{d,d}(\mathbb {C})[/latex] bildet.
  3. Beweisen Sie, dass die Abbildungen
    [latex]
    \begin{aligned}[](A,B) \in \operatorname {GL}_d(\mathbb {C}) \mapsto AB \in \operatorname {GL}_d(\mathbb {C}),\quad A \in \operatorname {GL}_d(\mathbb {C}) \mapsto A^{-1} \in \operatorname {GL}_d(\mathbb {C})\end{aligned}
    [/latex]

    stetig sind.

Hinweis.

In (c) können Sie die Tatsache verwenden, dass für alle [latex]A \in \operatorname {GL}_n(\mathbb {C})[/latex] die Inverse von [latex]A[/latex] durch [latex]\frac {1}{\det (A)}A^{\natural }[/latex] gegeben ist, wobei [latex]A^{\natural }[/latex] die Adjunkte von [latex]A[/latex] ist.

Übung: Topologie von Matrixgruppen

Sei [latex]d \geq 1[/latex].

  1. Zeigen Sie, dass [latex]\operatorname {GL}_d(\mathbb {C})[/latex] wegzusammenhängend und somit auch zusammenhängend ist. Verifizieren Sie auch, dass [latex]\operatorname {GL}_d(\mathbb {R})[/latex] nicht zusammenhängend ist.
  2. Zeigen Sie, dass [latex]\operatorname {SL}_d(\mathbb {R}) = \left \lbrace {A \in \operatorname {GL}_d(\mathbb {R})} \mid {\det (A) = 1}\right \rbrace[/latex] wegzusammenhängend ist.
  3. Zeigen Sie, dass [latex]\operatorname {SO}(d) = \left \lbrace {A \in \operatorname {SL}_d(\mathbb {R})} \mid {A^T A = \operatorname {id}}\right \rbrace[/latex] kompakt und zusammenhängend ist.

Hinweis.

Betrachten Sie in (i) zu [latex]A,B \in \operatorname {GL}_d(\mathbb {C})[/latex] das komplexe Polynom [latex]\det (zA+(1-z)B)[/latex] in der Variable [latex]z[/latex]. In (ii) können Sie verwenden, dass sich jede Matrix in [latex]\operatorname {SL}_d(\mathbb {R})[/latex] als Produkt von unteren und oberen Dreiecksmatrizen mit Einsen auf der Diagonalen schreiben lässt.

Übung: Topologische Eigenschaften von Produkträumen

Seien [latex]X[/latex] und [latex]Y[/latex] nicht-leere metrische Räume. In einer Übung in Abschnitt 9.9.2 wurden Metriken [latex]\mathrm {d}_\infty , \mathrm {d}_1, \mathrm {d}_2[/latex] auf dem Produkt [latex]X \times Y[/latex] definiert, welche allesamt die gleiche Topologie (die Produkttopologie) induzieren. Im Folgenden möchten wir topologische Eigenschaften des Produktraumes [latex]X \times Y[/latex] charakterisieren.

  1. Zeigen Sie, dass eine Folge in [latex]X \times Y[/latex] genau dann konvergiert, wenn sie komponentenweise konvergiert.
  2. Beweisen Sie, dass [latex]X \times Y[/latex] genau dann kompakt ist, wenn [latex]X[/latex] und [latex]Y[/latex] kompakt sind.
  3. Zeigen Sie, dass [latex]X \times Y[/latex] genau dann zusammenhängend ist, wenn [latex]X[/latex] und [latex]Y[/latex] zusammenhängend sind.
  4. Zeigen Sie, dass [latex]X \times Y[/latex] genau dann wegzusammenhängend ist, wenn [latex]X[/latex] und [latex]Y[/latex] wegzusammenhängend sind.

Übung: Der abzählbare Diagonalfolgentrick

Sei [latex][0,1]^{\mathbb {N}}[/latex] die Menge der [latex][0,1][/latex]-wertigen Folgen und sei

[latex]
\begin{aligned}[]\mathrm {d}({x},{y}) = \sum _{n=1}^\infty |x_n-y_n| 2^{-n}\end{aligned}
[/latex]

für [latex]{x}=(x_n)_n,{y}=(y_n)_n \in [0,1]^\mathbb {N}[/latex]. In einer Übung in Abschnitt 9.9.2 wurde gezeigt, dass [latex]\mathrm {d}[/latex] in der Tat eine Metrik auf [latex][0,1]^{\mathbb {N}}[/latex] definiert. Auch wurde die Topologie auf [latex][0,1]^{\mathbb {N}}[/latex] beschrieben. In dieser Übung möchten wir beweisen, dass [latex][0,1]^{\mathbb {N}}[/latex] kompakt ist. Sei also [latex]({x}^{(n)})_n[/latex] eine Folge in [latex][0,1]^\mathbb {N}[/latex].

  1. Zeigen Sie, dass eine Folge in [latex][0,1]^\mathbb {N}[/latex] genau dann konvergiert, wenn sie komponentenweise konvergiert.

Wir konstruieren nun eine konvergente Teilfolge von [latex]({x}^{(n)})_n[/latex] wie folgt.

  1. Finden Sie eine unendliche Teilmenge [latex]\mathcal {J}_1 \subseteq \mathbb {N}[/latex], so dass [latex](\pi _1({x}^{(n)}))_{n \in \mathcal {J}_1}[/latex] konvergiert. Genau genommen wählt man hier für die korrekte Formulierung eine streng monotone Bijektion [latex]\varphi _1: \mathbb {N} \to \mathcal {J}_1[/latex], womit die Teilfolge [latex](\pi _1({x}^{(\varphi _1(n))}))_{n}[/latex] konvergieren soll.
  2. Wenden Sie obige Idee rekursiv an, um unendliche Teilmengen [latex]\mathcal {J}_1 \supseteq \mathcal {J}_2 \supseteq \ldots[/latex] zu konstruieren, so dass [latex](\pi _k({x}^{(n)}))_{n \in \mathcal {J}_k}[/latex] für jedes [latex]k \in \mathbb {N}[/latex] konvergiert.

Nun würde man gerne den Schnitt [latex]\bigcap _{k \in \mathbb {N}} \mathcal {J}_k[/latex] zum Auffinden einer konvergenten Teilfolge betrachten. Da hier aber über unendliche viele Teilmengen geschnitten wird, könnte dieser leer sein. Stattdessen geht man wie folgt vor.

  1. Sei [latex](n_k)_k[/latex] eine aufsteigende Folge in [latex]\mathbb {N}[/latex], so dass [latex]n_k \in \mathcal {J}_k[/latex] für jedes [latex]k \in \mathbb {N}[/latex]. Zeigen Sie, dass [latex]({x}^{(n_k)})_{k}[/latex] konvergiert.

Übung: Umgebungsfilter

Sei [latex](X,\mathrm {d})[/latex] ein metrischer Raum. Der Umgebungsfilter eines Punktes [latex]x \in X[/latex] ist die Menge

[latex]
\begin{aligned}[]\mathcal {U}_x = \left \lbrace {U \subseteq X} \mid {\exists \varepsilon > 0: B_\varepsilon (x) \subseteq U}\right \rbrace .\end{aligned}
[/latex]

Zeigen Sie, dass [latex]\mathcal {U}_x[/latex] für jeden Punkt [latex]x \in X[/latex] einen Filter auf [latex]X[/latex] definiert.

Übung: Regeln für den Abschluss

Sei [latex](X,\mathrm {d})[/latex] ein metrischer Raum und [latex]Y \subseteq X[/latex] eine Teilmenge. Verifizieren Sie die Regeln

[latex]
\begin{aligned}[]\overline {Y_1 \cup Y_2} = \overline {Y_1} \cup \overline {Y_2},\quad \overline {Y_1 \cap Y_2} \subseteq \overline {Y_1} \cap \overline {Y_2}\end{aligned}
[/latex]

für Teilmengen [latex]Y,Y_1,Y_2 \subseteq X[/latex] und erklären Sie anhand eines Beispiels, dass die Gleichheit [latex]\overline {Y_1 \cap Y_2} = \overline {Y_1} \cap \overline {Y_2}[/latex] nicht immer gelten muss.

Übung: Zusammenhangskomponenten eines metrischen Raumes

Sei [latex](X,\mathrm {d})[/latex] ein metrischer Raum.

  1. Zeigen Sie, dass jeder Punkt [latex]x \in X[/latex] in einem maximalen zusammenhängenden Teilraum [latex]Z(x) \subseteq X[/latex] enthalten ist.
  2. Zeigen Sie, dass für [latex]x,y\in X[/latex] entweder [latex]Z(x) = Z(y)[/latex] oder [latex]Z(x) \cap Z(y) = \emptyset[/latex] gilt.
  3. Schliessen Sie, dass die Mengen [latex]Z(x)[/latex] — die sogenannten Zusammenhangskomponenten von [latex]X[/latex]— für [latex]x \in X[/latex] eine Partition von [latex]X[/latex] bilden.

Hinweis.

Für (i) können Sie Übung 9.32 verwenden.

Übung: Abschluss einer zusammenhängenden Teilmenge

Sei [latex](X,\mathrm {d})[/latex] ein metrischer Raum und sei [latex]Y \subseteq X[/latex] ein zusammenhängender Teilraum. Zeigen Sie, dass dann auch jeder Teilraum [latex]B\subseteq X[/latex] mit [latex]Y \subseteq B \subseteq \overline {Y}[/latex] zusammenhängend ist. Verwenden Sie dies, um zu zeigen, dass die Zusammenhangskomponenten von [latex]X[/latex] aus der vorherigen Übung abgeschlossen sind.

Übung: Direkter Beweis gewisser Implikation in Satz 9.66

Sei [latex]X[/latex] ein metrischer Raum. Beweisen Sie folgende Aussagen ohne auf Satz 9.66 zurückzugreifen.

  1. Ist [latex]X[/latex] folgenkompakt, so besitzt [latex]X[/latex] eine Lebesgue-Zahl und [latex]X[/latex] ist total beschränkt.
  2. Erfüllt [latex]X[/latex] das Schachtelungsprinzip, so ist [latex]X[/latex] überdeckungskompakt.
  3. Ist [latex]X[/latex] überdeckungskompakt, so nimmt jede stetige, reellwertige Funktion ein Maximum und ein Minimum an.

Hinweis.

Für (i) können Sie per Widerspruch vorgehen. Für (iii) können Sie die Übung nach Satz 9.72 verwenden (oder zuerst lösen).

Übung

In dieser Übung möchten wir zeigen, dass für gewisse stetige, bijektive Funktion auch deren Inverse stetig ist.

  1. Wir beginnen mit einem Gegenbeispiel zur allgemeinen Aussage. Finden Sie metrische Räume [latex]X,Y[/latex] und eine bijektive, stetige Abbildung [latex]f: X \to Y[/latex], so dass [latex]f^{-1}[/latex] nicht stetig ist.
  2. Seien [latex]X,Y[/latex] metrische Räume und [latex]f: X \to Y[/latex] bijektiv und stetig. Zeigen Sie, dass [latex]f^{-1}[/latex] stetig ist, falls [latex]X[/latex] kompakt ist.

Hinweis.

Zu (ii): Verifizieren Sie, dass für [latex]A \subseteq X[/latex] abgeschlossen auch [latex]f(A)[/latex] abgeschlossen ist.

Übung: Diskrete Teilmengen von kompakten Räumen

Sei [latex]X[/latex] ein metrischer Raum. Wir nennen eine Teilmenge [latex]A \subseteq X[/latex] diskret, falls es um jeden Punkt [latex]x_0[/latex] in [latex]A[/latex] einen Ball [latex]B_r(x_0) \subseteq X[/latex] gibt mit [latex]B_r(x_0) \cap A = \left \lbrace {x_0} \right \rbrace[/latex]. Beispielweise ist [latex]\mathbb {Z} \subseteq \mathbb {R}[/latex] diskret. Zeigen Sie, dass eine diskrete, abgeschlossene Teilmenge eines kompakten metrischen Raumes endlich ist.

Übung: Separable Räume

Wir nennen einen metrischen Raum [latex]X[/latex] separabel, falls er eine abzählbare dichte Teilmenge enthält. Beispielsweise ist [latex]\mathbb {R}[/latex] separabel, da [latex]\mathbb {Q} \subseteq \mathbb {R}[/latex] abzählbar und dicht ist. Zeigen Sie, dass jeder kompakte metrische Raum separabel ist.

Übung: Challenge — Satz von Arzelà-Ascoli

Der Satz von Heine-Borel beschreibt die kompakten Teilmengen von [latex]\mathbb {R}^d[/latex] für [latex]d\geq 1[/latex]. In dieser Übung möchten wir die kompakten Teilmengen in einer weiteren Situation beschreiben. Sei [latex]K[/latex] ein kompakter metrischer Raum und sei [latex]\mathcal {F} \subseteq C(K)[/latex] eine Familie stetiger Funktionen, wobei wir [latex]C(K)[/latex] mit der Norm [latex]\| {\cdot }\| _\infty[/latex] ausstatten.

Zeigen Sie, dass [latex]\mathcal {F}[/latex] genau dann kompakt ist, wenn die folgenden drei Bedingungen erfüllt sind:

  • [latex]\mathcal {F}[/latex] ist abgeschlossen.
  • [latex]\mathcal {F}[/latex] ist beschränkt.
  • [latex]\mathcal {F}[/latex] ist gleichstetig. Das heisst, es gibt für alle [latex]\varepsilon > 0[/latex] ein [latex]\delta > 0[/latex], so dass für alle [latex]f \in \mathcal {F}[/latex] und für alle [latex]x,y \in K[/latex] gilt [latex]\mathrm {d}(x,y)

Übung: Ein unendlich-dimensionaler vollständiger normierter Vektorraum

  1. Wir betrachten den Vektorraum
    [latex]
    \begin{aligned}[]\ell ^1 = \left \lbrace {(x_k)_k \in \mathbb {C}^\mathbb {N}} \mid {\sum _{k=1}^\infty x_k \text { konvergiert absolut}}\right \rbrace\end{aligned}
    [/latex]

    mit der Norm [latex]\| {(x_k)_k}\| _1 = \sum _{k=1}^\infty |x_k|[/latex]. Zeigen Sie, dass [latex]\ell ^1[/latex] vollständig ist. (Dies ist ein erstes Beispiel eines sogenannten Banachraums.)

  2. Zeigen Sie, dass der Unterraum [latex]U \subseteq \ell ^1[/latex] der Folgen, die schliesslich Null sind, dicht in [latex]\ell ^1[/latex] liegt und nicht vollständig ist.

Übung: Ein unendlich-dimensionaler, nicht-vollständiger, normierter Vektorraum

Sei [latex][a,b][/latex] ein kompaktes Intervall mit Endpunkten [latex]a9.12. Zeigen Sie, dass [latex]C([a,b])[/latex] mit dieser Norm nicht vollständig ist.

Übung

Seien [latex](V,\| {\cdot }\| _V), (W,\| {\cdot }\| _W)[/latex] normierte Vektorräume und sei [latex]F: V \to W[/latex] eine lineare Abbildung. Genau wie in Definition 9.75 setzen wir nun

[latex]
\begin{aligned}[]\| {F}\| _\mathrm {op} = \sup _{v \in V: \| {v}\| _V = 1} \| {F(v)}\| _W.\end{aligned}
[/latex]

Zeigen Sie, dass [latex]F[/latex] genau dann stetig ist, wenn [latex]\| {F}\| _\mathrm {op}

Hinweis.

Wenn [latex]\| {F}\| _\mathrm {op} = \infty[/latex] gilt, können Sie eine Folge [latex](v_n)_n[/latex] suchen mit [latex]v_n \to 0[/latex] und [latex]\| {F(v_n)}\| _W \to \infty[/latex] für [latex]n\to \infty[/latex]

Übung

Sei [latex]\alpha > 1[/latex]. Verwenden Sie den Banachschen Fixpunktsatz und dessen Beweis, um zu zeigen, dass die durch

[latex]
\begin{aligned}[]x_{n+1} = \frac {1}{x_n+\alpha } \text { für } n \geq 2,\ x_1 = 0\end{aligned}
[/latex]

rekursiv definierte Folge [latex](x_n)_n[/latex] konvergent ist.

Hinweis.

Betrachten Sie die Funktion [latex]f: x \in [0,1] \mapsto \frac {1}{x+\alpha } \in [0,1][/latex].

Übung

Zeigen Sie, dass es für jedes [latex]g \in C([0,1])[/latex] ein [latex]f \in C([0,1])[/latex] mit

[latex]
\begin{aligned}[]f(x) - \int _{0}^{x}e^{-y}f(y) \operatorname {d}\! x = g(x)\end{aligned}
[/latex]

gibt.

9.9.4 – Lernkarten

Sie können wiederum die Lernkarten oder den Graphen für Ihre Wiederholung der Themen des Kapitels verwenden.

<!– post meta –>


  1. Hier und auch im Folgenden können wir ebenso Vektorräume über [latex]\mathbb {C}[/latex] betrachten, doch inkludiert der Fall der reellen Vektorräume auch den Fall von komplexen Vektorräumen, weshalb wir Vektorräume über [latex]\mathbb {C}[/latex] hier und im Folgenden nicht mehr getrennt erwähnen werden.

License

Analysis-Skript CHAB MATH PHYS: 18/19 Copyright © by Manfred Einsiedler and Andreas Wieser. All Rights Reserved.

}